Está en la página 1de 176

Guía para la preparación del Examen Ser Bachiller

Asignatura de Lengua y Literatura

Elaborada por:

Victoria Novillo Rameix


Programa CLAVEMAT – Escuela Politécnica Nacional
2
3
Este texto es de autoría del Programa CLAVEMAT de la Escuela Politécnica Nacional. Contiene
resúmenes de los temas abordados frecuentemente en las formas Ser Bachiller de los últimos
años, y un banco de preguntas y respuestas con sus respectivas retroalimentaciones.

Esperamos que este material sirva como una guía para la preparación de las y los estudiantes al
Examen Ser Bachiller, según sus necesidades de aprendizaje.

Los temas desarrollados son independientes entre sí, por lo que no es necesario que su estudio
siga un orden secuencial.

4
Índice

Tema 1. Oraciones, párrafos y conectores 7


Texto resumen 7
Referencias bibliográficas 10
Banco de preguntas y respuestas 11
Tema 2. Reglas de concordancia 19
Texto resumen 19
Referencias bibliográficas 24
Banco de preguntas y respuestas 25
Tema 3. Tipos y estructura de los textos 30
Texto resumen 30
Referencias bibliográficas 56
Banco de preguntas y respuestas 56
Tema 4. Idea principal y consecuencia lógica de un texto 72
Texto resumen 72
Referencias bibliográficas 76
Banco de preguntas y respuestas 76
Tema 5. Niveles o tipos de uso del lenguaje 89
Texto resumen 89
Referencias bibliográficas 92
Banco de preguntas y respuestas 92
Tema 6. La paráfrasis 97
Texto resumen 97
Referencias bibliográficas 98
Banco de preguntas y respuestas 98
Tema 7. Sinónimos, antónimos y homónimos 102
Texto resumen 102
Referencias bibliográficas 119
Banco de preguntas y respuestas 120
Tema 8. Analogías entre palabras 135
Texto resumen 135
Referencias bibliográficas 136
Banco de preguntas y respuestas 136
Tema 9. Prefijos y sufijos 142
Texto resumen 142
Referencias bibliográficas 146
Banco de preguntas y respuestas 147
Tema 10. Elementos de la comunicación 150
Texto resumen 150
Referencias bibliográficas 152
Banco de preguntas y respuestas 152

5
Tema 11. Algunas reglas ortográficas 155
Texto resumen 155
Referencias bibliográficas 158
Banco de preguntas y respuestas 158
Tema 12. Léxico en Ecuador: quichuismos, arcaísmos, extranjerismos y neologismos 160
Texto resumen 160
Referencias bibliográficas 162
Banco de preguntas y respuestas 162
Tema 13. Uso adecuado de lo (s), la (s) y le (s) 165
Texto resumen 165
Referencias bibliográficas 166
Banco de preguntas y respuestas 166
Tema 14. Figuras literarias 168
Texto resumen 168
Referencias bibliográficas 171
Banco de preguntas y respuestas 172

6
Tema 1. Oraciones, párrafos y conectores

Texto resumen

Las oraciones simples y su construcción gramatical

La oración es una palabra o un conjunto de palabras con sentido completo, formada por un
sujeto y un predicado.

El sujeto de una oración corresponde a la persona, animal o cosa que realiza una acción. El
núcleo del sujeto es el sustantivo con su respectivo artículo, acompañado (aunque no siempre)
de un complemento que puede ser un adjetivo y/o un sintagma proposicional. A veces el sujeto
no es explícito.

El predicado es todo lo que se dice del sujeto. El verbo es el elemento principal del predicado;
sin embargo, a él se unen dos complementos: un complemento directo y/o un complemento
indirecto. El complemento directo es la persona, animal u objeto que concreta o limita el
significado del verbo. El complemento indirecto es la persona, animal u objeto beneficiario de
la acción nombrada por el verbo. El enunciado del complemento directo no necesariamente
debe ubicarse en una posición anterior a la del complemente indirecto.

Pongamos varios ejemplos:

SUJETO
PREDICADO

Sintagma Complemento Complemento


Artículo Sustantivo Adjetivo Verbo
preposicional directo indirecto
Expresa el
Califica o Complementa al Persona, animal u objeto Persona, animal u objeto
género y el Persona, animal
caracteriza sustantivo con una Acción del sujeto que concreta o limita el beneficiario de la acción
número del o cosa
al sustantivo preposición significado del verbo nombrada por el verbo
sustantivo

un trauma
1 Los médicos expertos del Hospital diagnosticaron al presidente.
encefálico

2 Carlos entregó un ramo de flores a Isabel.

3 Mis hermanos cariñosos viajaron en barco a Europa.

4 Yo solía cantar música nacional.

5 El niño pequeño de esa escuela lee cómics de héroes.

7
La oración compuesta

Una oración compuesta o compleja es un conjunto de palabras con sentido completo que tiene
dos o más verbos conjugados, a través del uso de nexos que pueden ser conectores (cuando es
una oración coordinada), signos de puntuación (cuando es una oración yuxtapuesta) o
preposiciones (cuando es una oración subordinada).

Oraciones coordinadas

Están integradas por dos o más oraciones simples que son sintácticamente independientes, es
decir, que podrían funcionar por separado pero que se integran en un mismo significado. Por
ejemplo: Juan y María vinieron y luego se fueron.

SUJETO PREDICADO

Nexo
Sintagma Complemento Complemento
Artículo Sustantivo Adjetivo Verbo
preposicional directo indirecto

1 Juan y María vinieron

2 y luego se fueron.

Oraciones yuxtapuestas

Están integradas por varias oraciones simples con un mismo valor sintáctico. Estas oraciones se
relacionan entre sí mediante signos de puntuación (coma, punto y coma o dos puntos). Por
ejemplo: El niño lloró, su madre también.

SUJETO PREDICADO

Nexo
Sintagma Complemento Complemento
Artículo Sustantivo Adjetivo Verbo
preposicional directo indirecto

1 El niño lloró

2 , su madre también (lloró).

8
Oraciones subordinadas

Están integradas por dos o más oraciones entre las cuales existe una relación de dependencia.
Esto quiere decir que hay una oración que se considera principal y de ella depende otra (u otras)
llamada "subordinada". Ejemplo: Estoy cansado de que me exploten.

SUJETO PREDICADO
Nexo
Sintagma Complemento Complemento
Artículo Sustantivo Adjetivo Verbo
preposicional directo indirecto

estoy
1 (Yo)
cansado

me
2 de que
exploten.

El párrafo

El párrafo es la mínima unidad de redacción de un texto, que explica y desarrolla el significado


de una idea. Un párrafo tiene diversas oraciones entrelazadas lógicamente entre sí mediante el
uso de signos de puntuación y conectores. Un párrafo bien escrito inicia con una oración
introductoria conectada con otras que la argumentan o la amplían. Así mismo, un párrafo
termina con una oración de cierre de la idea principal (explícita o implícita). Ejemplo:

Este era un hermoso castillo donde cada noche se celebraban fiestas llenas de esplendor, hasta
que una noche, cuando el rey nos visitaba y tomaba los alimentos, una enorme rata cayó del
techo justo en su sopa, manchándole toda la cara. Por ello, todos salieron llenos de pánico y
desde entonces ya nadie viene.1

Oración introductoria

Oraciones complementarias que amplían la oración introductoria, en orden cronológico

Conector

Oración final o conclusiva

Conectores gramaticales

Un conector es una palabra o un conjunto de palabras que une las oraciones de un párrafo y
establece una relación lógica entre ellas. Hay varios tipos de conectores gramaticales:

1 Tomado de: http://www.ejemplode.com/12-clases_de_espanol/1885-ejemplo_de_parrafo_narrativo.html

9
Tipo Conector Función Ejemplo

Porque, puesto que, ya que,


Llegué tarde porque el tráfico
Causa debido a que, a causa de que, Señala el motivo o razón.
estuvo fuerte.
en vista de que

Luego así que, de ahí que, de El Municipio anunció que no habrá


manera que, entonces, en Anuncian la consecuencia servicio de agua el día de mañana;
Consecuencia
conclusión, por lo tanto, de de algo. por lo tanto, no podrás bañarte a
modo que las 06h00.

Mas, pero, sino, por el


Contraste o contrario, sin embargo, más Señalan relaciones de No estudié; sin embargo, me fue
contradicción bien, en cambio, aunque, a oposición. bien en el examen.
pesar de

No creo que suban los precios de


Además, así mismo, inclusive, Añaden nuevas ideas los productos; además, ya
Adición
también, no sólo, sino sobre un mismo tema. anunciaron que los impuestos
bajarán.

En otras palabras, en otros


Anuncian una idea Fueron leales en todo, es decir, son
Equivalencia términos, o sea, vale decir, en
similar. buenos amigos.
suma

Todos podrán asistir a la fiesta;


Obviamente, como es obvio, sin Recuerdan un punto
Evidencia obviamente, están incluidos los
lugar a dudas, naturalmente. obvio.
niños.

Primero, en primer lugar, en


Enumeran una serie de En primer lugar, atenderemos a los
Orden segundo lugar, después,
puntos. niños, luego a los ancianos.
finalmente, por último

Indican una sucesión de Primero dedícate a estudiar, luego


Secuencia Después, luego, en seguida
ideas. piensa en casarte.

Referencias bibliográficas

• Culebra y Vives, Cecilia (2002). Taller de lectura y redacción II. Cuernavaca: CENIDET. Disponible en:
http://www.cenidet.edu.mx/subaca/web-dda/docs/TLR2Completo.pdf

• Conectores gramaticales. En: http://www.palabrasylibros.com/sitio/contenidos_mo_comentarios.php?it=6

10
Banco de preguntas y respuestas

1)

Con las siguientes palabras, construye una oración simple:

1. Un partido
2. Los vecinos
3. Jugaron
4. De mis tíos

a) 1, 4, 2, 3
b) 2, 1, 4, 3
c) 2, 4, 3, 1
d) 4, 3, 2, 1

Respuesta correcta: c) 2, 4, 3, 1.

La oración es un conjunto de palabras con sentido completo. La manera más sencilla de ordenar las palabras responde
a un esquema que, en la medida de lo posible, debemos seguir: sujeto (artículo + sustantivo + adjetivo + sintagma
preposicional) + predicado (verbo + complemento directo + complemento indirecto).

En este caso, el orden correcto de la oración sería: Los vecinos de mis tíos jugaron un partido. Podemos visualizarlo
así:

SUJETO PREDICADO

Sintagma Complemento Complemento


Artículo Sustantivo Adjetivo Verbo
preposicional directo indirecto

Los vecinos de mis tíos jugaron un partido.

2)

Ordene los elementos para formar una oración:

1. La degustación
2. De los condimentos
3. Con alterar
4. El sabor
5. De los comensales
6. Amenaza

a) 1, 2, 4, 3, 6, 5
b) 1, 5, 2, 6, 3, 4
c) 4, 2, 6, 3, 1, 5
d) 4, 5, 3, 6, 2, 1

Respuesta correcta: c) 4, 2, 6, 3, 1, 5

La oración es un conjunto de palabras con sentido completo. La manera más sencilla de ordenar las palabras responde
a un esquema que, en la medida de lo posible, debemos seguir: sujeto (artículo + sustantivo + adjetivo + sintagma
preposicional) + predicado (verbo + complemento directo + complemento indirecto).

11
En este caso, el orden correcto de la oración sería: El sabor de los condimentos amenaza con alterar la degustación
de los comensales. Podemos visualizarlo así:

SUJETO PREDICADO
Sintagma Complemento Complemento
Artículo Sustantivo Adjetivo Verbo
preposicional directo indirecto

amenaza con
El sabor de los condimentos la degustación de los comensales.
alterar

3)

Ordene los elementos para formar una oración:

1. La arrogancia
2. En las personas
3. Con alterar
4. Amenaza
5. De la sociedad
6. La solidaridad

a) 1, 2, 4, 3, 6, 5
b) 1, 5, 2, 6, 3, 4
c) 4, 2, 6, 3, 1, 5
d) 4, 5, 3, 6, 2, 1

Respuesta correcta: a) 1, 2, 4, 3, 6, 5

La oración es un conjunto de palabras con sentido completo. La manera más sencilla de ordenar las palabras responde
a un esquema que, en la medida de lo posible, debemos seguir: sujeto (artículo + sustantivo + adjetivo + sintagma
preposicional) + predicado (verbo + complemento directo + complemento indirecto).

En este caso, el orden correcto de la oración sería: La arrogancia en las personas amenaza con alterar la solidaridad
de la sociedad. Podemos visualizarlo así:

SUJETO PREDICADO

Sintagma Complemento Complemento


Artículo Sustantivo Adjetivo Verbo
preposicional directo indirecto

amenaza con
La arrogancia en las personas la solidaridad de la sociedad.
alterar

4)

Ordene los elementos para formar una oración:

1. La fraternidad
2. En las personas
3. Con alterar
4. La presunción
5. De la sociedad
6. Amenaza

12
a) 1, 2, 4, 3, 6, 5
b) 1, 5, 2, 6, 3, 4
c) 4, 2, 6, 3, 1, 5
d) 4, 5, 3, 6, 2, 1

Respuesta correcta: c) 4, 2, 6, 3, 1, 5

La oración es un conjunto de palabras con sentido completo. La manera más sencilla de ordenar las palabras responde
a un esquema que, en la medida de lo posible, debemos seguir: sujeto (artículo + sustantivo + adjetivo + sintagma
preposicional) + predicado (verbo + complemento directo + complemento indirecto).

En este caso, el orden correcto de la oración sería: La presunción en las personas amenaza con alterar la fraternidad
de la sociedad. Podemos visualizarlo así:

SUJETO PREDICADO
Sintagma Complemento Complemento
Artículo Sustantivo Adjetivo Verbo
preposicional directo indirecto

amenaza con
La presunción en las personas la fraternidad de la sociedad.
alterar

5)

Ordene las proposiciones según la sintaxis de las oraciones simples:

1. Con ternura
2. A sus crías
3. Amamantaba
4. Una cebra

a) 1, 3, 2, 4
b) 2, 1, 4, 3
c) 2, 3, 4, 1
d) 4, 3, 2, 1

Respuesta correcta: d) 4, 3, 2, 1

La oración es un conjunto de palabras con sentido completo. La manera más sencilla de ordenar las palabras responde
a un esquema que, en la medida de lo posible, debemos seguir: sujeto (artículo + sustantivo + adjetivo + sintagma
preposicional) + predicado (verbo + complemento directo + complemento indirecto).

En este caso, el orden correcto de la oración sería: Una cebra amamantaba con ternura a sus crías. Podemos
visualizarlo así:

SUJETO PREDICADO

Sintagma Complemento Complemento


Artículo Sustantivo Adjetivo Verbo
preposicional directo indirecto

Una cebra amamantaba con ternura a sus crías.

13
6)

Ordene las proposiciones según la sintaxis de las oraciones simples:

1. Con dulzura
2. Una vaca
3. A sus becerros
4. Alimentaba

a) 1, 3, 2, 4
b) 2, 1, 4, 3
c) 2, 4, 3, 1
d) 4, 3, 2, 1

Respuesta correcta: c) 2, 4, 3, 1

La oración es un conjunto de palabras con sentido completo. La manera más sencilla de ordenar las palabras responde
a un esquema que, en la medida de lo posible, debemos seguir: sujeto (artículo + sustantivo + adjetivo + sintagma
preposicional) + predicado (verbo + complemento directo + complemento indirecto).

El orden correcto de la oración sería: Una vaca alimentaba a sus becerros con dulzura. En este caso, el complemento
indirecto se ubica en una posición anterior a la del complemento directo (pues no hay una alternativa de respuesta
en la cual dicho orden sea contrario). Podemos visualizarlo del siguiente modo:

SUJETO PREDICADO
Sintagma Complemento Complemento
Artículo Sustantivo Adjetivo Verbo
preposicional indirecto directo

Una vaca alimentaba a sus becerros con dulzura.

7)

Ordene los enunciados para formar una oración compuesta:

1. Piensa contarle
2. Antonio estudia
3. Mientras espera a su madre
4. Que le asignaron una beca
5. Literatura e historia

a) 2, 3, 1, 4, 5
b) 2, 5, 3, 1, 4
c) 5, 2, 3, 4, 1
d) 5, 3, 1, 4, 2

Respuesta correcta: b) 2, 5, 3, 1, 4

Una oración compuesta o compleja es un conjunto de palabras con sentido completo que tiene dos o más verbos
conjugados, a través del uso de signos de puntuación o de nexos (conectores o preposiciones).

En este caso, la oración está compuesta por cuatro verbos conjugados. Su orden coherente de construcción sería:
Antonio estudia literatura e historia. Mientras espera a su madre, piensa contarle que le asignaron una beca.
Veámoslo en este cuadro:

14
SUJETO PREDICADO

Nexo Sintagma Complemento Complemento


Artículo Sustantivo Adjetivo Verbo
preposicional directo indirecto

literatura e
1 Antonio estudia
historia.

2 Mientras espera a su madre

piensa
3 ,
contarle

le
4 que una beca.
asignaron

8)

Ordene los enunciados para formar una oración compuesta:

1. El premio
2. De que ganaría
3. El actor
4. Parecía seguro
5. En el festival

a) 1, 2, 4, 5, 3
b) 1, 3, 5, 2, 4
c) 3, 4, 2, 1, 5
d) 4, 3, 1, 2, 5

Respuesta correcta: c) 3, 4, 2, 1, 5

Una oración compuesta o compleja es un conjunto de palabras con sentido completo que tiene dos o más verbos
conjugados, a través del uso de signos de puntuación o de nexos (conectores o preposiciones).

En este caso, la oración está compuesta por dos verbos conjugados. Su orden coherente de construcción sería: El
actor parecía seguro de que ganaría el premio en el festival. Veámoslo en este cuadro:

SUJETO PREDICADO
Nexo Sintagma Complemento Complemento
Artículo Sustantivo Adjetivo Verbo
preposicional directo indirecto

parecía
1 El actor
seguro

2 de que ganaría el premio en el festival.

15
9)

¿Cómo deben organizarse los elementos para formar una oración coherente?

1. Su abuelita
2. Le gusta
3. Que prepara
4. A Carmen
5. La sopa

a) 1, 2, 4, 5, 3
b) 3, 2, 5, 4, 1
c) 4, 2, 5, 3, 1
d) 5, 4, 1, 2, 3

Respuesta correcta: c) 4, 2, 5, 3, 1

Una oración compuesta o compleja es un conjunto de palabras con sentido completo que tiene dos o más verbos
conjugados, a través del uso de signos de puntuación o de nexos (conectores o preposiciones).

En este caso, la oración está integrada por dos verbos conjugados. Su orden coherente de construcción sería: A
Carmen le gusta la sopa que prepara su abuelita. Veámoslo en este cuadro:

SUJETO PREDICADO
Nexo
Sintagma Complemento Complemento
Artículo Sustantivo Adjetivo Verbo
preposicional directo indirecto

1 A Carmen le gusta la sopa

2 que prepara su abuelita.

10)

Ordene lógicamente los enunciados para formar un párrafo acerca de la historia del perro labrador:

1. Al llegar a Inglaterra, en el siglo XIX, estos perros fueron llamados labradores debido a su origen canadiense.
2. El precursor de esta raza fue el perro de agua de San Juan, surgido en la cría hecha por colonos isleños de
Terranova en el siglo XVI.
3. Actualmente, el labrador es bastante popular debido a su carácter (inteligencia y docilidad) y trabajo canino
como guía rescatista.
4. El labrador retriever, cobrador o perdiguero de labrador, es una raza canina originaria de Terranova, en
Canadá.
5. Edwin Landseer retrató en 1823 a una perra labrador, mientras que la revista Life dedicó su portada al
labrador Blinf of Arden en 1932.

a) 1, 5, 2, 3, 4
b) 2, 1, 3, 4, 5
c) 3, 5, 4, 1, 2
d) 4, 2, 1, 5, 3

Respuesta correcta: d) 4, 2, 1, 5, 3

16
Para formar un párrafo a partir de enunciados sueltos hay que ordenarlos lógicamente, es decir, de tal forma que la
conexión de uno con otro permita una lectura coherente. Un párrafo bien escrito inicia con una oración introductoria
conectada con otras que la argumentan o la amplían. Así mismo, un párrafo termina con una oración de cierre de la
idea principal (explícita o implícita).

En este caso, el párrafo completo se enunciaría del siguiente modo:

El labrador retriever, cobrador o perdiguero de labrador, es una raza canina originaria de Terranova, en Canadá. El
precursor de esta raza fue el perro de agua de San Juan, surgido en la cría hecha por colonos isleños de Terranova en
el siglo XVI. Al llegar a Inglaterra, en el siglo XIX, estos perros fueron llamados labradores debido a su origen
canadiense. Edwin Landseer retrató en 1823 a una perra labrador, mientras que la revista Lifededicó su portada al
labrador Blinf of Arden en 1932. Actualmente, el labrador es bastante popular debido a su carácter (inteligencia y
docilidad) y trabajo canino como guía rescatista.

Oración introductoria
Oraciones complementarias que amplían la oración introductoria, en orden cronológico
Conector
Oración final o conclusiva

11)

Complete la oración con un conector que señale contradicción:

La muerte no nos roba los seres amados, _______ nos los guarda y los inmortaliza en el recuerdo.

a) además
b) pues
c) sea
d) al contrario


Respuesta correcta: d)

Los conectores de contraste o contradicción se usan para señalar relaciones de oposición entre dos ideas. En este
caso: La muerte no nos roba los seres amados, al contrario, nos los guarda y los inmortaliza en el recuerdo.

12)

Complete la oración con su conector correspondiente.

Jorge no puede asistir a la clase de Filosofía _______ se siente con mucho dolor de cabeza.

a) además
b) porque
c) sin embargo
d) finalmente

Respuesta correcta: b)

La oración citada expresa que Jorge no puede asistir a su clase de filosofía y luego explica la razón del por qué no lo
hará: se siente con mucho dolor de cabeza. El conector correcto corresponde a un conector de causa: porque. Así:
Jorge no puede asistir a la clase de Filosofía porque se siente con mucho dolor de cabeza.

17
13)

Complete la frase con su conector correspondiente:

¿Qué te parece si vamos al cine a ver «La era del hielo» que está calificada «para todo público»? __________ puede
venir tu hijo Jorge, de 9 años.

a) Además
b) No obstante
c) Obviamente
d) Pues

Respuesta correcta: c)

La frase citada plantea que La era de hielo está calificada como una película “para todo público” y que, en ese sentido,
puede ser vista por un niño de 9 años. El conector adecuado de dicha frase correspondería a un conector de evidencia
(conector que permite recordar un punto obvio). Así: ¿Qué te parece si vamos al cine a ver «La era del hielo» que
está calificada «para todo público»? Obviamente, puede venir tu hijo Jorge, de 9 años.

14)

Complete la oración con un conector que señale adición:

No creo que suframos un terremoto; _______, nadie puede prever una catástrofe de ese calibre.

a) además
b) pues
c) sin embargo
d) al contrario


Respuesta correcta: a)

En la oración citada podría caber un conector de causa como pues. ¿Por qué? Porque una de las causas para no
pensar que podamos sufrir un terremoto es que nadie puede preverlos. Sin embargo, la pregunta dice claramente
que completemos la oración con un conector que señale adición. Los conectores de adición permiten añadir nuevas
ideas sobre un mismo tema, justificándolo. Y, en este caso, cabe perfectamente el conector “además” (inclusive por
los signos de puntuación empleados): No creo que suframos un terremoto; además, nadie puede prever una
catástrofe de ese calibre.

18
Tema 2. Reglas de concordancia

Texto resumen

Según la Real Academia de la Lengua Española, la concordancia es la “coincidencia obligada de


determinados accidentes gramaticales (género, número y persona) en los elementos variables
de la oración”2.

Hay dos tipos de concordancia: nominal y verbal.

La concordancia nominal

Consiste en la coincidencia entre sustantivo y adjetivo. La regla general es la siguiente: cuando


un sustantivo y su artículo va acompañado de un adjetivo, éstos deben coincidir en género y
número.

Esas mujeres
luchadoras me
provocan mucha
admiración.

Hay reglas específicas de concordancia nominal:

✓ Cuando un adjetivo califica a dos o más sustantivos de distinto género y va pospuesto a ellos,
lo más recomendable es que aquel vaya en plural y en masculino.

Juana Inés apareció con


traje y mantilla blancos.

✓ Cuando un adjetivo califica a varios sustantivos de distinto género y va antepuesto a ellos, lo


normal es que concuerde solo con el más próximo, tanto en género como en número.

2 Real Academia de la Lengua. Concordancia. En: http://lema.rae.es/dpd/srv/search?id=XEVeLzVZaD6CG25cW5. De aquí en


adelante, en este capítulo se incluyen ejemplos tomados dicho enlace.

19
La necesaria seguridad y
control nocturnos brillan
por su ausencia.

✓ Cuando varios numerales ordinales coordinan a un mismo sustantivo y van pospuestos a


este, lo normal es que dicho sustantivo vaya en plural.

El ascensor llegó abarrotado desde los


sótanos primero y segundo.

✓ Cuando un adjetivo va pospuesto a dos o más sustantivos unidos por la conjunción “o” y esta
denota exclusión o contraposición, dicho adjetivo debe ir en plural y en masculino.

Cada vez que mueren un hombre


o una mujer viejos, toda una
biblioteca muere con ellos.

✓ Cuando un adjetivo va pospuesto a dos o más sustantivos de distinto género unidos por la
conjunción “o” y esta denota identidad o equivalencia, dicho adjetivo debe ir en singular y
en masculino.

Ese cántico o melodía hermoso


debería ser presentado en una
nueva función.

20
Concordancia verbal

Consiste en la coincidencia entre el verbo y su sujeto. La regla general es la siguiente: el núcleo


del sujeto y el verbo deben coincidir en número y en persona.

También hay casos específicos de concordancia verbal:

✓ Cuando el núcleo del sujeto es un sustantivo colectivo unido a un sustantivo en plural, el


verbo se usa en singular.

✓ Cuando dos sustantivos están asociados como una unidad por un mismo artículo, y si el
núcleo del predicado corresponde a la forma ser o estar + verbo en participio (adjetivo), su
conjugación se usa en singular y respetando el género del artículo.

✓ Cuando dos o más sustantivos de un mismo género están separados porque tienen, cada
uno, su propio artículo, y si el núcleo del predicado corresponde a la forma ser o estar + verbo
en participio (adjetivo), su conjugación se usa en plural y respetando el género de los
sustantivos.

21
✓ Cuando dos o más sustantivos de diferente género están separados porque tienen, cada uno,
su propio artículo, y si el núcleo del predicado corresponde a la forma ser o estar + verbo en
participio (adjetivo), su conjugación se usa en plural y en masculino.

✓ Cuando a un sustantivo en singular le sigue otro, asociado a él mediante los nexos además
de, así como o al igual que, el verbo va en singular.

✓ Cuando el nexo que se une a un sustantivo funciona como conjunción copulativa (enlace)
dando lugar a un sujeto plural, el verbo se usa en plural.

22
✓ Cuando dos sustantivos están unidos por tanto... como, el verbo debe ir en plural.

✓ Cuando a un sustantivo en singular le preceden varios adjetivos ordinales, el verbo irá en


plural.

✓ Cuando el sustantivo es colectivo (gente, clero, familia, alumnado y otros), el verbo debe ir
en singular, así como el artículo, pronombres o adjetivos a él referidos.

23
✓ Cuando el sujeto se compone de sustantivo cuantificador + de + sustantivo en plural, el verbo
puede ir en singular o en plural pues cualquiera de los dos sustantivos puede considerarse el
núcleo del sujeto.

✓ Cuando el pronombre le o les refuerza o reemplaza el complemento indirecto (es decir, un


modificador del verbo que indica el destinatario de la acción), dicho pronombre debe
coincidir en número con el núcleo de este complemento. Además, el verbo que lo acompaña
debe guardar coherencia en número con el complemento directo.

Referencias bibliográficas

• Real Academia de la Lengua Española. Concordancia. En:


http://lema.rae.es/dpd/srv/search?id=XEVeLzVZaD6CG25cW5

24
Banco de preguntas y respuestas

1)

Son oraciones que cumplen las reglas de concordancia nominal y verbal, excepto 3:

a) Las piñas, limas, peras y manzanas verdes son las preferidas de Juan
b) Le informaron que Manuel, Sandra, Lorena y Gonzalo saldrán del país
c) El sueño, el anhelo y el vuelo imaginario es descartado por los pragmáticos
d) La montaña y el volcán congelados, mientras el imponente sol derretía el día

Respuesta correcta: c)

De las frases enlistadas, una NO cumple con las reglas generales de concordancia: “El sueño, el anhelo y el vuelo
imaginario es descartado por los pragmáticos”. ¿Por qué?

Porque, cuando dos sustantivos están separados debido a que tienen, cada uno, su propio artículo, el verbo se usa
en plural. La frase, por tanto, debería estar escrita del siguiente modo: “El sueño, el anhelo y el vuelo imaginarios son
descartados por los pragmáticos”.

2)

Son oraciones que cumplen las reglas de concordancia nominal y verbal, excepto:

a) Carlos en la noche disfruta mirar películas, series, telenovelas y documentales


b) Lucas, María y Pedro me informaron que van a renunciar a sus cargos
c) La luna, el sol y las estrellas ha sido fuentes de inspiración para los poetas
d) La luna y el sol oscurecidos, mientras la bella Tierra iluminaba el escenario

Respuesta correcta: c)

De las frases enlistadas, una NO cumple con las reglas generales de concordancia: “La luna, el sol y las estrellas ha
sido fuentes de inspiración para los poetas”. ¿Por qué?

Porque, cuando dos sustantivos están separados debido a que tienen, cada uno, su propio artículo, el verbo se usa
en plural. La frase, por tanto, debería estar escrita del siguiente modo: “La luna, el sol y las estrellas han sido fuentes
de inspiración para los poetas”.

3)

Son oraciones que cumplen las reglas de concordancia nominal y verbal, excepto:

a) Ismael en el colegio estudia las ranas, mapaches, leones y tigres


b) Fernando y Gabriel me informaron que abandonarán la aerolínea internacional
c) El elefante, el lagarto y el zorro ha sido observados por los científicos del refugio
d) La policía y la azafata sorprendidos; mientras que el piloto se veía alegre

Respuesta correcta: c)

De las frases enlistadas, una no cumple con las reglas generales de concordancia: “El elefante, el lagarto y el zorro ha
sido observados por los científicos del refugio”. ¿Por qué?

3Preguntas formuladas de esta manera son muy comunes en las formas Ser Bachiller, aun cuando puedan resultar algo confusas
para algunas y algunos estudiantes. Desde el MINEDUC, los expertos proponen formulaciones como esta: Subraye la oración que no
cumple las reglas de concordancia nominal y verbal. Estamos de acuerdo con su propuesta; sin embargo, es importante que las
chicas y los chicos se familiaricen con los modos de escritura establecidos en los exámenes Ser Bachiller.

25
Porque, cuando dos sustantivos están separados debido a que tienen, cada uno, su propio artículo, el verbo se usa
en plural. La frase, por tanto, debería estar escrita del siguiente modo: “El elefante, el lagarto y el zorro han sido
observados por los científicos del refugio”.

4)

Complete el enunciado:

Mónica comentó que a ella le ________ mucho los conejos; aunque su familia, siempre atareada con ocupaciones
y negocios, ___________ no llevar mascotas a tan ocupado hogar.

a) gustan – prefiere
b) gusta – prefiere
c) gustan – prefieren
d) gusta – prefieren

Respuesta correcta: a)

En esta frase debemos aplicar dos reglas de concordancia verbal:

• Cuando el pronombre le o les refuerza o reemplaza el complemento indirecto (es decir, un modificador del verbo
que indica el destinatario de la acción), dicho pronombre debe coincidir en número con el núcleo de este
complemento. Además, el verbo que lo acompaña, debe guardar coherencia en número con el complemento
directo: Mónica comentó que a ella le gustan (verbo en plural) mucho los conejos (complemento directo en
plural).

• El núcleo del sujeto y el verbo deben coincidir en número y en persona: Su familia (núcleo del sujeto en singular),
siempre atareada con ocupaciones y negocios, prefiere (verbo en singular) no llevar mascotas a tan ocupado
hogar.

La opción correcta, por tanto, es “gustan – prefiere”: Mónica comentó que a ella le gustan mucho los conejos; aunque
su familia, siempre atareada con ocupaciones y negocios, prefiere no llevar mascotas a tan ocupado hogar.

5)

Complete el enunciado:

Enrique comentó que a él le ________ mucho los autos clásicos; aunque su grupo de amigos, fascinados por el rally,
se ___________ por los autos de carreras.

a) encantan - inclina
b) encanta - inclina
c) encantan - inclinan
d) encanta - inclinan

Respuesta correcta: c)

En esta frase debemos aplicar una regla de concordancia verbal: cuando los pronombres le o les, me, se o te refuerzan
o reemplazan el complemento indirecto (es decir, un modificador del verbo que indica el destinatario de la acción),
dicho pronombre debe coincidir en número con el núcleo de este complemento. Además, el verbo que lo acompaña,
debe guardar coherencia en número con el complemento directo. Así:

• Enrique comentó que a él le encantan (verbo en plural) mucho los autos clásicos (complemento directo en
plural).
• Su grupo de amigos, fascinados por el rally, se inclinan (verbo en plural) por los autos de carreras
(complemento directo en plural).

26
La opción correcta, por tanto, es “encantan – inclinan”. La oración se escribiría así: Enrique comentó que a él le gustan
mucho los autos clásicos; aunque su grupo de amigos, fascinados por el rally, se inclinan por los autos de carreras.

6)

Complete el texto:

El tejido, que es el arte de crear textiles y piezas decorativas, __________ en ese lugar con la labor de educar a los
pequeños; así lo ____________ la comunidad de mujeres tejedoras de los valles interandinos.

a) Convivía – declararon
b) Convivían – declaró
c) Convivían – declararon
d) Convivía – declaró

Respuesta correcta: d)

En esta frase debemos aplicar dos reglas de concordancia verbal:

• El núcleo del sujeto y el verbo deben coincidir en número y en persona: El tejido (núcleo del sujeto en singular),
que es el arte de crear textiles y piezas decorativas, convivía (verbo conjugado en singular) en ese lugar con la
labor de educar a los pequeños.
• Cuando el sustantivo es colectivo (gente, clero, familia, alumnado y otros), el verbo debe ir en singular, así como
el artículo, pronombres o adjetivos a él referidos: así lo declaró (verbo en singular) la comunidad (sustantivo
colectivo) de mujeres tejedoras de los valles interandinos.

La opción correcta, por tanto, es “convivía – declaró”: El tejido, que es el arte de crear textiles y piezas decorativas,
convivía en ese lugar con la labor de educar a los pequeños; así lo declaró la comunidad de mujeres tejedoras de los
valles interandinos.

7)

Complete la oración:

Las investigaciones sobre destrucción de los bosques, ___________ en la selva tropical, ___________ adoptar
nuevas medidas correctivas con resultados eficientes que pueden replicarse en otras regiones del mundo.

a) Realizado – ha permitido
b) Realizadas – han permitido
c) Realizadas – ha permitido
d) Realizado – han permitido

Respuesta correcta: b)

En esta oración debemos aplicar dos reglas, una de concordancia nominal y otra de concordancia verbal:

• Cuando un sustantivo y su artículo va acompañado de un adjetivo, éstos deben coincidir en género y número: Las
investigaciones (sustantivo femenino en plural) sobre destrucción de bosques, realizadas (adjetivo o forma no
personal de un verbo que hace las veces de adjetivo, en femenino y en plural) en la selva tropical.

• El núcleo del sujeto y el verbo deben coincidir en número: Las investigaciones (sustantivo en plural) . . . han
permitido (conjugación del verbo en plural).

La opción correcta, por tanto, es “realizadas – han permitido”: Las investigaciones sobre destrucción de bosques,
realizadas en la selva tropical, han permitido adoptar nuevas medidas correctivas con resultados eficientes que
pueden replicarse en otras regiones del mundo.

27
8)

Complete la oración:

Se ha descubierto un nuevo tipo de medusa. El hábitat de estas medusas _______ amplias extensiones de las costas
de California. Un aspecto innovador de esta especie es que su cuerpo está cubierto de granos que _______ para
capturar sus presas.

a) comprenden – utilizan
b) comprenden – utiliza
c) comprende – utilizan
d) comprende – utiliza

Respuesta correcta: d)

En esta oración debemos aplicar una regla de concordancia verbal: El núcleo del sujeto y el verbo deben coincidir en
número. Así:

• El hábitat (núcleo del sujeto en singular) de estas medusas comprende (verbo en singular) amplias extensiones
de las costas de California.

• Su cuerpo (núcleo del sujeto en singular) está cubierto de granos que utiliza (verbo en singular) para capturar sus
presas.

La frase completa quedaría así: Se ha descubierto un nuevo tipo de medusa. El hábitat de estas medusas comprende
amplias extensiones de las costas de California. Un aspecto innovador de esta especie es que su cuerpo está cubierto
de granos que utiliza para capturar sus presas.

9)

Complete la oración:

Al pingullero, por su papel central, los danzantes le________ “mamaco”, según nos _________ algunas personas
del Valle de los Chillos. Existe un tono cuando los danzantes visitan la casa de los priostes o de los cabecillas del
grupo, para cuando caminan a la iglesia, y, principalmente, para la procesión.

a) dicen – han referido


b) dice – han referido
c) dicen – ha referido
d) dice – ha referido

Respuesta correcta: a)

En esta oración debemos procurar una concordancia entre el verbo con el género y número del sustantivo. Si
hablamos de “danzantes” como sustantivo plural masculino, su verbo deberá conjugarse en plural: Los danzantes le
dicen “mamaco”. Y si, además, se emplea como sustantivo plural “algunas personas” su verbo deberá conjugarse en
plural: nos han referido algunas personas.

La oración completa quedaría así: Al pingullero, por su papel central, los danzantes le dicen “mamaco”, según nos
han referido algunas personas del Valle de los Chillos. Existe un tono cuando los danzantes visitan la casa de los
priostes o de los cabecillas del grupo, para cuando caminan a la iglesia, y, principalmente, para la procesión.

10)

Complete la oración:

En un momento dado, Manuela decidió ingresar al Conservatorio Nacional de Música. Quiso aprender a tocar el
piano, la guitarra y el saxofón, instrumentos que creía __________ de manejar. Pero nada. Algo hizo que se

28
paralizara cada vez que intentaba ___________. Una vez me dijeron: no creo que tocar música, cantar y bailar ____
lo suyo.

a) fácil – tocarlo – sean


b) fáciles – tocarlos – sean
c) fáciles - tocarlos - sea
d) fácil – tocarlo – sea

Respuesta correcta: c)

En esta oración debemos aplicar dos reglas de concordancia, una nominal y otra verbal:

• Cuando un sustantivo y su artículo va acompañado de un adjetivo, estos deben coincidir en género y número:
Quiso aprender a tocar el piano, la guitarra y el saxofón, instrumentos (sustantivo en plural) que creía fáciles
(adjetivo en plural) de manejar.

• Cuando dos o más infinitivos (elementos gramaticalmente neutros) anteceden a la conjugación del verbo ser, éste
debe aparecer en singular: no creo que tocar música, cantar y bailar sea lo suyo.

La oración completa quedaría así:

En un momento dado, Manuela decidió ingresar al Conservatorio Nacional de Música. Quiso aprender a tocar el piano,
la guitarra y el saxofón, instrumentos que creía fáciles de manejar. Pero nada. Algo hizo que se paralizara cada vez
que intentaba tocarlos. Una vez me dijeron: no creo que tocar música, cantar y bailar sea lo suyo.

11)

Complete la oración:

El colibrí, así como otros pájaros de la Reserva Ecológica Yanacocha, _________ bajo cuidados especiales. En esta
reserva ___________ la entrada y salida de vehículos, pues su sonido se convierte en un elemento distractor para
dicha ave.

a) Permanece – están prohibidas


b) Permanecen – está prohibida
c) Permanecen – están prohibidas
d) Permanece – está prohibida

Respuesta correcta: d)

En esta oración debemos aplicar dos reglas de concordancia verbal:

• Cuando a un sustantivo en singular le sigue otro, asociado a él mediante los nexos además de, así como o al igual
que, el verbo va en singular: El colibrí (sustantivo en singular), así como (nexo) otros pájaros de la Reserva
Ecológica Yanacocha, permanece (verbo en singular) bajo cuidados especiales.

• Cuando dos sustantivos están asociados como una unidad por un mismo artículo, y si el núcleo del predicado
corresponde a la forma ser o estar + verbo en participio (adjetivo), su conjugación se usa en singular y respetando
el género del artículo: En esta reserva está prohibida (forma ser o estar + verbo en participio, en singular y en
femenino) la entrada y salida (sustantivos asociados como una unidad por un mismo artículo femenino, en
singular) de vehículos, pues su sonido se convierte en un elemento distractor para dicha ave.

La oración completa quedaría así: El colibrí, así como otros pájaros de la Reserva Ecológica Yanacocha, permanece
bajo cuidados especiales. En esta reserva está prohibida la entrada y salida de vehículos, pues su sonido se convierte
en un elemento distractor para dicha ave.

29
Tema 3. Tipos y estructura de los textos

Texto resumen

Un texto es el conjunto coherente de enunciados (orales o escritos) con una intención


comunicativa. Hay varias modalidades de texto, es decir, varias formas que escoge un hablante
o un escritor para organizar los contenidos de aquello que comunica: la argumentación, la
descripción, la exposición, la narración, la estética poética o el diálogo. Dependiendo de estas
modalidades, se pueden distinguir varios tipos de textos:

a. El texto argumentativo

Es un texto cuyo autor plantea una tesis sobre determinados hechos o comportamientos,
refutándola con planteamientos contrarios y proponiendo diversos argumentos como estrategia
para convencer a sus oyentes o lectores sobre los contenidos de dicha tesis.

Este texto suele organizarse en tres apartados: introducción, desarrollo y conclusión. La


introducción consiste en una breve exposición de la tesis del autor -es decir, de su postura
particular respecto de un hecho o comportamiento- cuyo propósito es captar la atención de los
receptores. El desarrollo o cuerpo del texto consiste en el argumento que avala dicha tesis, sea
un argumento racional (verdad admitida socialmente), un argumento de hecho (una prueba
comprobable), un argumento de ejemplificación (ejemplos concretos), o un argumento de
autoridad (opiniones de personas de reconocido prestigio). La conclusión es una síntesis de las
opiniones del hablante o escritor.

Partes de un texto argumentativo

Introducción
Breve exposición de la tesis

Desarrollo Conclusión
Cuerpo del texto o argumento de
Síntesis de la tesis
la tesis

30
Hay varios tipos de textos argumentativos:

• El ensayo

Del género literario, el ensayo es un escrito en el cual un autor desarrolla sus ideas basadas
en argumentos, pero sin necesidad de mostrar erudición, es decir, de una manera libre y
personal. El ensayo combina tanto la exposición como la argumentación. Por eso puede
concebirse como un tipo de texto mixto (expositivo y argumentativo) y sin una estructura
definida, pues el autor escoge el orden en que desarrolla su argumento. Su extensión
depende del autor y está dirigido a públicos amplios. 4

desarrolla ideas
argmentadas,
pero de forma
libre y
espontánea

su extensión combina la
depende del Ensayo exposición y la
autor argumentación

el autor escoge
el orden en que
desarrolla su
argumento

Proponemos un ejemplo de un ensayo corto:

La pena de muerte

Un tema bastante polémico es si una persona tiene el derecho de terminar o no con su propia
vida. La Asociación Médica Mundial define a la eutanasia de la siguiente forma: “El acto
deliberado de poner fin a la vida de un paciente". Ese derecho a vivir, es un derecho humano.
¿Quién tiene derecho a decir cuándo debe acabar esa vida? La respuesta más lógica es simple.
Cada uno es dueño de su propia salud y por ende, está en su facultad de decidir si se auto
inflige daño o si termina con su existencia.

4 Cáceres Ramírez, Orlando (2016). ¿Qué es un ensayo? Tomado de: https://www.aboutespanol.com/que-es-un-ensayo-2879495

31
La postura a favor de “asistir al paciente” si este desea poner fin a su vida puede resumirse
en una única idea: la libertad. La libertad del paciente de poder tomar la decisión de no
detener su sufrimiento. Quienes argumentan en contra sostienen que la vida humana es
inviolable y que la eutanasia equivale a un asesinato asistido. Afirman también que la vida
debe ser preservada, a pesar de que las condiciones en que se encuentre la salud de la
persona. 5

En este caso, la tesis del autor del ensayo es que “cada uno es dueño de su propia salud y,
por ende, está en su facultad de decidir si se auto inflige daño o si termina con su existencia”.
El argumento: su derecho a la libertad.

• El editorial

Del género periodístico, el editorial es un artículo que recoge la opinión institucional y


colectiva del periódico o revista y, por ello, no lleva la firma de ninguna persona. Ese carácter
institucional otorga a este tipo de artículos una gran trascendencia pública. Los lectores
pueden conocer la opinión abierta y directa del medio sobre distintos temas de actualidad,
así como sus planteamientos ideológicos implícitos. Todos los editoriales opinan acerca de
temas de actualidad (políticos, económicos, sociales o culturales) que entrañan una mayor
trascendencia y una gran importancia.

recoge la
opinión
colectiva de un
periódico
revista

no lleva la
aborda temas firma de la
de actualidad Editorial persona que
escribe

tiene
trascendencia
pública

5Cáceres Ramírez, Orlando (2016). Ejemplos de ensayos cortos. Tomado de: https://www.aboutespanol.com/ejemplos-de-
ensayos-cortos-2879489

32
Por ejemplo:

Tragedia cultural: inminente extinción de la lengua kiliwa

La extinción de la lengua kiliwa es casi inminente. El fin de ese idioma, el más amenazado de
los 20 en peligro de desaparecer en México, es una especie de tragedia lingüística que no sólo
afecta la diversidad cultural de Baja California, su lugar de origen, sino de México y el mundo.6

En este caso, la tesis del autor institucional (diario La Jornada) es que “la extinción de la
lengua kiliwa es casi inminente”.

• El artículo de opinión

El artículo de opinión es similar al editorial pues recoge opiniones sobre temas de actualidad,
solo que, en este caso, dichas opiniones no son institucionales sino personales: tienen una
firma de autoría. En ocasiones, esta opinión puede disentir manifiestamente de la postura
institucional del periódico expresada en sus editoriales.

recoge la
opinión de una
persona

Artículo lleva la firma


aborda temas
de la persona
de actualidad de opinión que escribe

tiene
trascendencia
pública

6 Publicado en el diario La Jornada. En: http://www.jornada.unam.mx/ultimas

33
Por ejemplo:

Seguridad vial
Por Estela Páez

Luego del accidente de tránsito sucedido hace apenas cuarenta y ocho horas deseo compartir
con los lectores mis más profundos sentimientos al respecto. Podría decirse que esta
catástrofe es prácticamente la crónica de una muerte anunciada. Nuestras calles se
encuentran atestadas de conductores que no conocen las reglas mínimas de tránsito e
inclusive muchos que no poseen la licencia correspondiente para hacer uso de un automóvil.
Es importante entender que un auto es una herramienta, pero también un arma, una muy
peligrosa. Por ello, es necesario que todos los conductores conozcan rigurosamente en su
totalidad las normas de tránsito y hayan superado los exámenes necesarios para hacer uso
de un vehículo. También es necesario concientizar a peatones y ciclistas la necesidad de
respetar de igual forma las reglas en la vía pública.7

b. El Texto descriptivo

Un texto descriptivo detalla las características de seres, objetos, escenas, momentos o


experiencias, a fin de acercar a los lectores de una manera vívida a la temática abordada. Utiliza
adjetivos fuertes para ilustrar los elementos que describen y detalla los sentimientos que estos
provocan. Así mismo, emplea -con medida- figuras literarias como metáforas o símiles.

detalla
características
de seres,
objetos,
escenas

pretende
acercar a los utiliza
lectores a la adjetivos para
Texto
temática ilustrar los
abordada, de descriptivo elementos
una manera descritos
vívida

suele emplear
figuras
literarias

7 Tomado de: http://ejemplosde.org/lengua-y-literatura/articulos-de-opinion/#ixzz4wd5G5hzS

34
Los tipos de textos descriptivos pertenecen en su mayoría a géneros literarios menores, entre
otros, los siguientes:

• Ecfrasis

Descripción de una escena representada en un objeto artístico como un friso, un bajo-relieve


o un cuadro. Puede ser real o ficticia y a menudo está insertada dentro de una narración
literaria. Por ejemplo:

El violinista azul, de Marc Chagall

[…] Chagall ha detenido su largo vuelo sobre mis libros, viene de sobrevolar los campos y las
aldeas, ha estremecido los árboles, ha derribado los frutos, la manzana que descalabró los
ojos miopes de Sir Isaac Newton.

Le digo que no crea que yo también entreveo la posibilidad de volar, de caminar por el
cielorraso de invitar a las muchachas a mirar la ciudad desde arriba. Chagall sonríe y sabe
que un hombre cauto no puede huir de la cordura. Si me atrevo y abro la ventana sé lo que
puede suceder: un hombre que se va sobre el aire inventando con un violín rojo una serenata.8

• Prosografía

Consiste en la descripción de rasgos físicos o externos de una persona, y suele emplearse


como un recurso de las novelas literarias. Miguel de Cervantes, en “El Quijote”, realiza la
siguiente prosografía:

Su nombre es Dulcinea; [...] su hermosura, sobrehumana, [...] sus cabellos son oro, su frente
campos elíseos, sus cejas arcos del cielo, sus ojos soles, sus mejillas rosas, sus labios corales,
perlas sus dientes, alabastro su cuello, mármol su pecho, marfil sus manos, su blancura
nieve... 9

8 Descripción de José Guatanabe. Tomada de: http://poesia-pintura.blogspot.com/2011/02/chagall-de-jose-watanabe.html


9 Ejemplos de prosografía. En: http://www.retoricas.com/2009/06/6-ejemplos-de-prosopografia.html

35
• Etopeya

Consiste en la descripción de los rasgos internos de una persona, sean éstos morales, de
personalidad, vicios o virtudes. Juan Valera, en “Pepita Jiménez”, realiza la siguiente
etopeya:

Don Gumersindo [...] era afable [...] servicial. Compasivo [...] y se desvivía por complacer y ser
útil a todo el mundo aunque costase trabajos, desvelos, fatiga, con tal que no le costase un
real [...] Nunca había tenido inclinación alguna amorosa a una mujer determinada [...] pero
inocentemente, sin malicia, gustaba de todas, y era el viejo más amigo de requebrar a las
muchachas.10

• Retrato

Consiste en la descripción de los rasgos internos y externos de una persona; es decir, es la


combinación de prosopografía y etopeya. Pedro Antonio de Alarcón, en “El sombrero de
tres picos”, realiza el siguiente retrato:

El tío Lucas era más feo que Picio. Lo había sido toda su vida, y ya tenía cerca de cuarenta
años. Sin embargo, pocos hombres tan simpáticos y agradables habrá echado Dios al mundo.
Luego venía la voz, vibrante, elástica, atractiva. Llegaba después lo que aquella voz decía:
todo oportuno, discreto, ingenioso, persuasivo.11

• Paisaje

Consiste en la descripción de un panorama o entorno ambiental, sea rural o urbano. Julio


Ramón Ribeyro, en “Los gallinazos sin plumas”, describe el siguiente paisaje:

A las seis de la mañana la ciudad se levanta de puntillas y comienza a dar sus primeros pasos.
Una fina niebla disuelve el perfil de los objetos y crea una atmósfera encantada. Las personas
que recorren la ciudad a esta hora parecen que están hechas de otra sustancia, que
pertenecen a un orden de vida fantasmal. Las beatas se arrastran penosamente hasta
desaparecer en los pórticos de las iglesias. Los noctámbulos, macerados por la noche,
regresan a sus casas envueltos en sus bufandas y en su melancolía. Los basureros inician por
la avenida Pardo su paseo siniestro, armados de escobas y de carretas.12

c. El texto expositivo o informativo

A diferencia de los textos descriptivos, los expositivos no dan detalles sobre las características
de una persona o un ambiente, sino que proporcionan información, instrucciones o
explicaciones de ideas objetivas. Hay gran variedad de textos informativos, cada uno con su
propia estructura. Veamos.

10 Ejemplos de etopeya. En: http://www.retoricas.com/2009/06/2-ejemplos-de-etopeya.html


11 Ejemplos de retrato. En: http://www.retoricas.com/2009/06/2-ejemplos-de-retrato.html
12 Ramón Ribeyro, Julio (2012). Los gallinazos sin plumas, La palabra del mudo, tomo I. Lima: Planeta Perú, página 53. Citado en:

Ministerio de Educación (2016). Lengua y Literatura. Octavo Grado. Texto para el estudiante. Quito: UASB, página 166.

36
• Carta formal

Es un texto que escribe una persona para comunicarse con otra a la que no conoce, sea para
tratar un tema concreto, o para abordar cuestiones laborales, académicas o institucionales.
La formalidad de la carta se evidencia en frases llamadas fórmulas de cortesía. 13

A continuación, explicamos cada una de las partes de una carta formal con un ejemplo
concreto:

Partes de una carta


Definición Ejemplo
formal

Lugar y fecha de emisión de


Lugar y fecha Latacunga, 20 de octubre de 2017
la carta

Nombre de la persona y,
Señora María Elena Villacís
debajo de ella, su cargo. Se
Encabezado Directora del Colegio Pascal
suele incluir también la
Presente.-
palabra “Presente”

Saludo Saludo cortés a la persona Estimada Directora:

Me dirijo a usted en mi calidad de Directora del Museo


Nacional para comunicarle que, del 3 al 26 de diciembre de
Primer párrafo de la carta,
2017, en coordinación con la Embajada de México, abriremos
Introducción con los antecedentes o
una exposición sobre “Culturas mesoamericanas”, compuesta
información previa.
por más de 150 reliquias originales de los antepasados Mayas
y Aztecas.

En este sentido, me permito solicitar su autorización para que


los docentes y alumnos de tercer año de Bachillerato, puedan
asistir a esta importante muestra y se beneficien de los
Desarrollo del mensaje
recorridos especializados que estaremos ofreciendo
Cuerpo central que se quiere
gratuitamente todos los días jueves y viernes, de 09h00 a
transmitir.
14h00. En caso de interesarle esta propuesta, solicito a usted
gestionar la reserva correspondiente llamando al teléfono
0999945567.

Esperando contar con la presencia de los docentes y


Cierre del texto con frases
Despedida estudiantes de su valiosa institución, me despido
amigables
atentamente.

Nombre y rúbrica de la Juana Uquillas


Firma
persona que emite la carta Directora del Museo Nacional

13 ¿Qué es una carta formal?. En: https://www.modelocarta.net/que-es-una-carta-formal.html

37
• Noticia periodística

La noticia periodística es un texto que informa un acontecimiento actual y que generalmente


es escrito y transmitido a través de algún medio de comunicación como periódicos, revistas
o sitios web. En una noticia, los hechos transmitidos deben ser verificables, objetivos (sin
opiniones personales del periodista), claros y breves. Debe responder a cuatro preguntas:
qué ocurrió, dónde ocurrió, a quién afectó o influyó lo que ocurrió y por qué ocurrió. 14 A
continuación, explicamos cada una de las partes de una noticia periodística con un ejemplo
concreto:

Partes de una
Definición Ejemplo
noticia periodística

Texto breve que se coloca


Epígrafe o antetítulo
encima del título y adelanta
alguna información que se Hallazgo histórico:
complementará con el título.

Es lo primero que capta la


Título
atención del lector, allí se debe Descubren ciudad más antigua
colocar la información
fundamental de la noticia
de Europa
Párrafo corto ubicado debajo del
título, que resume a grandes Los restos tendrían 2 mil años más que las pirámides de
Copete o entradilla
rasgos el acontecimiento Egipto
principal.

Un grupo de arqueólogos halló los restos de la que consideran


la ciudad más antigua de Europa, de unos 7 mil años de
antigüedad, y a los que creen son los vestigios de los primeros
mini-Estados de ese continente. Los restos tendrían 2 mil años
más que las pirámides de Egipto.
Desarrollo de toda la noticia
completa. Generalmente adopta
Luego de más de tres años de investigaciones, expertos
Cuerpo de la noticia una forma de pirámide invertida,
colocando la información de alemanes desenterraron partes de más de 150 templos
mayor a menor importancia. construidos con tierra y madera, cuya fecha data entre los años
4800 y 4600 AC, los que están esparcidos en una zona que se
expande entre Alemania, Austria y Eslovaquia. Los edificios
habrían sido construidos unos 2 mil años antes de las
Pirámides de Egipto y del monumento de Stonehenge en Gran
Bretaña, según dijo el diario inglés “The Independent”.

La foto no es fundamental, pero


sí es un elemento que completa
Foto y epígrafe la noticia y le da atractivo visual.
Debajo de la foto se coloca el
epígrafe, describiendo lo que se
ve en ella y su fuente.
Templo de tierra. Foto: National Geographic

Lugar, persona o agencia de


Fuente de la noticia Fuente: National Geographic
donde se han extraído los datos

14 Características y estructura de la noticia periodística. En: http://caracteristicas.org/caracteristicas-noticia-periodistica/

38
• Texto de divulgación científica o reportaje científico

En realidad, un texto de divulgación científica o reportaje científico es divulgativo y


argumentativo. Es una producción escrita que aborda temas sustentados en conocimientos
científicos. Entre otras características, los textos científicos: son claros, es decir, no generan
dudas ni cuestionamientos; son precisos, pues se ajustan fielmente a datos, cálculos,
medidas o expresiones; son verificables, esto es, susceptibles de comprobación posterior;
utilizan un lenguaje formal con terminología específica; no tienen una intención estética,
sino que, más bien, buscan plasmar conocimientos claros; y, finalmente, son objetivos, pues
se sustentan en hechos y en procedimientos lógicos. No suelen ser accesibles para públicos
no especializados.15

aborda temas
sustentados en
conocimientos
científicos

son objetivos Texto de son claros,


pues se
sustentan en
divulgación precisos,
científica verificables
hechos lógicos

utiliza lenguaje
formal con
terminología
científica

Un texto de divulgación científica está compuesto por una introducción, un nudo o cuerpo
del texto y un desenlace o fin. La introducción anuncia un resultado, un descubrimiento o un
invento científico; se trata de una tesis o postura del autor de carácter verificable. El nudo o
cuerpo del texto es la parte fundamental del escrito pues expone las ideas centrales del
resultado, descubrimiento o invento, con las respectivas argumentaciones. Y el desenlace es
la conclusión: un cierre del texto.

15Cáceres, Orlando (2016). Textos de divulgación científica. En: https://www.aboutespanol.com/textos-de-divulgacion-cientifica-


2879469

39
Veamos un ejemplo:

Científicos de la Universidad de California curaron las cataratas congénitas de doce bebés


usando células madre, luego de tener éxito en experimentos con conejos y monos. La cirugía
tradicional requiere de 6 milímetros para reemplazar el cristalino anómalo por uno artificial,
pero implica destruir células madre que protegen al ojo. El nuevo método permite una
abertura de solo 1,5 milímetros para extraer el tejido afectado y provocar que las células
madre del cristalino regeneren uno nuevo sin cataratas.

En las pruebas iniciales, el procedimiento evidenció una tasa mucho menor de complicaciones
frente a la técnica tradicional. El primer bebé operado hace dos años todavía tiene una buena
visión y se espera que en la niñez no se presente inconvenientes. Sin embargo, la cirugía no
funciona para adultos de edad avanzada porque sus células se regeneran lentamente. Aun si
la nueva técnica solo funcionara para niños, sigue significando un avance muy importante.

El texto citado hace referencia a la cirugía de cataratas congénitas con células madre aplicada
con éxito en doce bebés; ello se anuncia a manera de introducción. El nudo o cuerpo del
texto explica y argumenta en qué consiste esta cirugía, evidenciando cómo, este
procedimiento, tiene menores complicaciones respecto de las técnicas tradicionales. Se
concluye indicando que esta cirugía no funciona en adultos de edad avanzada y que, sin
embargo, sigue significando un avance importante.

d. El Texto narrativo

El texto narrativo es aquel que relata acontecimientos que se desarrollan en un lugar a lo largo
de un determinado espacio temporal. La narración está compuesta por una sucesión de hechos
que pueden ser reales o imaginarios y en los cuales intervienen diversos personajes.

Todo texto narrativo contiene las siguientes partes:

✓ Título o nombre del texto narrativo (sea cuento, novela o crónica, reportaje).

✓ Inicio o introducción, en donde se presentan personajes y acciones “normales” de la


narración en un ambiente específico. Los personajes pueden ser reales o ficticios. Las
acciones son los actos o funciones que realizan los personajes. Y el ambiente consiste en el
entorno físico, psicológico o social donde los personajes desarrollan sus acciones.

✓ Nudo o cuerpo del texto, que inicia cuando aparece un elemento de tensión que rompe
con aquella “normalidad” presentada en la introducción, y que se centra en desarrollar una
trama principal.

✓ Desenlace o fin, es decir, aquellos hechos que reordenan los elementos de tensión para
establecer una nueva “normalidad”. El desenlace deja planteado cómo serán las cosas a
partir de ese momento.16

16 Tomado de: https://definicion.de/texto-narrativo/

40
relata hechos
que se
desarrollan en
un lugar
determinado

los hechos se
los personajes
narran en una
de la narración
Texto sucesión
pueden ser
reales o narrativo temporal, con
un principio y
ficticios
un fin

los hechos de
la narración
pueden ser
reales o
ficticios

Partes de un texto narrativo

Título
nombre del texto

Desenlace o fin Inicio o introducción


hechos que reordenan los elementos de presentación de las personas (reales o
tensión para volver a un nuevp estado ficticios) y sus acciones "normales", y de su
normal ambiente físico, psicológico o social

Nudo o cuerpo del texto


inicia con un elemento de tensión que
rompe los aspectos "normales", y se centra
en desarrollar la trama central

41
Los textos narrativos pertenecen, en su mayoría, al género literario:

• Fábula

La fábula es una pequeña narración que siempre nos deja una enseñanza o moraleja. Sus
personajes pueden ser animales, seres humanos o seres fantásticos. Por ejemplo:

El lobo con piel de oveja

Pensó un día un lobo cambiar su apariencia para así facilitar la obtención de su comida. Se
metió entonces en una piel de oveja y se fue a pastar con el rebaño, despistando totalmente
al pastor.

Al atardecer, para su protección, fue llevado junto con todo el rebaño a un encierro, quedando
la puerta asegurada. Pero en la noche, buscando el pastor su provisión de carne para el día
siguiente, tomó al lobo creyendo que era un cordero y lo sacrificó al instante.

Moraleja: Según hagamos el engaño, así recibiremos el daño.17

El lobo con piel de oveja. Fábula de Esopo para niños. Imagen tomada de:
https://www.guiainfantil.com/articulos/ocio/cuentos-infantiles/el-lobo-con-piel-de-oveja-fabula-de-esopo-para-ninos/

• Anécdota

La anécdota es una narración de una situación real que le ocurre a una persona y que suele
ser chistosa o sorprendente. Por ejemplo:

Una tarde en Escocia nos dirigimos a las Fairy Pools, en la isla de Skye, un lugar idílico, para
darnos un chapuzón. Para acceder a ellas hay que caminar un ratillo por un sendero. Dejamos
la furgo en el aparcamiento y comenzamos a caminar. Serían las 5 de la tarde y todo el mundo
volvía de regreso. Según caminábamos íbamos disfrutando del río, las pozas y el paisaje.
Además, prácticamente sin gente. ¡Genial!

Llegamos a una poza preciosa. Nos pusimos el bañador y al agua. No fue demasiado fácil
porque el agua estaba congelada. Pero lo peor estaba por llegar. Comprendimos, en cosa de
segundos, por qué, a esas horas, no había un alma allí. Nada más salir del agua, ¡comenzaron

17Fábulas de Esopo para niños. En: https://www.guiainfantil.com/articulos/ocio/cuentos-infantiles/el-lobo-con-piel-de-oveja-


fabula-de-esopo-para-ninos/

42
a aparecer miles y miles de mosquitos highlanders dispuestos a chuparnos toda la sangre!
¡Los temidos mosquitos de las Highlands! Como si de una plaga de langostas se tratase nos
cubrieron enteritos, picándonos por todas las partes del cuerpo. Las niñas no podían vestirse,
acurrucadas gritaban “¡Mamá, no puedo!” incapaces de moverse, tan solo para intentar
quitarse los mosquitos. Pasamos de toalla, sin vergüenza ninguna, para vestirnos cuanto
antes y salir de allí pitando. Sin calcetines, sin sujetador… corrimos, literalmente, a la furgo,
dándonos tortazos para intentar quitarnos de encima estos minúsculos pero jodidos bichines.

Resultado: cientos de picaduras en todo el cuerpo, sobre todo Claudia que es un imán para
los mosquitos.

Los mosquitos de las Highlands no son una leyenda urbana. Son tan reales como la vida
misma.18

Highlands. Imagen tomada de:


https://www.escocia.es/highlands/

• Mito

El mito es una historia de la tradición oral cuyo propósito es entender los orígenes de un
pueblo o las acciones de los seres de la naturaleza o de ciertas divinidades. Por ejemplo:

Nunkui y la yuca

Los shuar habían consumido todos los recursos de las llanuras que habitaban. Un día Nunkui,
la madre tierra, le ofreció al pueblo a su hija como regalo. Les advirtió que, si cuidaban de
ella, les proveería de alimentos de toda clase, pero que si la llegaban a maltratarla, sufrirían
de nuevo de hambre.

Los shuar aceptaron y pudieron encontrar una cantidad de alimentos a su disposición. Un día
los niños de la comunidad maltrataron a la niña y, en castigo, la tierra se tragó a los
alimentos. Es por esto que hoy, elementos como la yuca tienen que ser buscados por debajo
de la tierra.19

18 Beats of my trips. “Viajando en Furgo”. Ebullición viajera: 25 anécdotas viajeras contadas por 25 blogueros de viajes. Tomado
de: http://www.beatsofmytrips.com/anecdotas-viajeras-blog-viajes/#Libre
19 Cajal, Alberto. Las 10 leyendas y mitos ecuatorianos más conocidas. Tomado de: https://www.lifeder.com/leyendas-

ecuatorianas/

43
Nunkui, creadora de las plantas. Imagen tomada de:
Ministerio de Educación. Lengua y Literatura – Noveno Grado. Quito: Universidad Andina Simón Bolívar, página 27.

• Cuento

El cuento es una breve narración de hechos imaginarios con un reducido número de


personajes, un argumento no demasiado complejo y un desenlace final rápido.

El espejo del cofre

En uno de sus múltiples viajes, un mercader compró a un buhonero un pequeño espejo, un


objeto que sus ojos jamás habían contemplado y le pareció algo sumamente extraordinario.
A pesar de no conocer cómo debía utilizarse, se lo llevó muy contento para mostrárselo a su
mujer.

Durante las largas jornadas del viaje de regreso a su hogar, descubrió en ese extraño objeto,
la familiar figura de su difunto padre. Asustado por esta extraña presencia, decidió no
contarle nada a su esposa y guardar el preciado retrato de su padre en uno de los baúles del
desván.

Todos los días, desde que regresó de su viaje, subía al desván para contemplar a su padre.
Cuando bajaba, siempre se mostraba entristecido y esquivo ante las preguntas de su mujer.

Harta de esta situación, subió al desván para descubrir el motivo de la tristeza de su marido.
Tras rebuscar en las pertenencias de su esposo, encontró el retrato de una hermosa mujer.
Muy enfadada ante el engaño del mercader, le echó en cara que la estaba engañando con
otra mujer. Una acusación a la que su marido respondía con que la persona del baúl era su
padre.

Tales eran los gritos que daban, que un monje se acercó hasta su hogar para medrar en la
disputa. El matrimonio le contó el motivo de su discusión y cuando el monje subió al desván,
lo único que encontró fue la efigie de un anciano monje zen.

44
El espejo del cofre. Cuento chino. Imagen tomada de:
http://www.desafiomaterno.com/el-espejo-de-matsuyama/

• Novela

La novela es una obra literaria desarrollada en prosa, que se encarga de narrar acciones que
pertenecen parcial o totalmente al ámbito de la ficción. Generalmente las novelas son de
gran extensión (más de 60.000 palabras) y, salvo algunas excepciones, presentan tramas de
gran complejidad, una cantidad de personajes elevada y, en algunos casos, varios narradores.
“Cien años de Soledad” de Gabriel García Márquez, “La casa de los espíritus” de Isabel
Allende y miles de ejemplos más, se incluyen dentro de este género narrativo.

Portada del libro Cien años de soledad, editado por Editorial Sudamericana. Imagen tomada de:
http://portadashojasylibros.blogspot.com/2014/06/cien-anos-de-soledad-gabriel-garcia.html

45
• Leyenda

La leyenda es una narración escrita u oral que, al sustentarse en hechos reales, se exhibe
como verdadera pero que incluye componentes propios de la imaginación y de la ficción. Las
leyendas se transmiten de generación en generación, lo que posibilita que se vayan
modificando de acuerdo al contexto social y a la época. Siempre se halla ligada a un
personaje, un espacio geográfico o un objeto preciso, y muestra cómo dicho elemento
concreto se integra a la historia o la vida cotidiana de la comunidad. Por ejemplo:

La quebrada fue durante muchos siglos la madriguera de los duendes. En ella se confundieron
los pálidos espíritus cristianos con las almas rojas de los incas, descendientes legítimos del
sol. Diablos y vírgenes, ascetas y emperadores, hicieron su palacio de jaspe en el abismo sin
fondo de la quebrada. La quebrada, durante la colonia y la vida libre, fue alcoba de corazones
transparentes, que refinaron las caricias prohibidas. Pero cuando nació el siglo XX, se
introdujo en El Tejar, como un apache, la luz eléctrica y asesinó a todos los fantasmas.20

Convento de El Tejar con la Quebrada de El Tejar, 1870-1875. Colección Alphons Stübel.


Foto registrada por el Instituto Nacional de Patrimonio Cultural. En:
http://www.fotografianacional.gob.ec/web/es/galeria/element/5523

Dentro del género periodístico se considera a la crónica como un tipo de texto narrativo.

• Crónica periodística

La crónica es un relato que narra sucesos reales (no fantasiosos) siguiendo, siempre, un orden
cronológico. La persona que se encarga de recopilar y narrar los hechos se conoce como
cronista. Los cronistas de Indias, por ejemplo, eran los autores que, tras la llegada europea
a América, se dedicaron a describir las costumbres de los aborígenes que habitaban en el
suelo americano. Actualmente, las crónicas periodísticas se publican en diarios o revistas, o
se las transmite por radio o televisión. Por ejemplo:

Los músicos llegaron temprano y todos los técnicos ya tenían preparados los equipos en el
escenario para brindar al público un espectáculo inolvidable. Solo restaba una cosa: que la
banda saliera a tocar. Luego de una media hora, a las diez de la noche, el concierto empezó

20 Salvador, Humberto (1993). En la ciudad he perdido una novela. Quito: Libresa, página 94.

46
para la alegría y la euforia de los miles de fanáticos presentes en aquel estadio. El sacrificio
que habían pasado no fue en vano. El frío no fue obstáculo para que una cantidad aproximada
de veinte mil personas hiciera colas de varias cuadras para poder ingresar y ver actuar a sus
estrellas. La banda brindó un colorido espectáculo iniciando con las canciones que le hicieron
famosa en sus comienzos. También interpretó algunos de los temas de su último disco, el cual
había sido lanzado en noviembre pasado en nuestro país.21

e. El texto poético

El texto poético es aquel que apela a recursos estilísticos para transmitir emociones y
sentimientos, respetando los criterios de estilo del autor. Comúnmente está escrito en verso y
recibe el nombre de poema o poesía. Existen, sin embargo, textos poéticos desarrollados en
prosa. Los versos, las estrofas y el ritmo componen la métrica del texto poético, donde los poetas
imprimen el sello de sus recursos literarios. 22

utiliza recursos
estilísticos o
figuras
literarias

cuando está
transmite
escrito en
prosa, se llama
Texto poético emociones y
sentimientos
"prosa"

Cuando está
escrito en
verso, se llama
"poema" o
"poesía"

Ejemplo de texto poético escrito en verso:

21 Crónica periodística. Tomado de: https://www.aboutespanol.com/cronica-periodistica-2879694


22 Definición de texto poético. Tomado de: https://definicion.de/texto-poetico/

47
Cerrar podrá mis ojos la postrera
Sombra que me llevare el blanco día,
Y podrá desatar esta alma mía
Hora a su afán ansioso lisonjera;
Más no, de esa otra parte, en la ribera,
Dejará la memoria, en donde ardía:
Nadar sabe mi llama el agua fría,
Y perder el respeto a ley severa.
Alma a quien todo un dios prisión ha sido,
Venas que humor a tanto fuego han dado,
Medulas que han gloriosamente ardido:
Su cuerpo dejará no su cuidado;
Serán ceniza, mas tendrá sentido;
Polvo serán, mas polvo enamorado23

Ejemplo de texto poético escrito en prosa:

Ya no recuerdo nada de ti. Ni tu mirada de ojos brillantes que hipnotizaba mis sentidos a la luz
de las velas. Ni tus besos en la frente que olían a café por las mañanas y a chocolate intenso justo
antes de la siesta. Ni tus sonrisas en la oscuridad que recorrían mis mejillas después de un suave
mordisco en la oreja.

No, no me acuerdo de las cosquillas que me hacía tu respiración en el cuello a la hora de dormir,
ni tu obsesión de abrazarme con todo el cuerpo para que no pudiera marcharme, ni siquiera los
te quiero que me susurrabas al oído para que soñase contigo toda la noche.24

f. El texto dramático

El texto dramático es aquel que representa algún episodio o conflicto de la vida de los seres
humanos por medio del diálogo de los personajes. Este texto forma parte de los géneros
literarios y suele ser creada para ser representada o interpretada por actores frente a un público,
pudiendo estar escrita en prosa o en verso o combinando a ambos.

Una obra dramática se estructura del siguiente modo: a) presentación del conflicto, es decir, de
las posturas contrarias que permiten avanzar en el argumento de la obra; b) desarrollo de la
acción dramática, es decir, del duelo o pugna entre los personajes; y c) desenlace de la acción
dramática o resolución del conflicto.

23 Amor constante más allá de la muerte de Francisco de Quevedo. Tomado de: http://10ejemplos.com/ejemplos-de-poesia.
24 Extracto de “No te recuerdo” del autor “Superando lo imposible”. Tomado de: http://tustextos.com/tags/prosa-poetica/

48
representa
episodios
conflictivos de
la vida de las
personas

el desenlace emplea el
Texto
siempre es diálogo como
dramático dramático recurso central

suele crearse
para que sea
interpretada o
representada
por actores

Hay varios tipos de textos dramáticos:

• Comedia

Es un texto que desarrolla conflictos divertidos y amables, con personajes pertenecientes al


mundo de la normalidad. Ejemplo:

Un futuro músico, de Fabián Sevilla25

(La escena transcurre en una tienda de música: hay instrumentos expuestos y afiches de
músicos famosos)

Vendedor: (Al ver que Lucas mira los instrumentos.) Buenas tardes, nene, ¿qué buscas?

Lucas: Quiero dedicarme a la música. Y mi papá me dio dinero para comprarme un


instrumento, pero no sé cuál elegir.

Vendedor: (Enérgico.) Viniste al sitio indicado: “¡SONAMOS!” es la mejor tienda de


instrumentos musicales de la ciudad. ¿Qué te parece este piano?

Lucas: Lindo, pero quisiera uno que, en vez de teclas blancas, las tenga azules, y que las
negritas, fueran rojas.

Vendedor: Pero todos los pianos tienen teclas blancas y negras.

25 Tomado de: http://www.todo-teatro.com/7-comedias-breves-de-fabian-sevilla/

49
Lucas: Eso porque nadie les ha enseñado a maquillarse aún.

Vendedor: Ese modelo de ahí tiene los pedales…

Lucas: ¿Aquí vende instrumentos o triciclos?

Vendedor: Los pianistas pisan los pedales para…

Lucas: Entonces es ideal para un corredor de Fórmula 1. No me convence.

Vendedor: ¿Y el órgano? Tengo todo tipo de órganos…

Lucas: Si no los tuviera estaría muerto.

Vendedor: Órganos eléctricos.

Lucas: No sabía que había corazones o pulmones que se enchufaran.

Vendedor: (Trata mantener la paciencia.) Mejor pasemos a otra cosa. Alguno de viento…

Lucas: Ventilador ya tengo.

Vendedor: Yo me refería a los que se deben soplar.

Lucas: ¿Una torta de cumpleaños?

Vendedor: No, algo como… como… como esa flauta.

Lucas: (Mira el instrumento.) ¡Esta fallada! No voy a comprar algo que tiene cualquier
cantidad de agujeritos. ¡Parecen los soquetes de mi abuelito!

Vendedor: Sirven para tocar la flauta dulce.

Lucas: Si es dulce, entonces no. A mí me encantan las comidas saladas. Además, imagínese si
olvido guardarla y se la comen las hormigas que son dulceras a rabiar.

Vendedor: ¿No te gustaría tocar la armónica?

Lucas: ¡Ni loco! Si llego a tocar a Mónica, mi compañera de banco, me muele a patadas.

Vendedor: Podría ser alguno de cuerda.

Lucas: Si quisiera algo a lo que hay que darle cuerda, me compraría un reloj despertador.

Vendedor: Me refiero a instrumentos que tienen las cuerdas tensadas y…

Lucas: ¡Menos! Lo único que me falta: terminar tendiendo la ropa al sol en esas cuerdas.

Vendedor: Por ejemplo el violín, la viola o el violonchelo.

50
Lucas: No quiero saber nada con el Chelo: en el último partido me metió un patadón que
terminó haciendo un gol con mi cabeza.

Vendedor: Tal vez una guitarra criolla.

Lucas: Las únicas criollas que me gustan son las empanadas que cocina mi abuela.

Vendedor: Un bajo o un contrabajo.

Lucas: ¿Insinúa que soy petiso?

Vendedor: ¿Y la batería?

Lucas: ¡Eso! Una con platillos que suenen como fuentes, tambores que se oigan como
cacerolas y un bombo que retumbe como cuando se golpea una olla.

Vendedor: Entonces te conviene comprar una batería de cocina.

Lucas: Y que pueda golpear durante horas.

Vendedor: Deberá ser una batería larga duración y recargable. Serías un gran batero…

Lucas: ¿A dónde va el tero?

Vendedor: Batero es un modo de decir baterista.

Lucas: No, yo no quiero ser carterista.

Vendedor: Es el que interpreta la batería. Te puedo ofrecer una que tiene unas hermosas
baquetas…

Lucas: ¡Otra que se va! ¿A dónde va Queta?

Vendedor: Las baquetas se usan para golpear la batería.

Lucas: Yo no quiero golpearla. A ver si la rompo, con lo cara que debe salir.

Vendedor: También se le dicen escobillas.

Lucas: Pero deseo ser músico, no barrendero.

Vendedor: Aunque no tienes la menor idea, creo que con práctica y estudio, podrás tocar
jazz.

Lucas: ¿Yas mismo?

Vendedor: Hacer pop.

Lucas: ¿Como una burbuja que explota?

51
Vendedor: Y rockear.

Lucas: Yo no soy de roncar.

Vendedor: (Al público.) Este cree que para tocar en clave de Sol hay que usar anteojos
oscuros.

Lucas: Si lo pienso, me gustaría comenzar tocando una bocina de bicicleta o de un camión.

Vendedor: (Al público.) Con lo poco que sabe de música, le convendría una bocina de avión

Lucas: ¿Qué dijo?

Vendedor: Creo que estás demasiado crudito. Por ahora te recomiendo que comiences
practicando con esto (Le pasa un timbre de puerta).

Lucas: ¡Buenísimo! Con esto, si me aburro de practicar puedo dedicarme a otra cosa.

Vendedor: ¿A qué?

Lucas: ¡Al ring raje! (Presiona el timbre y corre mientras se produce el sonido).

• Tragicomedia

Es una obra dramática en la que se mezclan los elementos trágicos y cómicos. También suele
emplear elementos sarcásticos. También se la conoce como pieza. El ejemplo más
característico es "La Celestina", de Fernando de Rojas.

Portada del libro “La Celestina” de Fernando de Rojas, editado por Biblioteca Sopena. Imagen tomada de:
https://www.todocoleccion.net/s/catalogo-antiguedades-arte-coleccionismo-subastas-I=c-0-Ranking_Hoy-1-2-DESC-gg-2146736329

52
• Tragedia

La tragedia presenta conflictos entre personajes -generalmente reyes o héroes- que son
víctimas de terribles pasiones que les llevan a la destrucción y a la muerte. Ejemplos de
tragedias son: Ayax de Sófocles, Los persas de Esquilo, y Hamlet o Romeo y Julieta de William
Shakespeare.

Portada del libro Romero y Julieta de William Shakespeare editado por Zig-Zag. Imagen tomada de:
https://www.antartica.cl/antartica/servlet/LibroServlet?action=fichaLibro&id_libro=196884

• Autosacramental

Son representaciones de episodios bíblicos, misterios de la religión o conflictos de carácter


moral y teológico. Inicialmente representados en los templos o pórticos de las iglesias. El
siguiente pasaje de un autosacramental de Juan de Timoneda titulado Los Desposorios de
Cristo es un buen ejemplo:

Humanidad:
Abre aqueste cofre triste,
¡Oh María!
Saca las donas que envía
la madrina y el padrino;
Saca el collar de oro fino
sembrado de pedrería;
Saca esa argentería
de tu amado.

Nuestra Señora:
(Saca del cofre unas monedas)
¡Ay, dineros ha enviado!...
¡Qué moneda tan rabiosa!
Son las arras que a su esposa
ha de dar el desposado.

53
Humanidad:
Por eso será comprado
tu placer;
son por los que ha de vender
tu hijo a los carniceros;
ellos son treinta dineros,
los cuales puedes bien ver.

Nuestra Señora:
(Sacando del mismo cofre una soga).
Esta soga al parecer
mucho espanta;
¡Ay, que el alma me quebranta,
que nunca tal dona he visto!

Humanidad:
Pues con ella a Jesucristo
desollarán la garganta.

Nuestra Señora:
¿Hay dolor y pena tanta?
¡Ay, mi amado!
¡Qué collar de oro tirado
Adán, vuestro padre, os da!
¡Collar que él os rasgará,
mi Dios y mi Hijo amado!

Humanidad:
Son azotes de amargura,
con que le den los sayones.

Nuestra Señora:
¡Quién supiese qué varones
le han de dar!
¡Quién los pudiera hablar
para decilles: «Señores,
dadme a mí esos dolores
que a mi Hijo habéis de dar!»
¡Oh celestial Rey sin par
y sin pecado!

Lázaro:
(Sacando una cruz.)
Cata la cruz.

Nuestra Señora:
¡Oh, qué afán!
¿Para qué es?

54
Lázaro:
Para do claven los pies.

Nuestra Señora:
¿Y qué pies?

Lázaro:
Los de Jesús26

• Entremés

Los entremeses son piezas teatrales cómicas surgidas en España en el siglo XV. Eran
representados en los intermedios de las jornadas de una obra. Los más famosos son de
autoría de Miguel de Cervantes como El juez de los divorcios, El rufián viudo llamado
Trampagos, La cueva de Salamanca y El viejo celoso.

Escena del entremés “El juez de los divorcios” de Miguel de Cervantes, interpretado por Máwüari Teatro y Producciones. Imagen tomada de:
http://mawuariteatro.blogspot.com/2017/04/el-juez-de-los-divorcios-de-miguel-de.html

• Zarzuela

Obra dramática y musical, en la que alternativamente se declama y se canta. Como género


específicamente español, tiene sus orígenes remotos en la musicalización de distintos
misterios y dramas. El creador fue Calderón de la Barca con su pieza El Jardín de Farelina.

• Sainete

Pieza jocosa de corta duración -inferior a un acto- de carácter y argumento popular, en la


que se ridiculizan los vicios y convenciones sociales.

• Ópera

Representación teatral en la cual sus personajes cantan. En ella la acción dramática se


conjuga con la intervención de la orquesta, la danza, la palabra y otros elementos.

26 Tomado de: http://thecult.es/Cronicas/los-autos-sacramentales-de-calderon-de-la-barca.html

55
Referencias bibliográficas

• Beats of my trips. “Viajando en Furgo”. Ebullición viajera: 25 anécdotas viajeras contadas por 25 blogueros de
viajes. Tomado de: http://www.beatsofmytrips.com/anecdotas-viajeras-blog-viajes/#Libre

• Cáceres Ramírez, Orlando (2016). ¿Qué es un ensayo? Tomado de: https://www.aboutespanol.com/que-es-un-


ensayo-2879495

• Cáceres Ramírez, Orlando (2016). Ejemplos de ensayos cortos. Tomado de:


https://www.aboutespanol.com/ejemplos-de-ensayos-cortos-2879489

• Cáceres Ramírez, Orlando (2016). Textos de divulgación científica. En: https://www.aboutespanol.com/textos-


de-divulgacion-cientifica-2879469

• Cajal, Alberto. Las 10 leyendas y mitos ecuatorianos más conocidas. Tomado de:
https://www.lifeder.com/leyendas-ecuatorianas/

• Características y estructura de la noticia periodística. En: http://caracteristicas.org/caracteristicas-noticia-


periodistica/

• Crónica periodística. Tomado de: https://www.aboutespanol.com/cronica-periodistica-287969

• Ejemplos de etopeya. En: http://www.retoricas.com/2009/06/2-ejemplos-de-etopeya.html

• Ejemplos de prosografía. En: http://www.retoricas.com/2009/06/6-ejemplos-de-prosopografia.html

• Ejemplos de retrato. En: http://www.retoricas.com/2009/06/2-ejemplos-de-retrato.html

• Género dramático. En: http://www.profesorenlinea.cl/castellano/generodramatico.htm

• Ministerio de Educación (2016). Lengua y Literatura. Octavo Grado. Texto para el estudiante. Quito: UASB.

• Ministerio de Educación (2016). Lengua y Literatura. Noveno Grado. Texto para el estudiante. Quito: UASB.

• ¿Qué es una carta formal? En: https://www.modelocarta.net/que-es-una-carta-formal.html

Banco de preguntas y respuestas

Sobre los textos argumentativos

1)

Lea el siguiente texto e identifique a qué tipo pertenece:

La mujer ha venido luchando por la igualdad de oportunidades con el hombre, dentro del ámbito laboral, político
e incluso religioso [...].
No es necesaria la supremacía del poder en el hombre o en la mujer, sino que se debería
mantener la equidad de género. No es verdad que el hombre siempre sea más inteligente o fuerte que la mujer;
ya se ha comprobado que, en varios aspectos, la mujer es capaz de hacer las cosas de forma igual o mejor que su
compañero masculino. Estas necesitan más oportunidades y voluntad para romper estereotipos creados y
transmitidos para minimizarla. Un ejemplo de lo afirmado fue Manuela Cañizares, quien arengó a los próceres de
la Independencia cuando estos querían darse por vencidos [...].

56
En síntesis, lo más grave es que hay mujeres que son víctimas de acoso y violencia, y que a pesar de conocer sus
derechos, se someten a la prepotencia del hombre.


a) Argumentativo
b) Introductorio
c) De conclusión
d) Descriptivo

Respuesta correcta: a)

Luego de una lectura detenida del texto citado en esta pregunta, podemos afirmar que se trata de un texto
argumentativo. ¿Por qué? Es, al parecer, un texto de opinión o un reportaje. Su intención es demostrar una tesis
definida en la parte introductoria: que “la mujer ha venido luchando por la igualdad de oportunidades con el hombre,
dentro del ámbito laboral, político e incluso religioso”. Para demostrar esta tesis se plantean varios argumentos; uno
de ellos es que “mujer es capaz de hacer las cosas de forma igual o mejor que su compañero masculino” (argumento
racional). Otro argumento es que ciertas mujeres, como Manuela Cañizares, han roto estereotipos creados para
minimizarlas. Manuela arengó a los próceres de la Independencia cuando estos querían darse por vencidos
(argumento de ejemplificación). Se plantea, además, un texto de conclusión citado en el último párrafo.

No se trata de un texto introductorio o de conclusión solamente (estos son componentes del texto argumentativo).
Y tampoco es un texto descriptivo porque no se centra en describir a una persona, un animal, un paisaje, un objeto o
una situación a fin de que los receptores puedan representarlos en sus mentes.

2)

Identifique la tesis del siguiente texto:

Inculcar el gusto por la lectura en la actualidad es una tarea ardua, teniendo en cuenta los avances de la tecnología
que cada día provee de más y mejores recursos para este propósito, como los audiolibros que permiten escuchar
gran variedad de textos literarios y de contenido científico; sin embargo, restan la posibilidad de compartir
experiencias, sobre todo el contacto humano que se establece entre el lector-oyente, y desde luego, la apropiación
particular del contenido impreso que cambia de persona a persona, debido a la individualidad que nos caracteriza,
que nos hace únicos. Me refiero a la relación emisor-lector.

El bibliotecario adquiere singular importancia porque, al conocer a los usuarios de este servicio, está en posibilidad
de proponer y promover, con el auspicio de universidades, ONG, entidades estatales y particulares, el acercamiento
y contacto con escritores nacionales y extranjeros. De esta manera se asegura el nacimiento de lectores con
perspectivas sensibles en torno a la realidad, al tiempo que permite a estos un acercamiento profundo al
pensamiento, a las costumbres, a las tradiciones y hábitos del otro.

a) Los audiolibros son un recurso tecnológico para los lectores


b) Los bibliotecarios llevan el registro de todos los libros
c) La lectura en la biblioteca pública es productiva
d) El bibliotecario asume un rol fundamental frente a la lectura

Respuesta correcta: d)

El texto citado puede concebirse como un texto argumentativo; seguramente se trata de un artículo de opinión. Y
todo texto argumentativo está compuesto por una tesis, un argumento y una conclusión. La tesis es una afirmación
demostrable que refleja el punto de vista del autor o autora del texto. El argumento corresponde a la proposición o
proposiciones que sustentan o justifican la tesis. Y las conclusiones son ideas finales que dan cierre a lo expuesto,
reafirmando como verdadera la tesis planteada.

El texto que se cita en esta pregunta plantea dos tesis: a) los audiolibros, aun cuando permiten escuchar gran variedad
de libros, restan la posibilidad de compartir experiencias; y b) los bibliotecarios asumen un rol fundamental frente a
la lectura. Las dos tesis se complementan.

57
No obstante, en esta pregunta NO se menciona como opción que “los audiolibros, aun cuando permiten escuchar
gran variedad de libros, restan la posibilidad de compartir experiencias”. La opción correcta es, entonces: “El
bibliotecario asume un rol fundamental frente a la lectura”.

3)

Identifique la tesis del siguiente texto:

De todos los medios para adquirir cultura, la lectura es la principal, porque permite el contacto con el arte literario,
con otras manifestaciones artísticas, con el ambiente y el pensamiento ajeno. Actualmente existe a escala mundial
un decrecimiento del hábito de lectura, situación que afecta desfavorablemente el desarrollo del hombre.

La lectura constituye uno de los contenidos del proceso de enseñanza -aprendizaje de la Educación Primaria de
mayor trascendencia para la vida de un niño. No aprender a leer o leer con dificultad, tiene efectos negativos, no
sólo sobre el aprendizaje del resto de las demás materias escolares, sino también sobre el desarrollo integral, pues
le impide apropiarse del conocimiento y la cultura que están plasmados en los materiales impresos. La lectura
representa una de las conquistas más arduas y sublimes de la humanidad; se reconoce como un factor de marcada
relevancia en la civilización muy ligada con su desarrollo intelectual, espiritual, social.

En virtud de ellos, los desafíos de la vida moderna relacionados con la educación, la cultura, la creatividad, el
empleo inteligente de los ratos libres, la identidad, tienen todos que ver con el uso de la lectura, pueden ser orales
en las que se ha de atender la pronunciación, la entonación, la fluidez y la comprensión del contenido; hay que
recordar que interpretar es comprender el mensaje que se trasmite.

a) De todos los medios para adquirir cultura, la lectura es la principal


b) Actualmente existe a escala mundial un decrecimiento del hábito de lectura
c) La lectura representa una de las conquistas más arduas y sublimes de la humanidad
d) Interpretar es comprender el mensaje que se transmite

Respuesta correcta: a)

La tesis es una afirmación demostrable que refleja el punto de vista del autor o autora del texto. En este caso, el autor
o autora del texto afirma, según su punto de vista, que “de todos los medios para adquirir cultura, la lectura es la
principal”.

4)

De acuerdo con el Diccionario de la Real Academia Española, la tesis consiste en una proposición que se mantiene
con razonamientos o argumentaciones. Con base en esta definición, Identifique la tesis del autor en el siguiente
texto:

El poema Alturas de Macchu Picchu, de Pablo Neruda, aborda dos temas que son reiterativos en su obra poética;
por un lado, el apego a la naturaleza y la tierra, y articulado a esto, la noción de pertenencia a una determinada
cultura: la cultura latinoamericana.

Uno de los ejes centrales del poema es el sentido de la tierra y lo enterrado. Para trabajar esto, se vale de los
aspectos fónicos del significante; así, por ejemplo, en la tercera estrofa de la primera parte del poema leemos:
“Alguien me esperó entre los violines / Encontró un mundo como una torre enterrada [...]”.

En el segundo verso destaca el uso de la rr que resuena en la tierra y el enterramiento, y nos lleva a las raíces, a los
orígenes. La sonoridad de esta consonante, por un lado nos ofrece una sensación, casi sinestésica, de lo áspero, de
la roca y de la fortaleza, y por otro, nos conecta con los dialectos propios de la zona andina, recordemos que en
varias zonas de esta región se acentúa fonéticamente en el habla la rr, incluso en palabras que convencionalmente
se escribirían y se pronunciarían sólo con la consonante r.

Todas estas figuras ayudan al poeta a representar el escenario salvaje, místico y áspero de las ruinas, lo cual
metaforiza el sentido de pertenencia a la naturaleza y a la cultura latinoamericana.

58
a) Neruda, en el poema Alturas de Macchu Picchu, utiliza la reiteración de la letra -rr- para satirizar y
desprestigiar los dialectos andinos
b) Neruda, en el poema Alturas de Macchu Picchu, se vale del sonido de las palabras para ofrecer diversas
sensaciones al lector
c) Neruda, en el poema Alturas de Macchu Picchu, aborda el apego a la naturaleza y el sentido de pertenencia
a la cultura latinoamericana

d) En el poema Alturas de Macchu Picchu, Neruda representa el escenario salvaje, místico y áspero de las
ruinas a través de figuras retóricas


Respuesta correcta: c)

El autor del texto (que podría considerarse como un texto argumentativo) realiza un análisis del poema Alturas de
Macchu Picchu, de Pablo Neruda. Su tesis -entendida como proposición que se mantiene con razonamientos o
argumentos- es que Pablo Neruda aborda dos temas reiterativos en su obra poética: el apego a la naturaleza y la
noción de pertenencia a la cultura latinoamericana. ¿Cómo argumenta esta tesis? El autor dice que Neruda, por un
lado, se vale de elementos fónicos del significante para apelar a “lo enterrado” o a la tierra, y, por otro lado, emplea
la sonoridad de la rr para ofrecer una sensación de pertenencia a la zona andina.

5)

Con base en el texto, identifique la postura crítica:

Alguna vez escuché a un colega [...] regodearse haciendo gala de no haber leído nunca una novela, porque, según
decía, solo leía obras de importancia. El pobre hombre, que desde luego inspiraba compasión, no sabía que la única
manera de comprender hasta sus entresijos a ese extraño ser que es el hombre, pasa necesariamente por las
grandes novelas. Lo entendería si se diera el trabajo de leer Soldados de Salamina.

a) Las novelas contienen historias de grandes hombres que no leían libros


b) La falta de lectura de obras interesantes es comprensible en gente considerada importante
c) Quienes afirman no leer novelas inspiran compasión por su incomprensión de lo humano
d) Las grandes novelas permiten explicar de mejor manera el presente


Respuesta correcta: c)

El texto citado, al parecer, es parte de un artículo de opinión y, por tanto, se trata de un texto argumentativo que
plantea una postura particular o tesis del autor. ¿Cuál es esa tesis? Que una de las maneras de comprender la
dimensión humana es a través de la lectura de las novelas. Por ello, se realiza una crítica a quienes afirman no leer
novelas pues inspiran compasión por su incomprensión de lo humano.

En ningún momento la crítica se centra en los contenidos o en la función específica de las novelas, ni menos aún en
una comprensión hacia la no lectura por parte de gente considerada “importante”.

6)

Identifique un argumento a favor de la tesis en el siguiente texto:

Manifiesto surrealista

Amada imaginación, lo que más amo en ti es que jamás perdonas. Únicamente la palabra libertad tiene el poder
de exaltarme. Me parece justo y bueno mantener indefinidamente este viejo fanatismo humano. Sin duda alguna,
se basa en mi única aspiración legítima. Pese a tantas y tantas desgracias como hemos heredado, es preciso
reconocer que se nos ha legado una libertad espiritual suma. A nosotros corresponde utilizarla sabiamente. Reducir
la imaginación a la esclavitud, cuando a pesar de todo quedará esclavizada en virtud de aquello que con grosero
criterio se denomina felicidad, es despojar a cuanto uno encuentra en lo más hondo de sí mismo del derecho a la
suprema justicia. Tan sólo la imaginación me permite llegar a saber lo que puede llegar a ser, y esto basta para
mitigar un poco su terrible condena; y esto basta también para que me abandone a ella, sin miedo al engaño (como
si pudiéramos engañarnos todavía más). ¿En qué punto comienza la imaginación a ser perniciosa y en qué punto

59
deja de existir la seguridad del espíritu? ¿Para el espíritu, acaso la posibilidad de errar no es sino una contingencia
del bien?

a) Únicamente la palabra libertad tiene el poder de exaltarme


b) Amada imaginación, lo que más amo en ti es que jamás perdonas
c) Como si pudiéramos engañarnos todavía mas
d) Se nos ha legado una libertad espiritual suma

Respuesta correcta: d)

El texto citado corresponde a un ensayo y, en este sentido, se trata de un texto argumentativo. Todo texto
argumentativo está compuesto por una tesis, un argumento y una conclusión. La tesis es una afirmación demostrable
que refleja el punto de vista del autor o autora del texto. El argumento corresponde a la proposición o proposiciones
que sustentan o justifican la tesis. Y las conclusiones son ideas finales que dan cierre a lo expuesto, reafirmando como
verdadera la tesis planteada.

El texto citado plantea como tesis que únicamente la libertad tiene poder de exaltación y que esa libertad es una
legítima aspiración. ¿Cuál es el argumento para esta tesis? O más precisamente, ¿por qué esa libertad es una legítima
aspiración? Porque es un legado y solo depende de uno utilizarla sabiamente.

7)

Todos los enunciados del siguiente texto son argumentos, excepto:

La benévola postura europea hacia el tabaquismo podría convertirse en humo si los países persisten en adoptar
medidas para obligar a sus poblaciones a renunciar al vicio.
En Irlanda pronto entrará en vigor una controvertida
prohibición contra el tabaquismo en el lugar de trabajo. David Byrne, comisionado de sanidad de la Unión Europea,
ha alzado una cruzada para erradicar el tabaquismo en restaurantes y bares, a pesar de que un bloque de 15
naciones sigue desembolsando mil millones de dólares anuales en subsidios tabacaleros.

Otras estrategias europeas en la lucha contra el tabaquismo incluyen enérgicas medidas para evitar la venta a
menores de edad y un alza del impuesto al tabaco.
Esta no es la primera vez que el tabaco cae de la gracia europea.
El rey Jacobo I de Inglaterra, uno de los primeros monarcas del siglo XVII en condenar el hábito, escribió un famoso
tratado contra el tabaquismo.

a) La benévola postura europea hacia el tabaquismo podría terminar si se siguen aplicando medidas
restrictivas
b) En Irlanda pronto entrará en vigor una controvertida prohibición contra el tabaquismo en el lugar de
trabajo
c) David Byrne, de la Unión Europea, ha alzado una cruzada para erradicar el tabaquismo en restaurantes y
bares

d) El rey Jacobo I de Inglaterra, uno de los primeros opositores del hábito, escribió un tratado contra el
tabaquismo

Respuesta correcta: a)

Para empezar, el texto citado corresponde seguramente a un editorial y, por tanto, es un texto argumentativo.

Los argumentos son proposiciones que sustentan o justifican la tesis de todo texto argumentativo. La tesis del texto
citado es que, si los países europeos aplican medidas restrictivas que obliguen a la población a dejar de fumar, su
postura benévola hacia este vicio podría terminar. Para argumentar esta tesis se emplean varios ejemplos: la
prohibición del tabaquismo en los lugares de trabajo de Irlanda, la cruzada para erradicar este vicio en restaurantes
y bares por parte de David Byrne, y la publicación de un tratado en contra del tabaquismo de autoría del Rey Jacobo
I de Inglaterra. Estos tres ejemplos son los argumentos. Por tanto, de las opciones citadas, una es parte de la tesis y
NO del argumento: “la benévola postura europea hacia el tabaquismo podría terminar si se siguen aplicando medidas
restrictivas”.

60
Sobre los textos descriptivos

8)

Lea el siguiente texto e identifique a qué tipo pertenece:

A las seis de la mañana la ciudad se levanta de puntillas y comienza a dar sus primeros pasos. Una fina niebla
disuelve el perfil de los objetos y crea una atmósfera encantada. Las personas que recorren la ciudad a esta hora
parecen que están hechas de otra sustancia, que pertenecen a un orden de vida fantasmal. Las beatas se arrastran
penosamente hasta desaparecer en los pórticos de las iglesias. Los noctámbulos, macerados por la noche, regresan
a sus casas envueltos en sus bufandas y en su melancolía. Los basureros inician por la avenida Pardo su paseo
siniestro, armados de escobas y de carretas.

a) Argumentativo
b) Descriptivo
c) Expositivo o informativo
d) Narrativo

Respuesta correcta: b)

En efecto, el texto describe el ambiente de una ciudad. No es un texto argumentativo porque no propone una tesis
argumentada para convencer a los lectores. Tampoco es un texto narrativo porque no relata hechos en un espacio
temporal específico. Y tampoco es un texto informativo netamente.

9)

Lea el siguiente texto e identifique a qué tipo pertenece:

Buen mozo, petulante, de los que hacen resonar las espuelas, con unas patillas unidas al bigote, los dedos siempre
cubiertos de sortijas y vestido de llamativos colores, tenía trazas de bravucón y la animación fácil de un viajante de
comercio. Una vez casado, vivió dos o tres años de la fortuna de su mujer, cenando bien, levantándose tarde,
fumando en grandes pipas de porcelana, no volviendo a casa por las noches hasta después del teatro y
frecuentando los cafés. 27

a) Retrato
b) Prosografía
c) Etopeya
d) Paisaje

Respuesta correcta: a)

En efecto, el texto describe los rasgos físicos y psicológicos de una persona; es decir, es una combinación de
prosografía y etopeya, respectivamente. No se trata de un paisaje pues no describe un ambiente físico.

Sobre los textos expositivos o informativos

10)

¿A qué tipo de texto pertenece el siguiente párrafo?

Nuestra organización no tiene afiliación con ninguna secta o religión, ningún partido político, ninguna organización
ni institución, no toma parte en polémicas, no defiende ni combate ninguna causa. No existe cuota alguna para
hacerse miembro.

27 http://www.ejemplode.com/12-clases_de_espanol/4486-ejemplo_de_retrato.html

61
a) Poético
b) Argumentativo
c) Expositivo
d) Narrativo

Respuesta correcta: c)

El texto informa sobre las características generales de una organización: no tiene filiación con sectas o religiones,
partidos políticos y otras organizaciones, no toma parte de polémicas, no defiende ni combate ninguna causa y no
exige ninguna cuota para su membresía. NO es un texto poético, pues no emplea figuras literarias ni busca transmitir
emociones o sentimientos. Tampoco es un texto argumentativo, pues no presenta elementos que justifiquen o
argumenten porqué dicha organización, por ejemplo, no defiende ni combate ninguna causa. Y tampoco es un texto
narrativo pues no relata un acontecimiento que se desarrolle en un espacio temporal específico. Es, simplemente, un
texto expositivo o informativo pues expone lo que hace dicha organización.

11)

Lea el texto e identifique su tipología:

El Laboratorio de Referencia de la UE para la Salud de las Abejas (EURL por sus siglas en inglés), con sede en Anses,
Francia, publicó en abril los resultados del primer programa de vigilancia sobre el despoblamiento de las colmenas
en 17 países europeos. Los datos que se tomaron en más de 30 000 colmenas durante el 2012 y el 2013, muestran
unos índices de mortalidad invernal muy variables entre países (la horquilla cubre el 3,5% al 33,6%). En general, la
situación es más leve en España y otros países mediterráneos (por debajo del 10%) que en el norte del continente
(por encima del 20%). Las cifras contradicen a las del sector apícola español que denuncia mortandades entre el
20% y el 40% en un ejemplo más de lo dificultoso que resulta acordar los criterios y las metodologías en este campo.

a) Artículo de opinión
b) Editorial
c) Reportaje científico
d) Crónica

Respuesta correcta: c)

El texto describe los resultados de un programa de vigilancia del despoblamiento de las colmenas en 17 países
europeos. NO es un artículo periodístico de opinión pues NO expresa el sentir o el pensar de su autor. Tampoco es
un editorial pues NO explica, valora y juzga un hecho noticioso, en concordancia con un lineamiento ideológico del
medio de comunicación que lo acoge. Y NO se trata de una crónica periodística pues no narra hechos históricos o de
trascendencia de forma cronológica. Se trata de un reportaje científico pues expone los avances de una investigación
científica de carácter ecológico.

12)

Elija las características que corresponden a un texto científico:

1. Subjetividad
2. Claridad

3. Particularidad
4. Precisión
5. Objetividad
6. Expresividad
7. Verificabilidad

a) 1, 2, 4, 6
b) 1, 3, 5, 6
c) 2, 4, 5, 7
d) 3, 4, 5, 7

Respuesta correcta: c) 2, 4, 5, 7.

62
El texto científico es una producción escrita que aborda conceptos, teorías o cualquier otro tema sustentado en
conocimientos científicos. Entre otras características, los textos científicos: a) son claros, es decir, no generan dudas
ni cuestionamientos; b) son precisos, pues se ajustan fielmente a datos, cálculos, medidas o expresiones; c) son
verificables, esto es, susceptibles de comprobación posterior; d) utilizan un lenguaje formal con terminología
específica; e) no tienen una intención estética, sino que, más bien, busca plasmar conocimientos claros; por ello, f)
son objetivos, pues se sustentan en hechos y en procedimientos lógicos.

Por tanto, de las opciones de respuesta, la correcta es “2, 4, 5, 7”, es decir, “Claridad, precisión, objetividad y
verificabilidad”.

13)

A partir de la lectura de la siguiente noticia, indique sus partes correspondientes.

1  De última hora:

2  Suspenden a Director y Profesora de una escuela por falta de diligencia

3  Durante su turno una alumna fue agredida físicamente por sus compañeros.

En la escuela “José María Morelos”, ubicada en este Estado, el director de la misma junto con la profesora
del grupo 6° “A” fueron suspendidos por tiempo indeterminado, debido a que mientras este grupo se
encontraba a su cargo, una alumna se vio agredida verbal y también físicamente, cuando uno de sus
4  compañeros la tomó del cuello y comenzó a ahorcarla hasta dejarla inconsciente, la escena fue grabada y
se encuentra actualmente en varias redes sociales. Ambas autoridades son juzgadas gravemente, pero la
profesora es la que mayor carga tiene, pues se reportó que dejó a sus alumnos solos por un largo tiempo
sin justificación alguna

5  Reportero: Alejandra Jiménez

a. Copete o entradilla
b. Cuerpo de la noticia
c. Epígrafe
d. Fuente de la noticia
e. Título

a) 1a, 2c, 3b, 4d, 5e


b) 1c, 2e, 3a, 4b, 5d
c) 1a, 2e, 3c, 4b, 5d
d) 1d, 2e, 3c, 4b, 5a

Respuesta correcta: b) 1c, 2e, 3a, 4b, 5d.

En efecto, “De última hora” corresponde al epígrafe de la noticia. El título es “Suspenden a Director y Profesora de
una escuela por falta de diligencia”. El copete o entradilla va debajo del título: “Durante su turno una alumna fue
agredida físicamente por sus compañeros”. El cuerpo de la noticia corresponde al relato mismo de lo ocurrido. Y la
fuente es el nombre de la persona que escribe la noticia.

63
14)

¿Qué característica del texto científico se evidencia en el escrito?

La medición rutinaria de la temperatura atmosférica en estaciones meteorológicas ha impedido el monitoreo de


esta variable en diversas regiones del planeta desde finales del siglo XIX. Gracias a estos datos, es muy claro que la
temperatura media del planeta ha experimentado un incremento significativo de casi 0,5 grados centígrados si
tomamos como nivel base la temperatura media registrada entre los años 1961 a 1990, y de casi 1 grado centígrado
si la comparamos con la segunda mitad del siglo XIX (1850 – 1900). En estos datos es evidente que los años más
calurosos están concentrados durante las últimas décadas, esto es, de 1980 a la fecha.

a) Polisémico
b) Dialogado
c) Emotivo
d) Verificable

Respuesta correcta: d)

El texto nos habla acerca del incremento de la temperatura media del planeta en las últimas décadas, tomando como
punto de comparación los datos registrados desde fines del siglo XIX. NO se trata de un texto polisémico, pues no
nos lleva a varias interpretaciones o significados. Tampoco es un texto dialogado, pues no plantea una conversación
entre un hablante y un oyente con su respectiva alternancia de papeles. El texto tampoco es emotivo, pues no
expresa el estado emocional de una persona. Se trata de un texto verificable. ¿Por qué? Porque los datos referidos
al incremento de la temperatura que se describen en los enunciados, son susceptibles de comprobación.

15)

¿Qué características tiene el siguiente texto?

Un asteroide bautizado como 2012 TC4, que podría medir entre 12 y 40 m de diámetro [...], pasará peligrosamente
cerca de nuestro planeta el próximo 12 de octubre de 2017. Las conclusiones de las observaciones realizadas por
los expertos han sido publicadas en la revista Astrowatch.net.

Según Judit Györgyey Ries, astrónoma del observatorio McDonald de la Universidad de Texas (EEUU), la roca tiene
un 0,00055 % de probabilidades de que nos alcance. Para Detlef Koschny, director del programa de observación de
Objetos Cercanos a la Tierra (NEO) de la ESA, la probabilidad es de uno entre un millón. Sea como fuere, según las
estimaciones de los expertos, el asteroide pasará a una distancia de entre 13 200 y 433 200 km de la Tierra.

La primera vez que este asteroide se acercó a nuestro planeta fue en 2012. Pasó a una distancia de 94 800 km a
una velocidad de 26,5 km/h. Sin embargo, para averiguar a qué distancia exacta pasará la próxima vez es necesario
realizar más observaciones.
Romero, S. (2015, 14 de abril). Un asteroide pasará cerca de la tierra en 2017.

1. Verificable
2. Dialogado
3. Subjetivo
4. Objetivo

a) 1, 3
b) 1, 4
c) 2, 3
d) 2, 4

Respuesta correcta: b) 1, 4.

El texto informa sobre un asteroide que se acercará a la tierra en octubre de 2017, sustentándose en fuentes
científicas. No es un texto dialogado, pues no plantea una conversación entre un hablante y un oyente con su
respectiva alternancia de papeles. Tampoco es un texto subjetivo, pues no expone juicios de valor ni opiniones
personales del autor. Se trata de un texto verificable y objetivo. ¿Por qué? Porque los datos de los expertos en este
tema se sustentan en hechos objetivos (de experiencias previas), pero también susceptibles de comprobación.

64
16)

¿Qué característica del texto científico se evidencia en este escrito?

Para controlar los movimientos del Párkinson, hasta hace poco había que abrir la cabeza. Ahora, un grupo de
investigadores de Estados Unidos ha logrado hacer lo mismo, pero sin tener que abrir el cráneo. Lo han logrado
experimentando con las neuronas de los ratones, pero ya lo están investigando en humanos. Los distintos tipos de
neuronas se excitan a diferentes frecuencias, pero siempre bajas, en el rango de unas pocas decenas de hercios.
Sin embargo, los electrodos colocados a ambos lados del cerebro de los ratones del experimento emitían las señales
eléctricas en una frecuencia de 2000 hercios (kHz), exactamente un electrodo a 2Hz y el otro a 2,01 kHz.

a) Polisémico
b) Dialogado
c) Emotivo
d) Verificable

Respuesta correcta: d)

El texto informa sobre los experimentos realizados con ratones y humanos para controlar los movimientos del
Párkinson. No es un texto polisémico, pues no plantea significados distintos, sino complementarios, respecto de un
mismo tema. Tampoco es un texto dialogado, pues no plantea una conversación entre un hablante y un oyente con
su respectiva alternancia de papeles. Tampoco es un texto emotivo pues no provoca emociones en los lectores. Se
trata de un texto verificable. ¿Por qué? Porque los datos de los expertos en este tema se sustentan en hechos
objetivos (de experiencias previas y susceptibles de comprobación.

17)

Identifique la tesis del texto:

Estamos habituados a ver anuncios de complejos vitamínicos que nos recomiendan un aporte extra de este
nutriente en cualquier época del año. En la actualidad, muchas son las personas que los utilizan para sentirse mejor,
pues creen que es lo correcto; sin embargo, las cosas tomadas en exceso, incluso las que nos hacen bien –como las
vitaminas C y E y el selenio (un mineral antioxidante)- pueden ocasionarnos daño. Más de 2000 miligramos de
vitamina C pueden provocar diarrea; es probable que más de 1000 miligramos de vitamina E afecten la coagulación
sanguínea y que una dosis de selenio superior a 400 miligramos ocasione caída de cabello y descamación de las
uñas.

a) Más de 1000 miligramos de vitamina E pueden afectar a la coaugulación sanguínea


b) Las cosas tomadas en exceso, incluso las que nos hacen sentir bien, pueden ocasionarnos daño
c) Más de 400 miligramos de selenio pueden ocasionar caída de cabello y descamación de las uñas
d) Los anuncios de complejos vitamínicos recomiendan un aporte extra de estos nutrientes

Respuesta correcta: b)

El texto citado podría considerarse como un reportaje científico. Recordemos que un reportaje científico plantea una
tesis o postura del autor de carácter verificable, así como una serie de argumentos que sostienen esta tesis y que se
desarrollan a lo largo del cuerpo del texto. Así mismo, todo reportaje científico culmina con una conclusión o síntesis
del planteamiento del autor.

En este caso su autor plantea una tesis (en contraposición a lo que dicen los anuncios publicitarios y a lo que hacen
algunas personas): que las cosas tomadas en exceso, incluso las que nos hacen sentir bien, pueden ocasionarnos
daño. El argumento para esta tesis es que, probablemente, “más de 1000 miligramos de vitamina E afectan la
coagulación sanguínea” y que “más de 400 miligramos de selenio ocasionan la caída de cabello y la descamación de
las uñas”.

18)

Con base en el texto, identifique la tesis:

65
Durante siglos, los expertos han debatido sobre si el secreto de la estupenda resonancia de los violines Stradivarius,
considerados como muy superiores a los contemporáneos, reside en el barniz o en los tratamientos de la madera.
Henri Grissino-Mayer, de la Universidad de Tennesse, experto en la determinación del tiempo mediante el estudio
de los anillos de crecimiento de los troncos, y Lloyd Burckle, un climatólogo de la Universidad de Columbia, tienen
una nueva teoría: las propiedades acústicas de la madera se desarrollaron durante su crecimiento en virtud de
prolongados periodos de inviernos largos y veranos fríos. Estudios actuales sugieren que la calidad del sonido de
los instrumentos Stradivarius podría explicarse por el patrón climático que prevaleció en Europa y otras partes del
mundo entre 1645 y 1715. Estas condiciones favorecieron el crecimiento lento y uniforme de las maderas,
propiedades perfectas para la producción de instrumentos musicales.

a) La calidad del sonido de los Stradivarius podría explicarse por el clima que prevaleció entre 1645 y 1715
b) El patrón climático que prevaleció en Europa favoreció el crecimiento lento y uniforme de la madera de los
Stradivarius
c) Los instrumentos conocidos como Stradivarius tienen mejor resonancia que los instrumentos modernos
d) Las propiedades acústicas especiales en la madera se desarrollan durante su crecimiento en prolongados
periodos de frío

Respuesta correcta: c)

El texto citado podría considerarse como un reportaje científico. Recordemos que un reportaje científico plantea una
tesis o postura del autor de carácter verificable, así como una serie de argumentos que sostienen esta tesis y que se
desarrollan a lo largo del cuerpo del texto. Así mismo, todo reportaje científico culmina con una conclusión o síntesis
del planteamiento del autor.

En este caso, su autor plantea, como afirmación “demostrable”, que los violines Stradivarius tienen una estupenda
resonancia, a tal punto que son considerados por algunos expertos como muy superiores a los contemporáneos. Esa
es la tesis. Y de esta tesis se desprenden varios argumentos que explican por qué los violines Stradivarius tienen una
magnífica resonancia: por su barniz, por los tratamientos del material, por las propiedades de su madera durante su
crecimiento en virtud de los prolongados periodos de inviernos lagos y veranos fríos, o por patrones climáticos que
hacen que el crecimiento de dicha madera sea lento y uniforme. El texto carece de una conclusión.

19)

Todos los enunciados son argumentos, excepto:

Mucho se ha dicho acerca de las razones por las cuales los instrumentos musicales de madera Stradivarius,
fabricados entre finales del siglo XVII y comienzos del XVIII, tienen tonalidades superiores a los instrumentos
modernos. Las teorías van desde las habilidades que tenían los artesanos hasta la geometría y el diseño, pasando
por una serie de técnicas secretas tales como los barnices especiales, el secado de la madera, el tiempo de
almacenamiento y el uso de maderas viejas provenientes de estructuras históricas. Sin embargo, estudios actuales
sugieren que la calidad del sonido de los instrumentos Stradivarius de esta época podría explicarse por el patrón
climático que prevaleció en Europa y otras partes del mundo entre 1645 y 1715. Ese periodo se caracterizó por una
dramática reducción de la actividad solar, lo que provocó inviernos fríos y prolongados. Estas condiciones
favorecieron el crecimiento lento y uniforme de las maderas, propiedades perfectas para la producción de
instrumentos musicales.

a) la calidad del sonido de los Stradivarius podría explicarse por el clima que prevaleció entre 1645 y 1715
b) el frío favoreció el crecimiento lento y uniforme de las maderas de los Stradivarius
c) los instrumentos conocidos como Stradivarius tienen tonalidades superiores a los instrumentos modernos

d) el periodo entre 1645 y 1715 se caracterizó por una dramática reducción de la actividad solar

Respuesta correcta: c)

El texto citado podría considerarse como un reportaje científico. Recordemos que un reportaje científico plantea una
tesis o postura del autor de carácter verificable, así como una serie de argumentos que sostienen esta tesis y que se
desarrollan a lo largo del cuerpo del texto. En este caso se plantea, como afirmación “demostrable”, que los violines
Stradivarius tienen tonalidades superiores a los instrumentos modernos. Esa es la tesis. Y de esta tesis se desprenden
varios argumentos: el clima que prevaleció entre 1645 y 1715, y el crecimiento lento y uniforme de las maderas del
instrumento gracias a las temperaturas frías que fueron ocasionadas por una dramática reducción de la actividad
solar.

66
De las opciones citadas, por tanto, una corresponde a la tesis y no a los argumentos que la justifican: “los
instrumentos, conocidos como Stradivarius, tienen tonalidades superiores a los instrumentos modernos”.


20)

Identifique el enunciado que forma parte del cuerpo del texto de divulgación científica.

Científicos de la Universidad de California curaron las cataratas congénitas de doce bebés usando células madre,
luego de tener éxito en experimentos con conejos y monos. La cirugía tradicional requiere de 6 milímetros para
reemplazar el cristalino anómalo por uno artificial, pero implica destruir células madre que protegen al ojo. El
nuevo método permite una abertura de solo 1,5 milímetros para extraer el tejido afectado y provocar que las
células madre del cristalino regeneren uno nuevo sin cataratas.

En las pruebas iniciales, el procedimiento evidenció una tasa mucho menor de complicaciones frente a la técnica
tradicional. El primer bebé operado hace dos años todavía tiene una buena visión y se espera que en la niñez no se
presente inconvenientes. Sin embargo, la cirugía no funciona para adultos de edad avanzada porque sus células se
regeneran lentamente. Aun si la nueva técnica solo funcionara para niños, sigue significando un avance muy
importante.

a) La cirugía no funciona en adultos de edad avanzada, sus células se regeneran lentamente


b) Científicos de la Universidad de California curaron las cataratas congénitas de doce bebés
c) Aún si la nueva técnica solo funcionara para niños, sigue significando un avance muy importante
d) El procedimiento evidenció una tasa menor de complicaciones frente a la técnica tradicional

Respuesta correcta: d)

Todo texto de divulgación científica plantea una tesis o postura del autor de carácter verificable, así como una serie
de argumentos que sostienen esta tesis y que se desarrollan a lo largo del nudo o cuerpo del texto.

El texto citado afirma que la cirugía de cataratas congénitas con células madre ha sido aplicada con éxito en doce
bebés; esta afirmación es la tesis. El nudo o cuerpo del texto explica y argumenta en qué consiste esta cirugía,
evidenciando cómo este procedimiento tiene menores complicaciones respecto de las técnicas tradicionales. Se
concluye indicando que esta cirugía no funciona en adultos de edad avanzada y que, sin embargo, sigue significando
un avance importante.

Por tanto, del listado de opciones de respuesta, ésta forma parte del cuerpo del texto: “El procedimiento evidenció
una tasa menor de complicaciones frente a la técnica tradicional”.

Sobre los textos narrativos

21)

Lea el texto e identifique su tipología:

Los músicos llegaron temprano y todos los técnicos ya tenían preparados los equipos en el escenario de tal manera
que pudieron brindar al público un espectáculo inolvidable. Solo restaba una cosa: que la banda saliera a tocar.
Luego de una media hora, a las diez de la noche, el concierto empezó para la alegría y la euforia de los miles de
fanáticos presentes en aquel estadio.

a) Editorial
b) Texto de divulgación científica
c) Crónica periodística
d) Noticia

Respuesta correcta: c)

En efecto, el texto citado narra un acontecimiento real de forma cronológica. No es una noticia pues no se ajusta a
su estructura (epígrafe, título, entradilla, cuerpo, foto y fuente). Tampoco es un editorial pues no manifiesta la opinión

67
del escritor ni intenta convencer a los lectores sobre algo. Y no es un texto de divulgación científica pues no expone
resultados, inventos o descubrimientos científicos.

22)

Con base en el texto, elija los elementos de la estructura narrativa.

El viento y el mar


Los mantas creían que el viento y el mar eran hermanos. Apostaron para ver cuál de los dos era el más destructivo.
El viento volteó las embarcaciones y el mar arrasó con las pequeñas cabañas. Desde entonces nadie vive en la
playa.

1. Título

2. Tema
3. Inicio
4. Nudo
5. Opinión
6. Desenlace

a) 1, 2, 3, 5
b) 1, 3, 4, 6
c) 2, 3, 4, 5
d) 2, 4, 5, 6

Respuesta correcta: b)

El texto citado corresponde a un mito y, en esta medida, se trata de un texto narrativo. La estructura básica de un
texto narrativo es la siguiente: a) título o nombre del texto, b) inicio o introducción, en donde se presentan los
personajes y ambientes “normales” de la historia, b) nudo o cuerpo del texto, que inicia cuando aparece un elemento
de tensión que rompe con aquella ”normalidad” presentada en la introducción, y c) desenlace o fin, es decir, aquellos
hechos que re-ordenan los elementos de tensión para establecer una nueva “normalidad”; el desenlace deja
planteado cómo serán las cosas a partir de ese momento.

Justamente, todas las partes mencionadas se identifican claramente en el texto citado:

• Título: “El viento y el mar”


• Inicio: “Los mantas creían que el viento y el mar eran hermanos”
• Nudo: “Apostaron para ver cuál de los dos era el más destructivo. El viento volteó las embarcaciones y el mar
arrasó con las pequeñas cabañas”.
• Desenlace: “Desde entonces nadie vive en la playa”.

23)

Relacione los componentes del siguiente texto con su situación.

La quebrada fue durante muchos siglos la madriguera de los duendes. En ella se confundieron los pálidos espíritus
cristianos con las almas rojas de los incas, descendientes legítimos del sol. Diablos y vírgenes, ascetas y
emperadores, hicieron su palacio de jaspe en el abismo sin fondo de la quebrada. La quebrada, durante la colonia
y la vida libre, fue alcoba de corazones transparentes, que refinaron las caricias prohibidas. Pero cuando nació el
siglo XX, se introdujo en El Tejar, como un apache, la luz eléctrica y asesinó a todos los fantasmas.

Componentes:

1. Ambiente
2. Acción
3. Resultado

68
Situación:

a. La luz eléctrica invadió la quebrada


b. Los espectros y aparecidos vivían en la quebrada
c. Todos los fantasmas fueron asesinados

a) 1a, 2c, 3b
b) 1b, 2a, 3c
c) 1b, 2c, 3a
d) 1c, 2a, 3b

Respuesta correcta: b) 1b, 2a, 3c.

El texto citado corresponde a una leyenda y, por tanto, se trata de un texto narrativo. Los textos narrativos relatan
hechos o acontecimientos que se desarrollan en un espacio temporal específico. La leyenda citada, por ejemplo,
relata sucesos en torno a la quebrada del Tejar durante las épocas colonial, independentista y de inicios del siglo XX.

Pues bien. En todo texto narrativo es posible identificar un ambiente, una acción y un resultado. El ambiente consiste
en el entorno físico, psicológico o social donde los personajes -en este caso, los duendes, espíritus y otros seres
míticos- desarrollan la acción. El ambiente se correspondería con la siguiente frase: “los espectros y aparecidos vivían
en la quebrada”.

La acción, por su parte, corresponde a aquellos acontecimientos y situaciones que provocan o inciden en el resultado
o punto culminante de la narración; en este caso, la introducción de la luz eléctrica en la quebrada (acción) provocó
que los fantasmas fueran asesinados (resultado).

24)

Lea el texto e Identifique el subgénero narrativo.

Sorprendió un león a una liebre que dormía tranquilamente. Pero cuando estaba a punto de devorarla, vio pasar a
un ciervo. Dejó entonces a la liebre por perseguir al ciervo. Despertó la liebre ante los ruidos de la persecución, y
no esperando más, huyó. Mientras tanto el león, que no pudo dar alcance al ciervo, ya cansado, regresó a tomar la
liebre, pero no la encontró. Entonces se dijo el león:

- Bien me lo merezco, pues teniendo ya una presa en mis manos, la dejé para ir tras la esperanza de obtener una
mayor.

Si tienes en tus manos un pequeño beneficio, cuando busques uno mayor, no abandones el pequeño que ya tienes,
hasta tanto no tengas realmente en tus manos el mayor.


a) Mito
b) Cuento
c) Novela
d) Fábula


Respuesta correcta: d)

La fábula es una pequeña narración que siempre nos deja una enseñanza o moraleja. Sus personajes pueden ser
animales, seres humanos o seres fantásticos. Es el caso del ejemplo citado, la fábula del León y la Liebre cuya moraleja
es: “si tienes en tus manos un pequeño beneficio, cuando busques uno mayor, no abandones el pequeño que ya
tienes, hasta tanto no tengas realmente en tus manos el mayor”.

69
Sobre los textos dramáticos

25)

Relacione el subgénero dramático con su característica.

Subgénero:

1. Tragicomedia
2. Tragedia
3. Comedia

Característica:

a. Es la representación humorística de un tema humano


b. Relata una historia o un incidente imaginario
c. Representa conflictos cuyo desenlace es fatal
d. Mezcla lo trágico y humorístico

a) 1a, 2b, 3d
b) 1a, 2c, 3b
c) 1c, 2b, 3d
d) 1d, 2c, 3a

Respuesta correcta: d) 1d, 2c, 3a

La tragicomedia mezcla lo trágico con lo humorístico; la tragedia representa conflictos cuyo desenlace es fatal; y la
comedia representa un tema humano de forma humorística.

26)

Identifique el subgénero dramático al que pertenece el fragmento de la obra Electra.

Electra: Sucumbiremos, si es preciso, habiendo vengado a nuestro padre.

Crisótemis: Nuestro padre mismo, lo sé, me perdona esto.


Electra: Sólo a los cobardes pertenece aprobar esas palabras.

Crisótemis: ¿No cederás? ¿No serás persuadida por mí?

Electra: No, por cierto. No soy insensata hasta ese punto.


Crisótemis: Iré, pues, allí donde debo ir.


Electra: ¿Adónde vas? ¿A quién llevas esas ofrendas sagradas?

Crisótemis: Mi madre me envía a hacer libaciones a la tumba de mi padre.

Electra: ¿Qué dices? ¿Al más detestado de los mortales?

Crisótemis: Que ella misma mató. Eso es lo que quieres decir.


Electra: ¿Qué amigo la ha aconsejado? ¿A qué se debe que le haya placido eso?

Crisótemis: A un terror nocturno, según me ha parecido.


Electra: ¡Oh, Dioses paternos, venid! ¡Venid ahora!


70
a) Farsa
b) Comedia
c) Autosacramental
d) Tragedia

Respuesta correcta: d)

La tragedia presenta conflictos entre personajes -generalmente reyes o héroes- que son víctimas de terribles pasiones
que les llevan a la destrucción y a la muerte. La obra Electra, justamente, es un ejemplo de tragedia.

27)

Seleccione las características de la tragedia.

1. Causa hilaridad y burla



2. Personaje principal es exaltado

3. Acaba con muerte y destrucción

4. Mezcla lo risible con lo doloroso

5. Enfrenta inevitablemente al destino
6. Tiene escenas cómicas

a. 1, 2, 4
b. 1, 5, 6
c. 2, 3, 5
d. 3, 4, 6

Respuesta correcta: c) 2, 3, 5.

En efecto, en la tragedia el personaje principal (un héroe o alguien de la nobleza) es exaltado pues enfrenta
inevitablemente al destino. Además, toda tragedia culmina con destrucciones o muertes.

71
Tema 4. Idea principal y consecuencia lógica de un texto

Texto resumen

La idea principal de un texto

Todo texto está formado por oraciones que expresan ideas. La idea principal o central es aquella
que encierra en sí todo lo que exponen las demás oraciones pues proporciona un mensaje
sintético o concluyente. Las demás oraciones son ideas secundarias o de apoyo y anteceden,
amplían, justifican, ejemplifican y/o refutan la idea principal, es decir, dependen de ella.

El gráfico ilustra la idea principal y las ideas secundarias del siguiente texto:

Aunque hay varias marcas de celulares, todos sirven para lo mismo. La mayoría de la población
tiene acceso a ellos. Ocurre que el uso de los celulares ha facilitado la comunicación. Estos
aparatos permiten salvar distancias en cualquier lugar del planeta.

72
La idea principal puede estar ubicada en cualquier parte del texto y presentarse de forma
explícita o directa o de forma implícita o que necesite de deducción. Por lo general, en los textos
argumentativos y explicativos -principalmente científicos- esta idea aparece de forma explícita:
corresponde a la tesis o al planteamiento teórico del autor. En los textos descriptivos y poéticos
suele ser distinto: hay que inferir la idea principal implícita, buscando una frase concluyente o
que englobe sus contenidos.

Ahora bien. Si en una pregunta del Examen Ser Bachiller se propone un listado de opciones y se
solicita seleccionar de ellas la que corresponda a la idea principal de un texto puntual, ¿qué
estrategias podrían emplearse para escoger la que sea correcta?

María Consuelo Acosta28 plantea dos medios:

• Seleccionar una opción y suprimirla del texto. Si el texto queda sin una idea concluyente, con
seguridad, dicha opción será la correcta.

• Seleccionar una opción y mirar si las que no fueron seleccionadas dependen de ella; es decir,
si se presentan como antecedentes, aclaraciones, ejemplos o argumentos, relacionándose
con ella de modo natural. Si ello ocurre, la opción seleccionada es la correcta.

Pongamos un ejemplo de un texto cuya idea principal se encuentre explícita:

Más de 1000 turistas caminan diariamente a lo largo de los senderos, escaleras y terrazas de
granito que el pueblo inca construyó entre los picos sagrados de su Imperio andino. Según la
revista Rutas del Perú, los enormes bloques de piedra están peligrosamente erosionados y los
templos del pueblo Choquequirao, una antigua ciudad incaica ubicada en el Cusco, en la
provincia de la Convención, sufren bajo el polvo levantado por la construcción de nuevos hoteles.

El artículo advierte que el turismo descontrolado podría acabar con uno de los más grandes
tesoros arqueológicos del mundo y recomienda al Gobierno peruano hacer cambios drásticos en
el manejo del turismo de la región al reducir el número de visitantes que recorren una distancia
de 60 kilómetros durante 4 días.

Seleccione la idea central del texto:

a. El turismo descontrolado podría acabar con las ruinas de Choquequirao


b. La revista Rutas del Perú realizó un artículo sobre el impacto del turismo en Choquequirao
c. La cultura inca construyó uno de los tesoros arqueológicos más importantes en los Andes
d. En Choquequirao, los turistas recorren 60 kilómetros durante 4 días

En este caso, la opción correcta es a) El turismo descontrolado podría acabar con las ruinas de
Choquequirao. ¿Por qué? Sigamos las recomendaciones de María Consuelo Acosta:

28Acosta, María Consuelo (2013). Estrategias para localizar la idea principal de un texto. En:
http://larondadelaspalabras.blogspot.com/2013/02/estrategias-para-localizar-la-idea.html

73
• Suprimamos esta oración del texto citado. Al hacerlo, ¿identificamos un significado
concluyente en dicho texto? Pues no.

• Ahora, leamos las otras opciones y preguntémonos si dependen de la oración seleccionada.


Pues sí. Se trata de oraciones que se presentan a manera de antecedentes de dicha oración
y no son ideas concluyentes.

Ahora pongamos un ejemplo de un texto cuya idea principal se encuentre implícita:

Hemos progresado muy deprisa, pero nos hemos encarcelado a nosotros mismos. El
maquinismo, que crea abundancia, nos deja en la necesidad. Nuestro conocimiento nos ha hecho
cínicos. Nuestra inteligencia, duros y secos. Pensamos demasiado, sentimos muy poco. Más que
máquinas necesitamos más humanidad. Más que inteligencia, tener bondad y dulzura (Chaplin,
Charles. Discurso del Gran Dictador)

Identifique la inferencia adecuada del texto citado:

a) El advenimiento del progreso ha deshumanizado al ser humano


b) La abundancia nos crea necesidad
c) El ser humano debe tener una actitud crítica frente al progreso
d) La inteligencia es un peligro para la humanidad

En este caso, la opción correcta es a) El advenimiento del progreso ha deshumanizado al ser


humano. ¿Por qué? Porque se trata de una frase concluyente del texto citado. En él, se plantea
que el desarrollo tecnológico y los elementos del progreso devenidos del conocimiento, nos ha
vuelto duros, secos y con pocos sentimientos. Ergo: el advenimiento del progreso ha
deshumanizado al ser humano.

Las otras opciones de respuesta no son concluyentes del texto.

La consecuencia lógica de un texto

La consecuencia lógica es la conclusión que se deriva de un conjunto de premisas de un


argumento deductivamente válido. Como en general se argumenta para establecer una
conclusión, se procura que las premisas o proposiciones impliquen a la conclusión y que sean
verdaderas. Siendo así, la consecuencia lógica es una proposición final verdadera y no una
“opinión”. Pongamos un ejemplo sencillo:

• Premisa o proposición 1: Todos los mamíferos son de sangre caliente.

• Premisa o proposición 2: Todos los humanos son mamíferos.

• Consecuencia lógica o conclusión: Por lo tanto, todos los humanos son de sangre caliente.

Veamos otro ejemplo más gráfico:

74
Ahora bien. Si en una pregunta se propone un listado de opciones y se solicita seleccionar de
ellas la que corresponda a la consecuencia lógica de un texto puntual, hay que asegurarte de
que la opción correcta sea verdadera, en función de las premisas planteadas dentro del texto.
Por ejemplo:

Identifique la consecuencia lógica del siguiente texto:

En un país se establecieron aranceles para algunos objetos importados. A pesar de los intentos
de fomentar la producción nacional, se continúa importando celulares de todo tipo, incluidos los
de última tecnología. Por lo tanto:

a) Probablemente los celulares importados pagan aranceles


b) Los aranceles se aplicarán para los celulares
c) Los aranceles no se aplicarán para los celulares
d) Quizá ese país reduzca los aranceles para los celulares

Para responder correctamente, dividamos el texto en premisas:

• Premisa o proposición 1: En un país se establecieron aranceles para ALGUNOS objetos


importados.

• Premisa o proposición 2: Se continúa importando celulares de todo tipo, incluidos los de


última tecnología.

Como no sabemos si los celulares que se siguen importando forman parte de aquellos objetos a
los cuales se establecieron aranceles (y que son ALGUNOS objetos, no TODOS), podríamos
deducir que PROBABLEMENTE estos pagarán aranceles.

Es decir, la opción correcta es a) Probablemente los celulares importados pagan aranceles.

No son correctas las demás opciones por lo siguiente:

75
• NO podemos decir necesariamente que “los aranceles se aplicarán para los celulares” pues
algunos de ellos podrían ser de origen nacional (aun cuando en su mayoría no sean
consumidos); además, no sabemos si los celulares importados forman parte de aquellos a los
que se gravarán impuestos.

• Tampoco podemos decir que “los aranceles no se aplicarán para los celulares” o que se
realizará una reducción de dichos impuestos, pues es probable que los celulares importados
sí estén contemplados dentro del listado de objetos a los que se gravarán aranceles.

Referencias bibliográficas

• Acosta, María Consuelo (2013). Estrategias para localizar la idea principal de un texto. En:
http://larondadelaspalabras.blogspot.com/2013/02/estrategias-para-localizar-la-idea.html

• Stanford Encyclopedia of Philosophy (2005). Logical Consequence. Disponible en:


https://plato.stanford.edu/archives/fall2009/entries/logical-consequence/

Banco de preguntas y respuestas

1)

Identifique la idea principal del texto:

Pachacuti recibió a las mujeres y a los niños con piedad y mansedumbre. Les dijo que la culpa no era de ellas –las
mujeres- ni aun de los pobres guerreros conducidos a la sublevación y a la muerte; que los responsables únicos
eran los soberbios y ambiciosos curacas de los ayllus chancas, que habían tenido la audacia de creer que podían
derrotar a los hijos del Sol.

a) Pachacuti dijo que ni las mujeres, ni los pobres guerreros tenían la culpa
b) Pachacuti dijo que los únicos responsables eran los soberbios y ambiciosos curacas de los ayllus chancas
c) Pachacuti recibió a las mujeres y a los niños con piedad y mansedumbre
d) Los curacas de los ayllus chancas habían tenido la audacia de creer que podían derrotar a los hijos del Sol

Respuesta correcta: a)

La idea principal o central es la idea conclusiva o que sintetiza todas las ideas de un texto. En este caso, la idea principal
está explícita, es decir, está escrita en una parte del texto: “ni las mujeres ni los pobres guerreros fueron culpables”.

Las demás oraciones son secundarias. Una de ellas justifica la idea principal: “Pachacuti recibió a las mujeres y a los
niños con piedad y mansedumbre”. Y dos de ellas la complementan: “Pachacuti dijo que los únicos responsables eran
los soberbios y ambiciosos curacas de los ayllus chancas” y “Los curacas de los ayllus chancas habían tenido la audacia
de creer que podían derrotar a los hijos del Sol”.

76
2)

Identifique la idea principal del texto:

Niijima, un volcán que emergió del océano en noviembre de 2013, se unió a una isla cercana que se formó de un
volcán, cuya última erupción fue hace 40 años. Niijima surgió a unos 500 m del antiguo volcán Nishinoshuma.
Ahora, según las observaciones hechas a fines marzo, las islas se han vuelto una y miden cerca de 1 km de ancho;
en su punto más alto, la nueva isla tiene 60 m sobre el nivel del mar. Eso es el triple del punto más alto observado
en diciembre, según publicó la CNN.

a) La isla Niijima se formó tras una erupción volcánica en el océano, en noviembre de 2013

b) La nueva isla tiene un kilómetro de ancho y en su punto más alto alcanza 60 metros sobre el nivel del mar
c) La unión de dos formaciones volcánicas, una antigua y otra reciente, ha dado lugar a la formación de una
nueva isla
d) El volcán Nishinoshuma, que conforma la isla antigua, se formó hace aproximadamente 40 años

Respuesta correcta: c)

La idea principal o central es la idea conclusiva o que sintetiza todas las ideas de un texto. En este caso, la idea principal
es explícita, es decir, aparece escrita de alguna manera dentro del texto: dos formaciones volcánicas, una antigua y
una reciente, se unieron y dieron lugar a una nueva isla. Las demás oraciones son secundarias pues anteceden o
complementan dicha idea.

3)

Identifique la idea principal del texto:

En el Índice de Aptitud en inglés, publicado en el 2014, Argentina fue el único país de la región que se ubicó en la
lista de países con alta aptitud hacia este idioma y solo República Dominicana alcanzó la columna de moderada
aptitud. El resto de naciones de la región fueron catalogados como de baja aptitud y muy baja aptitud, superados
por naciones asiáticas como Corea del Sur, Japón, Vietnam y China, que ni siquiera comparten el mismo alfabeto
que el inglés.

Argentina fue la sorpresa en la región y eso se debe a las mejoras en su sistema de educación, explica Minh Ngan,
coautor del informe. El gobierno ha invertido en la educación pública y en la capacitación de los maestros en inglés
para asegurar un sólido nivel en el idioma. Por eso, tal vez, los países que ocupan los primeros cinco puestos del
índice como Dinamarca, Holanda y Suecia tienen un envidiable sistema de educación pública.

a) Dinamarca, Holanda y Suecia tienen un envidiable sistema de educación


b) Argentina se colocó en la lista de países con alta aptitud para el inglés
c) En el año 2014 se publicó el informe Índice de Aptitud en inglés
d) Argentina ha invertido grandes cantidades de dinero en educación

Respuesta correcta: b)

La idea principal o central es la idea conclusiva o que sintetiza todo lo que se dice en un texto. En este caso, la idea
principal es explícita, es decir, aparece escrita dentro del texto: “Argentina se colocó en la lista de países con alta
aptitud para el inglés”. Las demás opciones son ideas secundarias.

4)

Identifique la idea principal del texto:

Más de 1000 turistas caminan diariamente a lo largo de los senderos, escaleras y terrazas de granito que el pueblo
inca construyó entre los picos sagrados de su Imperio andino. Según la revista Rutas del Perú, los enormes bloques
de piedra están peligrosamente erosionados y los templos del pueblo Choquequirao, una antigua ciudad incaica
ubicada en el Cusco, en la provincia de la Convención, sufren bajo el polvo levantado por la construcción de nuevos
hoteles. El artículo advierte que el turismo descontrolado podría acabar con uno de los más grandes tesoros

77
arqueológicos del mundo y recomienda al Gobierno peruano hacer cambios drásticos en el manejo del turismo de
la región al reducir el número de visitantes que recorren una distancia de 60 kilómetros durante 4 días.

a) El turismo descontrolado podría acabar con las ruinas de Choquequirao


b) La revista Rutas del Perú realizó un artículo sobre el impacto del turismo en Choquequirao
c) La cultura inca construyó uno de los tesoros arqueológicos más importantes en los Andes
d) En Choquequirao, los tutistas recorren 60 kilómetros durante 4 días

Respuesta correcta: a)

La idea principal o central es la idea conclusiva o que sintetiza todo lo que se dice en un texto. Si suprimes la opción
“a) El turismo descontrolado podría acabar con las ruinas de Choquequirao”, el texto no tendría un significado
concluyente.

5)

Identifica la idea central del texto:

Hemos progresado muy deprisa, pero nos hemos encarcelado a nosotros mismos. El maquinismo, que crea
abundancia, nos deja en la necesidad. Nuestro conocimiento nos ha hecho cínicos. Nuestra inteligencia, duros y
secos. Pensamos demasiado, sentimos muy poco. Más que máquinas necesitamos más humanidad. Más que
inteligencia, tener bondad y dulzura (Chaplin, Charles. Discurso del Gran Dictador)

a) El advenimiento del progreso ha deshumanizado al ser humano


b) La abundancia nos crea necesidad
c) El ser humano debe tener una actitud crítica frente al progreso
d) La inteligencia es un peligro para la humanidad

Respuesta correcta: a)

La idea principal o central es la idea conclusiva o que sintetiza todas las ideas de un texto. En este caso, la idea principal
está implícita, es decir, hay que inferirla. En el texto citado se plantea que el desarrollo tecnológico y los elementos
del progreso devenidos del conocimiento nos ha vuelto duros, secos y con pocos sentimientos. Ergo: el advenimiento
del progreso ha deshumanizado al ser humano.

Las otras opciones de respuesta no son concluyentes del texto.

6)

Identifique la idea principal del texto:

La Alhambra no es solo uno de los mayores tesoros arquitectónicos de España, sino una de las más impresionantes
maravillas del mundo. Al pasar por su austero exterior podrá vislumbrar unas fuentes increíbles que le inundarán
de sonidos orgánicos y podrá apreciar la belleza de sus cuidados y estéticos jardines; un verdadero espectáculo de
olores y colores. Diariamente, más de 2500 turistas pueden visitar uno de los mayores tesoros arquitectónicos de
España. Según la Revista Mundos, la Alhambra actualmente soporta los efectos de la contaminación de las
edificaciones aledañas. En este artículo se advierte que el turismo descontrolado podría acabar con uno de los más
grandes tesoros arqueológicos de España y recomienda al gobierno de este país hacer cambios drásticos en el
manejo del turismo de la región al reducir el número de visitantes de este inusual paraje.

a) El turismo descontrolado y la contaminación podrían acabar con uno de los mayores tesoros españoles, La
Alhambra
b) La revista Mundos realizó un artículo sobre el impacto que el turismo descontrolad tendrá en La Alhambra
c) La Alhambra, uno de los tesoros españoles, soporta los efectos de la contaminación de edificaciones
cercanas.
d) Una recomendación del artículo de la revista es realizar cambios drásticos en el manejo del turismo en La
Alhambra

78
Respuesta correcta: a)

La idea principal o central es la idea conclusiva o que sintetiza todo lo que se dice en un texto. Si suprimes la opción
“a) El turismo descontrolado y la contaminación podrían acabar con uno de los mayores tesoros españoles, La
Alhambra”, el texto no tendría un significado concluyente. Además, las demás opciones (oraciones secundarias)
dependen de ella.

7)

Identifique la idea principal del texto:

Santorini, a nuestro entender, es la joya de las Cycladas. Es absolutamente diferente a todas las islas Cycladas o a
cualquier isla del Egeo. Su configuración geológica le da esa belleza salvaje y única. En Santorini, cada detalle es
belleza. Tiene forma de media luna y en su parte interior, donde antiguamente estaba el gran volcán, existen islas
más pequeñas pero muy bellas. Diariamente, más de 1500 turistas pueden visitar el Museo de la Prehistoria de
Santorini. Según la revista Grecos, el Museo de la Prehistoria de Santorini actualmente soporta los efectos de la
contaminación de las edificaciones aledañas. En este artículo se advierte que el turismo podría acabar con uno de
los más grandes tesoros arqueológicos de Grecia y recomienda al gobierno de este país hacer cambios drásticos en
el manejo del turismo de la región al reducir el número de visitantes de este inusual paraje.

a) El turismo descontrolado y la contaminación podrían acabar con el Museo de Prehistoria de Santorini


b) La revista Grecos realizo un artículo sobre el impacto que el turismo descontrolado tendrá en el Museo de
la Prehistoria de Santorini
c) El Museo de la Prehistoria de Santorini soporta los efectos de la contaminación de edificaciones cercanas
d) La recomendación del artículo es realizar cambios drásticos en el manejo del turismo en el Museo de la
Prehistoria de Santorini

Respuesta correcta: a)

La idea principal o central es la idea conclusiva o que sintetiza todo lo que se dice en un texto. Si suprimes la opción
“a) El turismo descontrolado y la contaminación podrían acabar con el Museo de Prehistoria de Santorini”, el texto
no tendría un significado concluyente. Además, las demás opciones (oraciones secundarias) dependen de ella.

8)

Identifique la idea principal del texto:

En Tenochtitlán, capital azteca, la alimentación no suponía problema alguno, gracias a sus ingeniosos sistemas de
cultivo basados en las chinampas, plataformas artificiales construidas sobre palos y lodo por encima del lago, en
las que se recogían hasta seis cosechas anuales. Las chinampas permitían también una gran producción de
legumbres, todavía muy presentes y no solo en la cocina mexicana. Las bebidas más extendidas entre los aztecas
fueron el pulque, extraído del zumo del maguey o agave, y un aguardiente llamado mezcal. Estaba también muy
difundido el cultivo del cacao, con el que los aztecas hacían chocolate mezclándolo con harina de maíz y vainilla,
una de sus aportaciones más populares a la gastronomía mundial. Los granos de cacao tenían el valor de moneda
de cambio y se valoraban más que el oro.

a) Los granos de cacao tenían valor de cambio entre los aztecas


b) Las chinampas eran plataformas artificiales de cultivo sobre los lagos
c) El cultivo en chinampas permitía a los aztecas hasta seis cosechas al año
d) La chinampa fue clave para la exitosa producción de alimentos entre los aztecas

Respuesta correcta: d)

La idea principal o central es la idea conclusiva o que sintetiza todo lo que se dice en un texto. En este caso, la idea
principal es explícita, es decir, aparece dentro del texto: la alimentación entre los aztecas no suponía problema alguno
gracias a las chinampas. De esta idea se derivan las demás y la complementan, a través de una descripción de la
riqueza gastronómica azteca.

79
9)

Identifique la idea principal del texto:

Un grupo de científicos de la Universidad de Michigan (Estados Unidos) ha determinado que los cerebros de los
astronautas que realizan misiones especiales se comprimen y se expanden durante el vuelo espacial. Los resultados
de la investigación fueron palpables: el volumen de la materia gris de los astronautas aumentó o disminuyó, y la
magnitud de esta alteración dependió de la duración del tiempo en el espacio. La gravedad empuja todos los fluidos
a los pies; el problema es que esto no sucede en el espacio. Esto tiene como consecuencia que haya más fluido
dirigido a la cabeza. También descubrieron que el volumen de materia gris en las regiones que controlan el
movimiento de las piernas aumentó. En otras regiones del cerebro observaron que disminuía el volumen de
materia gris, posiblemente debido a la redistribución del fluido cefalorraquídeo que recubre el sistema nervioso
central.

a) Los cerebros de los astronautas que realizan misiones espaciales se comprimen y se expanden durante el
vuelo espacial
b) La magnitud de la alteración en el volumen de la materia gris depende de la duración del tiempo en el
espacio
c) Se descubrió que el volumen de materia gris en las regiones que controlan el movimiento de las piernas
aumentó
d) En algunas regiones del cerebro disminuía el volumen de materia gris, debido a la redistribución del fluido
cefalorraquídeo

Respuesta correcta: a)

La idea principal o central es la idea conclusiva o que sintetiza todo lo que se dice en un texto. En este caso, la idea
principal es explícita, es decir, aparece dentro del texto: los cerebros de los astronautas que realizan misiones
especiales se comprimen y se expanden durante el vuelo espacial. Las demás ideas (secundarias) complementan dicha
idea central (que vendría a ser la tesis del grupo de científicos), a través de una serie de argumentos y aclaraciones
del porqué ocurre este hecho.

10)

Identifique la idea principal del texto:

Las fiestas populares de las comunidades de Chimborazo, tanto las de origen mestizo como las indígenas, reavivan
la música autóctona. Por ello, los músicos suelen ser los protagonistas de las mismas. Los pobladores afirman que,
durante la época festiva, los ritmos preferidos por el público son los más alegres: cachullapis, saltashpas, danzantes
y sanjuanitos. Las letras de estas canciones rememoran las prácticas cotidianas de la gente, como la siembra y la
cosecha, la crianza de animales, las mingas y los enamoramientos entre jóvenes.

Pero la música tradicional no solo es común durante las fiestas, también se usa para rituales. Las canciones que se
escuchan en las ceremonias se escogen de acuerdo con la temporada del año y la fiesta sagrada que se celebre, por
ello existen cantos al Sol, a la Luna, a la fertilidad de la tierra, al fuego, al viento, al agua y al aire. Las canciones se
acompañan solo con el sonido de un tambor hecho con piel de borrego, y mientras más personas las canten al
unísono, más apus (espíritus) llegarán al ritual.

a) Las fiestas populares de las comunidades de Chimborazo, tanto las de origen mestizo como las indígenas,
reavivan la música autóctona
b) Las canciones que se escuchan en las ceremonias se escogen de acuerdo con la temporada del año y la
fiesta sagrada que se celebre
c) Los músicos suelen ser los protagonistas de las festividades tradicionales en las comunidades de
Chimborazo
d) Los pobladores afirman que, durante las fiestas, los ritmos musicales preferidos son los más alegres

Respuesta correcta: a)

80
La idea principal o central es la idea conclusiva o que sintetiza todo lo que se dice en un texto. En este caso, la idea
principal es explícita, es decir, aparece dentro del texto: Las fiestas populares de las comunidades de Chimborazo,
tanto las de origen mestizo como las indígenas, reavivan la música autóctona. Las demás ideas (secundarias)
complementan dicha idea central (que además vendría a ser la tesis del autor), a través de una serie de argumentos
y aclaraciones del porqué ocurre este hecho.

11)

Identifique la idea central del poema:

Ya las gentes murmuran que soy tu enemiga


porque dicen que en verso doy al mundo mi yo.
Mienten, Julia de Burgos. Mienten, Julia de Burgos.
la que se alza en mis versos no es tu voz: es mi voz
Porque tú eres ropaje y la esencia soy yo;
Y el más profundo abismo se tiende entre las dos.
Tú eres fría muñeca de mentira social,
Y yo, viril destello de la humana verdad.
Tú, miel de cortesanas hipocresías, yo no;
que en todos mis poemas desnudo el corazón.
Tú eres como tu mundo, egoísta; yo no;
que en todo me lo juego a ser lo que soy yo.
Tú eres solo la grave señora señorona;
yo no, yo soy la vida, la fuerza, la mujer…

a) Disputa interior que sufre la voz narrativa y su álter ego


b) Conflicto que surge en la voz narrativa por los murmullos de la gente
c) Contienda que se genera entre la sociedad y la voz narrativa
d) Conflicto entre la mentira social y su verdad humana

Respuesta correcta: a)

La idea principal o central es la idea conclusiva o que sintetiza todas las ideas de un texto. En este caso, la idea principal
está implícita, es decir, hay que inferirla. Se trata de un poema de Julia de Burgos titulado “A Julia de Burgos”. Ella se
escribe a sí misma. El poema “expresa la batalla que existe en sí misma, consigo misma. Julia habla de ser su misma
enemiga, ya que el mundo a su alrededor es dueña de ella. Está ligada a las hipocresías y a su marido, pues le importa
mucho las opiniones de los demás. A la par, existe esta otra Julia que en sus versos deja hablar la verdad, no sucumbe
a la realidad a su alrededor. Ella se da a sí misma en su poesía para que los lectores sientan su corazón”. 29 En suma,
este poema expresa la “disputa interior que sufre la voz narrativa y su álter ego”.

12)

Identifique la idea central del poema

El padre del hijo pródigo

El polvo del camino me ha contaminado los ojos


he visto cómo el viento desbarata las ramas
algo me dice no eres el dueño de los que amas
y el alma se me llena de vacíos y despojos
sin embargo yo espero y refreno el enojo
al recordar un niño que saltaba en mi cama
tal vez no fui muy firme /pecado de quien ama/
o tal vez sea el destino que ni juzgo ni escojo
como un antiguo oasis me engaña un espejismo
la mentira que brinda al viajero el desierto

29Tomado de:
http://www.myteacherpages.com/webpages/mmalkerson/files/a%20julia%20de%20burgos%20analasis.pdf

81
en los ojos heridos por la espera y el llanto
una silueta antigua /la sombra de mí mismo/
perdido y encontrado vivo y nunca más muerto
es el hijo que vuelve y cubro con mi manto

a) Recuerdos que guarda el padre de su hijo


b) Amor incondicional de un padre
c) Retorno a la morada del padre
d) Poca firmeza del padre a la crianza del hijo

Respuesta correcta: b)

La idea principal o central es la idea conclusiva o que sintetiza todas las ideas de un texto. En este caso, la idea principal
está implícita, es decir, hay que inferirla. Se trata de un poema que expresa el amor incondicional y humano de un
padre a su hijo. El padre, refrenando su enojo, recuerda cómo su hijo se marchó y la forma en que lo educó para,
finalmente, advertir que, pese a ello, recibe con amor a su hijo que vuelve a su lecho. Ese amor paterno es lo que le
lleva al autor a escribir este texto de origen bíblico.

13)

Con base en el texto, identifique la inferencia adecuada

Cuando una persona escucha a otra que le habla, no solo reacciona a lo que se le está diciendo, sino también, a
otras características del habla, como el torno emocional y el género del hablante. Un reporte de la revista Currente
Biology proporciona las primeras evidencias de que los perros también diferencian y procesan esos diversos
componentes de la voz humana. “Aunque no podemos decir qué tanto o de qué manera los perros pueden
entender la información del discurso, es posible afirmar que reaccionan tanto a los registros verbales como a la
información relacionada con el hablante, y que esos componentes parecen ser procesados en diferentes áreas del
cerebro animal”, dice Victoria Ratcliffe, de la Universidad de Sussex. Estudios previos han demostrado que los
perros tienen sesgos hemisféricos -cerebro izquierdo contra el derecho- cuando procesan sonidos de la vocalización
de otros perros. Ratcliffe y su supervisor David Raby pronunciaron un discurso a cada lado del perro, para que
captaran los sonidos con cada uno de los oídos, al mismo tiempo y con la misma amplitud. “El aporte de cada oído
es transmitido principalmente al hemisferio opuesto del cerebro”, explica Ratcliffe.

a) Al pronunciar un discurso en los oídos de los perros, los científicos comprobaron que éstos escuchan muy
bien.
b) Los perros, por ser seres vivos, identifican las características de la voz, como cuando las personas hablan
entre sí
c) Los perros captan la voz de otros perros, porque son capaces de procesar los sonidos humanos
d) Los sesgos hemisféricos en los perros permiten procesar cierta información de los sonidos de otros caninos.

Respuesta correcta: b)

Inferir es deducir la idea central implícita en un texto. El texto citado nos habla acerca de cómo algunos científicos, a
partir de ciertos experimentos con perros, han probado que estos animales escuchan bien lo que se les dice y captan
y procesan las voces de otros perros. ¿Qué podemos inferir o DEDUCIR de ello? Quizá que los perros tienen un nivel
de inteligencia o, como consta en las opciones de respuesta, que “los perros, por ser seres vivos, identifican las
características de la voz, como cuando las personas hablan entre sí”.

14)

Identifique la oración que se infiere a partir del texto:

Estoy segura de que la mayoría de ustedes sabrá quién es Stephen Hawking: ese científico británico que se presenta
en silla de ruedas y con todo el aspecto de sufrir una total inmovilidad. A los 21 años, recién acabada su licenciatura
en Ciencias Físicas, se le diagnosticó esclerosis lateral amiotrófica. La enfermedad progresó dejándole postrado,
pero él siguió concibiendo más y más proyectos. Le movía el deseo de conocer qué pasaba en el universo, cómo
empezó la vida y cómo funcionaban las leyes básicas de la naturaleza. Hoy, con 66 años de edad, es uno de los

82
científicos más prestigiosos de la humanidad y, a pesar de su inmovilidad, con la ayuda de un sintetizador de voz,
da conferencias y conoce el universo mejor que nadie.

¿Y sabéis por qué pudo llegar hasta ahí? Porque su cerebro está intacto y lo ha utilizado a tope, contando con la
insustituible voluntad.

a) Las personas con esclerosis pueden comunicarse con sintetizador de voz


b) El estudio y la voluntad permiten al hombre apropiarse del conocimiento
c) El uso del cerebro permite realizar proyectos que sirvan a la tecnología
d) La edad no es un impedimento para conocer lo que sucede en el universo

Respuesta correcta: b)

Inferir es deducir la idea central implícita en un texto. El texto citado nos habla acerca de cómo, Stephen Howking,
pese a su enfermedad de esclerosis lateral amiotrófica, ha logrado conocer el universo mejor que nadie. Lo hace con
la ayuda de un sintetizador de voz y gracias a su deseo de conocimiento e insustituible voluntad. ¿Qué se podría
deducir del texto? Quizá, que ninguna enfermedad es impedimento para seguir adelante si nos apoyamos en los
avances tecnológicos, o quizá, como consta en las opciones de respuesta, que "el estudio y la voluntad permiten al
hombre apropiarse del conocimiento”.

15)

Identifique la consecuencia lógica:

Las personas que nacieron en Francia se comunican en francés con otras personas que también hablan esta lengua;
los luxemburgueses también emplean el idioma francés en Luxemburgo. La historia nos recuerda que el rey Felipe
II habló en francés ante la Asamblea Nacional; por lo tanto, el Rey Felipe II:

a) Nació en Francia
b) Quizás nació en Francia
c) Seguro no nació en Francia
d) Seguro nació en Luxemburgo

Respuesta correcta: b)

La consecuencia lógica es la conclusión que se deriva de un conjunto de premisas de un argumento deductivamente


válido. En el texto citado hay tres premisas:

• Premisa o proposición 1: Las personas que nacieron en Francia se comunican en francés con otras personas que
también hablan esta lengua.

• Premisa o proposición 2: Los luxemburgueses también emplean el idioma francés en Luxemburgo.

• Premisa o proposición 3: El rey Felipe II habló en francés ante la Asamblea Nacional.

¿Cuál sería la consecuencia lógica correcta?

Pues que el rey Felipe II QUIZÁ nació en Francia. ¿Por qué?

NO se podría decir que el Rey Felipe II (que habló francés ante una Asamblea Nacional) nació necesariamente en
Francia pues pudo haber nacido en Luxemburgo donde también se habla francés. Tampoco se puede decir que dicho
rey “seguro no nació en Francia”, pues pudo haber nacido allí dado que en ese país se habla ese idioma. Y tampoco
es correcto afirmar que “seguro nació en Luxemburgo” pues no solo en Luxemburgo se habla francés. Lo correcto es
inferir que el Rey Felipe II quizá nació en Francia.

16)

Identifique la consecuencia lógica:

83
Las personas que nacieron en Francia se comunican en francés con otras personas que también hablan esta lengua;
los suizos también emplean el idioma francés en Suiza. Un reconocido lingüista expuso en francés ante la Academia
de Ciencias. Por lo tanto, el lingüista:

a) Nació en Francia
b) Quizás nació en Suiza
c) Seguro no nació en Francia
d) Seguro nació en Suiza

Respuesta correcta: b)

La consecuencia lógica es la conclusión que se deriva de un conjunto de premisas de un argumento deductivamente


válido. En el texto citado hay tres premisas:

• Premisa o proposición 1: Las personas que nacieron en Francia se comunican en francés con otras personas que
también hablan esta lengua

• Premisa o proposición 2: Los suizos también emplean el idioma francés en Suiza

• Premisa o proposición 3: Un reconocido lingüista expuso en francés ante la Academia de Ciencias

¿Cuál sería la consecuencia lógica correcta?

Pues que el reconocido lingüista QUIZÁS nació en Suiza. ¿Por qué?

NO se podría decir que el lingüista (que habló francés ante la Academia de Ciencias) nació necesariamente en Francia
pues pudo haber nacido en Suiza donde también se habla francés. Tampoco se puede decir que dicho lingüista
“seguro no nació en Francia”, pues pudo haber nacido allí dado que en ese país se habla ese idioma. Y tampoco es
correcto afirmar que “seguro nació en Suiza” pues no solo en Suiza se habla francés. Lo correcto es inferir que el
lingüista quizá nació en Suiza.

17)

Identifique la consecuencia lógica:

Las personas que nacieron en Francia se comunican en francés con otras personas que también hablan esta lengua;
los tunecinos también emplean el idioma francés en Túnez. Un fotógrafo presentó su obra en francés para el
público de una galería. Por lo tanto, el fotógrafo:

a) Nació en Francia
b) Quizás nació en Túnez
c) Seguro no nació en Francia
d) Seguro nació en Túnez

Respuesta correcta: b)

La consecuencia lógica es la conclusión que se deriva de un conjunto de premisas de un argumento deductivamente


válido. En el texto citado hay tres premisas:

• Premisa o proposición 1: Las personas que nacieron en Francia se comunican en francés con otras personas que
también hablan esta lengua

• Premisa o proposición 2: Los tunecinos también emplean el idioma francés en Túnez

• Premisa o proposición 3: Un fotógrafo presentó su obra en francés para el público de una galería

¿Cuál sería la consecuencia lógica correcta?

Pues que el fotógrafo QUIZÁS nació en Túnez. ¿Por qué?

84
NO se podría decir que el fotógrafo (que presentó su obra en francés en una galería) nació necesariamente en Francia
pues pudo haber nacido en Túnez donde también se habla francés. Tampoco se puede decir que dicho lingüista
“seguro no nació en Francia”, pues pudo haber nacido allí dado que en ese país se habla ese idioma. Y tampoco es
correcto afirmar que “seguro nació en Túnez” pues no solo en Túnez se habla francés. Lo correcto es inferir que el
lingüista quizás nació en Túnez.

18)

Identifique la consecuencia lógica:

En un país se establecieron aranceles para algunos objetos importados. A pesar de los intentos de fomentar la
producción nacional, se continúa importando celulares de todo tipo, incluidos los de última tecnología. Por lo
tanto:

a) Probablemente los celulares importados pagan aranceles


b) Los aranceles se aplicarán para los celulares
c) Los aranceles no se aplicarán para los celulares
d) Quizá ese país reduzca los aranceles para los celulares

Respuesta correcta: a)

La consecuencia lógica es la conclusión que se deriva de un conjunto de premisas de un argumento deductivamente


válido. En el texto citado hay dos premisas:

• Premisa o proposición 1: En un país se establecieron aranceles para ALGUNOS objetos importados

• Premisa o proposición 2: Se continúa importando celulares de todo tipo, incluidos los de última tecnología.

Como no sabemos si los celulares que se siguen importando forman parte de aquellos objetos a los cuales se
establecieron aranceles (y que son ALGUNOS objetos, no TODOS), podríamos deducir que PROBABLEMENTE estos
pagarán aranceles. Es decir, la opción correcta es “probablemente los celulares importados pagan aranceles”.

No son correctas las demás opciones por lo siguiente:

• NO podemos decir necesariamente que “los aranceles se aplicarán para los celulares” pues algunos de ellos
podrían ser de origen nacional (aun cuando en su mayoría no sean consumidos); además, no sabemos si los
celulares importados forman parte de aquellos a los que se gravarán impuestos.

• Tampoco podemos decir que “los aranceles no se aplicarán para los celulares” o que se realizará una reducción
de dichos impuestos, pues es probable que los celulares importados sí estén contemplados dentro del listado de
objetos a los que se gravarán aranceles.

19)

Identifique la consecuencia lógica:

Al inicio del curso, el profesor de Álgebra explicó a sus estudiantes las condiciones para aprobar el módulo. Primero,
presentar puntualmente todos los trabajos académicos, y segundo, registrar una calificación superior a 7/10 en al
menos tres de cada cuatro evaluaciones escritas. Carlos obtuvo 8/10 en seis de las ocho evaluaciones escritas,
mientras que José obtuvo un puntaje igual o mayor a 7/10 en cinco de las ocho evaluaciones. Además, los dos
presentaron de forma puntual todos los trabajos académicos que solicitó el profesor. Por tanto:

1) José y Carlos reprobaron el módulo de álgebra


2) Ni José ni Carlos reprobaron el módulo de álgebra
3) José aprobó el módulo de álgebra; sin embargo, Carlos no lo consiguió
4) Carlos aprobó el módulo de álgebra; sin embargo, José no lo consiguió

85
Respuesta correcta: d)

El texto plantea dos condiciones para aprobar el módulo de Álgebra: a) registrar una calificación superior a 7/10 en 3
de cada 4 evaluaciones, y b) presentar puntualmente todos los trabajos académicos. Carlos y José cumplieron con la
segunda condición: presentaron puntualmente sus tareas.

Carlos obtuvo 8/10 en 6 de 8 evaluaciones, lo que equivale a 3 de 4 evaluaciones. Él cumplió con la primera condición
para aprobar.

José obtuvo 7/10 o más en 5 de 8 evaluaciones; es decir, no llegó a cumplir con una calificación mayor a 7 en 3 de
cada 4 evaluaciones.

Por tanto, Carlos aprobó el módulo de álgebra; sin embargo, José no lo consiguió.

20)

El genoma humano contiene la secuencia de ADN presente en 23 pares de cromosomas en el núcleo de cada célula
humana. El par 23 determina el sexo de la persona; cuando este par presenta dos cromosomas X, el sexo del
individuo es cromosómicamente hembra, y cuando presenta un cromosoma X y uno Y, el sexo es macho.

Con base a esta información, el genoma humano:

a) Incluye al menos un cromosoma X


b) Contiene exactamente 23 cromosomas
c) Incluye ADN con 23 cromosomas hembras y 23 machos
d) Contiene 46 ADN, incluido un par que determina el sexo

Respuesta correcta: a)

Al leer detenidamente el texto podemos afirmar que:

• El genoma humano NO “contiene exactamente 23 cromosomas”. El texto afirma que el genoma humano contiene
23 PARES de cromosomas (o 46 cromosomas en total).
• El genoma humano tampoco “incluye ADN con 23 cromosomas hembras y 23 machos”. El texto no dice cuántos
cromosomas hembras (XX) y cuantos machos (XY) conforman los 23 pares de cromosomas.
• El genoma humano tampoco “contiene 46 ADN, incluido un par que determina el sexo”; en realidad, contiene 46
cromosomas con un par (el 23) que determina el sexo.
• Lo que sí es correcto es que el genoma humano incluye al menos un cromosoma X.

21)

Si sabemos que en la familia Quiroga, Cristóbal es el único hijo del abuelo de Esteban y Cristina es la hija de
Cristóbal, ¿qué es Esteban para Cristina?

a) Hermano
b) Primo
c) Tío
d) Padre

Respuesta correcta: a)

La pregunta nos pide determinar la consecuencia lógica derivada de un conjunto de premisas:

• Premisa o proposición 1: Cristóbal es el único hijo del abuelo de Esteban.

• Premisa o proposición 2: Cristina es la hija de Cristóbal.

86
Si, según la premisa 1, el abuelo de Esteban tiene un único hijo que es Cristóbal, entonces éste resulta ser padre de
Esteban. Y si, según la premisa 2, Cristina es hija de Cristóbal, padre de Esteban, entonces éste resulta ser HERMANO
de ella.

22)

Si sabemos que en la familia Quiroga, Cristóbal es el único hijo del abuelo de Esteban, y Esteban tiene una
hermana llamada Cristina que es hija de Cristóbal, ¿qué es Cristóbal para Esteban?

a) Padre
b) Nieto
c) Abuelo
d) Hijo

Respuesta correcta: a)

La pregunta nos pide determinar la consecuencia lógica derivada de un conjunto de premisas:

• Premisa o proposición 1: Cristóbal es el único hijo del abuelo de Esteban.

• Premisa o proposición 2: Esteban tiene una hermana llamada Cristina.

• Premisa o proposición 3: Cristina es hija de Cristóbal.

Si, según la premisa 1, el abuelo de Esteban tiene un único hijo que es Cristóbal, entonces éste resulta ser padre de
Esteban, así como lo es de Cristina (según la premisa 3). Por ello, Cristina es hermana de Esteban.

23)

Cristóbal Quiroga es el único hijo del abuelo de Manuel. Cristóbal tiene una hija llamada Cristina. ¿Qué es Manuel
para Cristina?

a) Hermano
b) Primo
c) Tío
d) Padre

Respuesta correcta: a)

La pregunta nos pide determinar la consecuencia lógica derivada de un conjunto de premisas:

• Premisa o proposición 1: Cristóbal Quiroga es el único hijo del abuelo de Manuel.

• Premisa o proposición 2: Cristóbal tiene una hija llamada Cristina.

Si, según la premisa 1, el abuelo de Manuel tiene un único hijo que es Cristóbal, entonces éste resulta ser padre de
Manuel, así como lo es de Cristina (según la premisa 2). Por ello, Manuel es hermano de Cristina.

24)

Jorge Paladines es el único hermano del abuelo de Carlos. Jorge tiene un nieto llamado Daniel. ¿Qué es Carlos
para Daniel?

a) Hermano
b) Primo
c) Tío
d) Padre

87
Respuesta correcta: b)

La pregunta nos pide determinar la consecuencia lógica derivada de un conjunto de premisas:

• Premisa o proposición 1: Jorge Paladines es el único hermano del abuelo de Carlos.

• Premisa o proposición 2: Jorge tiene un nieto llamado Daniel.

Si, según la premisa 1, el abuelo de Carlos tiene un único hermano que es Jorge, entonces éste resulta ser tío abuelo
de Carlos. Y si, según la premisa 2, Jorge (tío abuelo de Carlos) tiene un nieto llamado Daniel, entonces éste resulta
ser primo de Carlos. Carlos, por tanto, es primo de Daniel.

25)

Humberto Cevallos es el único hijo del tío de Manuel. Humberto tiene una hija llamada Cristina. ¿Qué es Manuel
para Cristina?

a) Hermano
b) Primo
c) Tío
d) Padre

Respuesta correcta: c)

La pregunta nos pide determinar la consecuencia lógica derivada de un conjunto de premisas:

• Premisa o proposición 1: Humberto Cevallos es el único hijo del tío de Manuel.

• Premisa o proposición 2: Humberto tiene una hija llamada Cristina.

Si, según la premisa 1, el tío de Manuel tiene un único hijo que es Humberto Cevallos, entonces éste resulta ser primo
de Manuel. Y si, según la premisa 2, Humberto (primo de Manuel) tiene una hija llamada Cristina, entonces ésta
resulta ser sobrina -en segundo grado- de Manuel. Manuel, por tanto, es tío -en segundo grado- de Cristina.

88
Tema 5. Niveles o tipos de uso del lenguaje

Texto resumen

El lenguaje puede presentar, en sus manifestaciones concretas, diferentes variedades


producidas por causas sociales, es decir, debido a la forma en que incide nuestro hábitat, nuestra
profesión o nuestro entorno cultural. “No hablará del mismo modo una persona con una
titulación universitaria que otra con estudios básicos, o una persona procedente de un medio
rural respecto a otra que viva en una ciudad”. 30

A estas variedades se las conoce como niveles o tipos de uso del lenguaje. Según las condiciones
sociales, se pueden distinguir los siguientes: nivel vulgar, nivel popular o coloquial, nivel culto o
estándar, nivel literario y nivel científico.

Nivel vulgar

Corresponde a un lenguaje que rompe con el sistema lingüístico formal; usa jergas y malas
palabras, entendidas estas últimas como términos agresivos, intimidantes o insultantes. Una
frase que emplea lenguaje vulgar es, por ejemplo:

Ahurita me presentaron a una morrita que está bien chula. La neta que me dejó coqueado. Pero
vino ese cagón a decirme que me vaya a la verga y que ni me acerque a la morrita.

Otro ejemplo gráfico:

Imagen modificada de:


https://pl.depositphotos.com/60892485/stock-illustration-cartoon-people-set.html

30 Ministerio de Educación (2016). Lengua y Literatura. Segundo Año del BGU. Texto para el estudiante. Quito: UASB, página 211.

89
Nivel popular o coloquial

Se trata de un lenguaje propio de la tradición cultural de una región o de un país; no es culto,


pero tampoco es vulgar; se lo usa en contextos cotidianos y brota naturalmente. Una parte de
este lenguaje ya es reconocido por la Real Academia de la Lengua. Por ejemplo: Estás hablando
demasiado y ya te fuiste por las ramas.

Imagen modificada de:


https://pl.depositphotos.com/60892485/stock-illustration-cartoon-people-set.html

Nivel culto o estándar

Corresponde a un lenguaje empleado en contextos formales, respetando las normas de la Real


Academia de la Lengua Española. Conocerla requiere de varios años de práctica académica. Se
usa entre los profesionales o en contextos políticos y diplomáticos. Por ejemplo: El patrimonio
bibliográfico del Instituto Nacional de Patrimonio es extenso. Allí se encuentran obras incunables
que deberían preservarse con excelso cuidado. O este ejemplo gráfico:

Imagen modificada de:


https://pl.depositphotos.com/60892485/stock-illustration-cartoon-people-set.html

90
Nivel literario

Es un lenguaje propio de obras literarias sometidas a una estructura estética que busca deleite
en los lectores. Emplea figuras literarias. Por ejemplo:

Del salón en el ángulo oscuro,


de su dueño tal vez olvidada,
silenciosa y cubierta de polvo
veíase el arpa.31

Compartimos otro ejemplo: un fragmento de “Las seis cuerdas” de Federico García Lorca.32

Imagen modificada de:


https://pl.depositphotos.com/60892485/stock-illustration-cartoon-people-set.html

Nivel técnico

Corresponde a un lenguaje que incluye términos de dominio científico. Es un lenguaje


prácticamente restringido a los intercambios técnicos de ingenieros, químicos, abogados,
informáticos, biólogos, médicos, lingüistas y otros especialistas. Por ejemplo:

La Segunda Ley de Newton enuncia que la aceleración que toma un cuerpo es proporcional a la
Fuerza usada en moverlo, e inversamente proporcional a su masa, que lo va frenando. La
Segunda Ley de Newton sienta las bases de La Dinámica, que es la rama de la Física que estudia
el movimiento de los objetos y su respuesta a las fuerzas.

31 Extracto de la Rima VII de la obra “Rimas” de Gustavo Adolfo Bécquer. Tomado de:
http://www.badosa.com/bin/obra.pl?id=p110-13
32 Tomado de: https://www.ejemplos.co/20-ejemplos-de-poesia/#ixzz5NdeSK5XA

91
Imagen modificada de:
https://pl.depositphotos.com/60892485/stock-illustration-cartoon-people-set.html

Referencias bibliográficas

• Ministerio de Educación (2016). Lengua y Literatura. Segundo Año del BGU. Texto para el estudiante. Quito:
UASB.

• Carratalá Teruel. Niveles de uso de la lengua y registros del habla. Tomado de: http://www.contrastiva.it

Banco de preguntas y respuestas

1)

Identifique el nivel de uso del lenguaje en el siguiente chat:

Usuario: Buenos días señores, ¿hay ya respuesta sobre mi requerimiento del día de ayer? Es sobre una actualización
de mi computadora.

Atención al cliente: Sí, estimado cliente, respecto a los componentes externos le recomendamos reciclar algunos
elementos como el teclado (siempre que sea el expandido), la disquetera (si es de 3 1/2 y no es un PS/2 de IBM), y si
su máquina es PS/2 a color puede también conservar el monitor.

Usuario: ¿Y acerca de la tarjeta de sonido?

Atención al cliente: Es aconsejable que sea, como mínimo, compatible Sound Blaster 16 a nivel de registros. Si cuenta
con poco presupuesto puede elaborar usted mismo un DAC, que le servirá igual que una tarjeta de sonido.

Usuario: Muchas gracias por su ayuda.

92
a) Literario
b) Técnico
c) Coloquial
d) Vulgar

Respuesta correcta: b)

El lenguaje empleado en este ejemplo corresponde a un lenguaje técnico pues incluye jerga propia de una tecnología
(computadora) a fin de explicar su correcto funcionamiento. NO es un lenguaje literario pues no emplea figuras
literarias ni se somete a una estructura estética. Tampoco es un lenguaje coloquial pues, pese a que se trata de un
chat que se supone más “informal”, no incluye términos espontáneos que son propios de una conversación cotidiana.
Tampoco es un lenguaje vulgar pues no usa palabras ajenas a la RAE ni vulgarismos.

2)

Identifique el nivel del lenguaje empleado en este texto:

Hola Sebas.
Espero que tu viaje haya estado súper. Te extrañamos en la oficina, todos esperamos que termines
bien el curso. Te cuento que a tu regreso debes presentar un reporte de las actividades desarrolladas en Madrid.
Tu ñaño me pidió que te cuente que está pasando de película en la universidad; dice que, aunque vive sólo, se las
ha arreglado con las cosas del nuevo departamento.
Recuerda traer para nuestro proyecto un multímetro y cuatro
placas board para prototipos 160 x 125 mm.


Saludos,

Mónica

a) Culto
b) Vulgar
c) Coloquial
d) Técnico


Respuesta correcta: c)

El texto que se cita NO utiliza un lenguaje culto pues no emplea de forma estricta el léxico de la Real Academia de la
Lengua (en lugar de “excelente” emplea la palabra “super”, por ejemplo). Tampoco emplea un lenguaje técnico pues,
aunque al final incorpora léxico de la tecnología (“placas board para prototipos 160 x 125 mm”), éste no es dominante.
Y tampoco utiliza vulgarismos. El texto corresponde a una carta o correo electrónico de carácter amigable o familiar;
es decir, es un texto coloquial: incluye términos espontáneos que son propios de una conversación cotidiana.

3)

Identifique el enunciado que utiliza lenguaje coloquial:

a) Este es un locrito de queso casi tan bueno como el que preparan en tu casa, vas a querer doble yapa y
nosotros estaremos encantados de servirte un segundo plato.
b) Yo estaba así como “ufff” y no cachaba lo que me dijistes por el celu, y lo que pasa es que no tenía ni
minutos.
c) La meningitis es la inflamación del tejido delgado que rodea el cerebro y la médula espinal, llamada
meninge.
d) El cuadro de Mando Integral, desarrollado por los profesores Kaplan y Norton, constituye una herramienta
de gran utilidad en la gestión organizacional.

Respuesta correcta: a)

En efecto, la opción a) corresponde a una frase coloquial pues, sin irrespetar reglas gramaticales, incluye términos
espontáneos que son propios de una conversación cotidiana y, además, proporciona identidad: hace referencia a un
plato típico ecuatoriano. Las demás opciones no son coloquiales. La opción b) “Yo estaba así como “ufff” y no cachaba
lo que me dijistes por el celu, y lo que pasa es que no tenía ni minutos” incurre en errores gramaticales (dijistes).

93
4)

Identifique el nivel del lenguaje empleado en este texto:

Hago lo que me sale de las narices sin que nadie me joda. No me importa que ese bueno para nada me suelte que
soy fácil. Bien pendejo es el man.

a) Literario
b) Vulgar
c) Coloquial
d) Técnico


Respuesta correcta: b)

El texto que se cita NO utiliza un lenguaje literario pues no emplea figuras literarias ni se somete a una estructura
estética. NO es un ejemplo de lenguaje coloquial porque, si bien es espontáneo, usa muchos vulgarismos. Y NO se
trata de un texto con lenguaje técnico porque no incluye términos de dominio científico. El texto emplea un lenguaje
vulgar, con palabras grotescas.

5)

Identifique el nivel de uso del lenguaje en este diálogo:

Jorge: Buenos días señorita, estoy buscando una dirección. ¿Podría usted ayudarme?

Manuela: Con gusto. Dígame a qué destino desea usted llegar.

Jorge: Busco un restaurante llamado “El Paraíso de Francia”. Me indicaron que quedaba ubicado en esta calle.

Manuela: No exactamente. Aquel restaurante se ubica en una calle contigua. Usted debe caminar hasta ese
inmueble cuyo letrero es de color gris y luego debe virar a la derecha. No le será difícil divisar la rúbrica del
restaurante.

Jorge: Le quedo muy agradecido.

a) Literario
b) Culto o estándar
c) Coloquial
d) Técnico


Respuesta correcta: b)

En efecto, el diálogo emplea correctamente las normas gramaticales de la Real Academia de la Lengua, con palabras
que no son comúnmente empleadas en la vida cotidiana (inmueble en lugar de “edificio” o rúbrica en lugar de cartel).

6)

Identifique el enunciado que utiliza lenguaje técnico:

a) Gracias al llamado de atención de los expertos, se ha visto la necesidad de mejorar aún más los estilos en
este tipo de lenguaje
b) Cervantes dijo: “Señor, las tristezas no se hicieron para las bestias, sino para los hombres”
c) El núcleo celular es un orgánulo membranoso que se encuentra en el centro de las células eucariotas
d) Su lenguaje en sí es bello, artístico y elaborado

Respuesta correcta: c)

En efecto, la opción c) corresponde a una oración que utiliza un lenguaje técnico, con términos de dominio científico
y, específicamente, de la Biología.

94
7)

Identifique el nivel del lenguaje empleado en este texto:

¡Cambia esa cara! Ven, te invito a ver la tele en mi casa para divertirnos un poco. Capaz, así se mejora tu ánimo.

a) Literario
b) Vulgar
c) Coloquial
d) Técnico


Respuesta correcta: c)

El texto que se cita NO utiliza un lenguaje literario pues no emplea figuras literarias ni se somete a una estructura
estética. NO es un ejemplo de lenguaje vulgar pues no emplea palabras grotescas. Tampoco es un lenguaje técnico,
pues no aborda temas de dominio científico. Es un lenguaje coloquial, espontáneo, muy propio de contextos
cotidianos.

8)

Identifique el nivel del lenguaje empleado en este texto:

Volverán las oscuras golondrinas


en tu balcón sus nidos a colgar,
y otra vez con el ala a sus cristales
jugando llamarán.33

a) Literario
b) Vulgar
c) Coloquial
d) Técnico


Respuesta correcta: a)

El texto que se cita NO es un ejemplo de lenguaje vulgar pues no emplea palabras grotescas. Tampoco es un lenguaje
técnico, pues no aborda temas de dominio científico. Y NO es un lenguaje coloquial, propio de contextos cotidianos.
Es un lenguaje literario, poético, sometido a una estructura estética de la Rima.

9)

Identifique el nivel de uso del lenguaje en este diálogo:

Natasha: Buenas noches señor, deseo que en esta ocasión me sirva una sopa de legumbres, bien caliente.

Ricardo: Con todo gusto. ¿La desea con queso o sin queso?

Natasha: Prefiero que el queso lo coloque en un plato aparte, por favor.

Ricardo: Pierda cuidado. ¿Desea algo de beber?

Natasha: Sí por favor. Un vaso con jugo de naranja estaría perfecto.

Ricardo: ¿Al clima o helada?

Natasha: Al clima, gracias.

33 Fragmento de la Rima LIII de Gustavo Adolfo Bécquer. En: https://www.badosa.com/bin/obra.pl?id=p110-38

95
a) Literario
b) Culto o estándar
c) Coloquial
d) Técnico


Respuesta correcta: b)

En efecto, el diálogo emplea correctamente las normas gramaticales de la Real Academia de la Lengua, con palabras
que no son comúnmente empleadas en la vida cotidiana (“pierda cuidado”, por ejemplo).

10)

Identifique el nivel de uso del lenguaje en este chat:

Héctor: Hola mi querida amiga, ¿tienes un tiempo para chatear conmigo?

Camila: Hola Héctor, claro, dime.

Héctor: Ayer pasé por una librería y encontré un texto muy interesante sobre los vectores. Estoy seguro que allí
encontraremos las respuestas a nuestro dilema.

Camila: ¿En serio? ¿Acaso tiene un apartado especial sobre enfermedades parasitarias crónicas?

Héctor: Efectivamente, es más, de la esquistosomiasis se dice mucho y hay fotografías de gusanos muy parecidos a
los de la finca. Las personas con parasitosis posiblemente fueron contagiadas por estos gusanos.

a) Literario
b) Culto o estándar
c) Coloquial
d) Técnico


Respuesta correcta: d)

El lenguaje empleado en este ejemplo corresponde a un lenguaje técnico pues incluye jerga propia de la medicina.
NO es un lenguaje literario pues no emplea figuras literarias ni se somete a una estructura estética. Tampoco es un
lenguaje coloquial pues, pese a que se trata de un chat que se supone más “informal”, no incluye términos
espontáneos que son propios de una conversación cotidiana. Tampoco es un lenguaje vulgar pues no usa palabras
ajenas a la RAE ni vulgarismos.

96
Tema 6. La paráfrasis

Texto resumen

El Diccionario de la Lengua Española indica que una paráfrasis es una “frase que, imitando en su
estructura otra conocida, se formula con palabras diferentes”34. Las paráfrasis deben ser
veraces, manteniendo las ideas originales del autor. La persona que parafrasea no puede
inventar hechos u opiniones personales que no se encuentren en el texto original.

Existen dos tipos de paráfrasis: mecánica y constructiva. La paráfrasis mecánica es la sustitución


simple de expresiones que aparecen en un texto por sinónimos y con cambios sintácticos
mínimos. La paráfrasis constructiva es la reelaboración del enunciado, dando origen a otro con
palabras o características que pueden ser muy distintas, pero que conservan el mismo
significado.

Por ejemplo, se puede suponer que un texto original dice así: La casa ya estaba desocupada.
Dos hombres esperaban a Pedro Páramo, quién se unió a ellos para seguir su camino.

Una paráfrasis mecánica podría ser: El hogar ya estaba vacío. Varios hombres aguardaban por
Pedro Páramo, que se reunió junto a ellos para continuar la travesía.

Una paráfrasis constructiva podría ser: Ya el edificio estaba vacío. Pedro Páramo se juntó con
los hombres que lo estaban esperando para emprender el viaje.35

Emplea sinónimos y cambios


Mecánica sintáctivos mínimos
Paráfrasis
Frase que imita a otra con
palabras diferentes
Modifica la estructura
Constructiva completa de la frase sin
modificar su significado

34 “Paráfrasis”. Diccionario de la Lengua Española. En: http://dle.rae.es/?id=Rq6dJ6v


35 Ejemplos tomados de: Cajal, Alberto. Paráfrasis: tipos, características y ejemplos. En: https://www.lifeder.com/parafrasis/

97
Referencias bibliográficas

• Cajal, Alberto. Paráfrasis: tipos, características y ejemplos. En: https://www.lifeder.com/parafrasis/

• Diccionario de la Lengua Española. En: http://dle.rae.es/

Banco de preguntas y respuestas

1)

El Diccionario de la Real Academia Española indica que una paráfrasis es una “frase que, imitando en su estructura
otra conocida, se formula con palabras diferentes”. Con base en esta definición, identifique la paráfrasis correcta
de la cita:

La música es el corazón de la vida (Franz Liszt)

a) La música despierta el corazón


b) El corazón necesita de la música
c) La música es el centro de la vida
d) El corazón late al ritmo de la música

Respuesta correcta: c)

En efecto, la opción c) se trata de una paráfrasis mecánica que reemplaza la palabra “corazón” por “centro” (corazón
es una metáfora de “centro”). Lo que Franz Liszt quiere decirnos con esta frase es que la vida no tiene sentido sin la
música. La música es el centro, el eje, el “corazón” de la vida.

2)

Identifique la paráfrasis del siguiente enunciado:

Muy poco adelantarás si vuelves la cara atrás

a) Mirar atrás implica recordar para no volver a equivocarse


b) Es conveniente concentrarse en el pasado para mejorar
c) No es posible tener un buen futuro si se vive pensando en hechos pasados
d) Mirar adelante implica avanzar y hacer planes para el futuro

Respuesta correcta: c)

En efecto, la opción c) se trata de una paráfrasis constructiva. ¿Qué quiere decir “muy poco adelantarás si vuelves la
cara atrás”? Pues, básicamente, que NO es posible tener un buen futuro si se vive pensando en hechos pasados. Esta
paráfrasis está respetando el significado de la frase original, pero con palabras completamente distintas.

3)

Identifique la paráfrasis adecuada de la siguiente frase:

La razón es una antorcha que alumbra todo ser espiritual, que da colorido e ilumina a las operaciones del alma
(Eugenio Espejo).


98
a) La razón y el alma son lo mismo
b) Sin el alma no hay guía posible
c) La razón guía el alma
d) El espíritu guía al alma

Respuesta correcta: c)

La opción c) se trata de una paráfrasis constructiva. ¿Qué nos quiere decir Eugenio Espejo con su frase La razón es
una antorcha que alumbra todo ser espiritual, que da colorido e ilumina a las operaciones del alma? NO nos dice que
“la razón y el alma son lo mismo”, ni que “sin el alma no hay guía posible”, ni menos aún que “el espíritu guía el alma”.
Eugenio Espejo dice que la razón ilumina el alma. E iluminar es sinónimo de "guiar". Por tanto, la razón guía el alma.
Esta paráfrasis está respetando el significado de la frase original, pero con palabras completamente distintas.

4)

Identifique la paráfrasis adecuada:

Si los perros ladran es señal de que avanzamos

a) El comentario de la gente revela que avanzamos


b) Los ladridos de los perros son escandalosos
c) Los comentarios de la gente los convierte en perros
d) Los comentarios son señal de que hay perros cerca

Respuesta correcta: a)

En efecto, la opción a) se trata de una paráfrasis constructiva. Cuando se dice “los perros ladran” se afirma que la
gente o la sociedad comenta o critica algo. La frase “Si los perros ladran es señal de que avanzamos” alude a lo
siguiente: los comentarios de las personas son signo de que avanzamos o mejoramos. Esta paráfrasis está respetando
el significado de la frase original, pero con palabras completamente distintas.

5)

Identifique la paráfrasis del enunciado:

Hay quien cruza el bosque y únicamente ve leña para el fuego

a) La leña de los árboles es lo más visible de un bosque


b) Existen personas para las cuales la vida solo es un recurso
c) Quien va a un bosque, sabe qué madera puede utilizar
d) El fuego requiere ser avivado con leña de los árboles

Respuesta correcta: b)

La opción b) corresponde a una paráfrasis constructiva. Cuando se dice “hay quien cruza el bosque y únicamente ve
leña para el fuego” se afirma que hay gente que mira su entorno sólo como un recurso para poder extraer algo (como
la leña para hacer fuego).

6)

Identifique la paráfrasis del enunciado

Quien ríe al último ríe mejor

a) La venganza provoca mucha gracia cuando consigue sus objetivos


b) Quien sabe reír nunca se ríe después de concluido un chiste
c) Quien no sabe de humor siempre arruina el remate del chiste
d) Espera pacientemente el final, pues la situación puede sufrir cambios

99
Respuesta correcta: d)

En efecto, esta opción se trata de una paráfrasis constructiva. Cuando se dice “quien ríe al último ríe mejor” se alude
a lo siguiente: quien hace las cosas sin adelantarse a sus resultados, es decir, sin precipitarse, podrá evaluarlas de
mejor manera hasta lograr un objetivo que sea regocijante. Por ejemplo: Jorge debe participar en un concurso de
Matemática con contrincantes de un colegio prestigioso. Tiene tiempo de prepararse, pero le dice a su maestro: “no
creo que lograré un buen puntaje si mis competidores son chicos con mejores conocimientos que yo”. Su profesor le
contesta: “no te adelantes a los hechos. Tienes tiempo para prepararte. Recuerda que quien ríe al último ríe mejor”.

7)

Con base en el enunciado, identifique la frase que reproduzca la misma idea en otras palabras:

El verdadero viaje de descubrimiento no consiste en buscar nuevas tierras, sino en ver con nuevos ojos (Marcel
Proust)

a) La búsqueda de nuevos lugares otorga nuevas formas de ver al observador


b) La persona que emprende nuevos viajes obtiene mejores descubrimientos
c) El descubrimiento le sucede a quien adquiere una nueva manera de mirar su realidad
d) El buen espectador siempre está capacitado para generar hallazgos en distintos lugares

Respuesta correcta: c)

En efecto, la opción c) corresponde a una paráfrasis constructiva. Cuando se dice “el verdadero viaje de
descubrimiento no consiste en buscar nuevas tierras, sino en ver con nuevos ojos”, se alude a lo siguiente: no es
necesario viajar a otro sitio para descubrir algo nuevo. En nuestro propio lugar de vida, en nuestro entorno, podemos
encontrarnos con un montón de cosas nuevas siempre y cuando lo miremos con una óptica diferente. Ese es un
verdadero viaje de descubrimiento. Entonces: El descubrimiento le sucede a quien adquiere una nueva manera de
mirar su realidad.

Nota: La opción correcta podría aparecer formulada del siguiente modo: El encuentro con lo nuevo le ocurre a quien
mira su entorno con otra óptica.

8)

Con base en el enunciado, identifique la frase que reproduzca la misma idea en otras palabras:

“Tres condiciones se requieren para llegar a ser felices: ser terco, ser egoísta y gozar de buena salud; pero bien
entendido que si os falta la primera condición, todo está perdido” (Gustave Flaubert).

a) El obstáculo tiene como característica ser egoísta y muy saludable


b) Una persona benevolente jamás llegará a ser muy feliz
c) El egoísmo y la terquedad son el camino a la felicidad
d) Un egocéntrico necesita evidenciar siempre un estado muy saludable

Respuesta correcta: b)

En efecto, la opción b) corresponde a una paráfrasis constructiva. Cuando se dice que “tres condiciones se requieren
para llegar a ser felices: ser terco, ser egoísta y gozar de buena salud; pero bien entendido que, si os falta la primera
condición, todo está perdido” estamos aludiendo a que las personas que no cumplen esas condiciones (terquedad,
egoísmo y buena salud) y sobretodo, aquellas que no son tercas, es decir, las benevolentes, tolerantes o
comprensivas, no llegarán a ser felices. Por eso, la frase citada de Gustave Flaubert significa que “una persona
benevolente jamás llegará a ser muy feliz”.

100
9)

Con base en el enunciado, identifique su paráfrasis:

“Confiar en ti mismo no garantiza el éxito, pero no hacerlo garantiza el fracaso” (Albert Bandura)

a) La baja autoestima es garantía del fracaso


b) El éxito es posible cuando desconfías del resto
c) La autoestima es necesaria pero no suficiente para llegar a nuestros objetivos
d) Una persona que no confía en sí misma siempre fracasará

Respuesta correcta: c)

En efecto, la opción c) corresponde a una paráfrasis constructiva. Cuando se dice que “confiar en ti mismo no
garantiza el éxito, pero no hacerlo garantiza el fracaso”, estamos aludiendo a que la autoestima es necesaria pero no
es suficiente para alcanzar nuestras metas.

10)

Con base en el enunciado, identifique su paráfrasis:

“A través de los otros nos convertimos en nosotros mismos” (Lev Vygotski)

a) Nuestra identidad individual se adquiere cuando nos relacionamos con otras personas
b) Gracias al apoyo de nuestros seres queridos podemos seguir adelante
c) Cuando criticamos a alguien nos criticamos a nosotros mismos
d) La sociedad influye en nuestras decisiones personales

Respuesta correcta: a)

En efecto, la opción a) corresponde a una paráfrasis constructiva. Cuando se dice que “a través de los otros nos
convertimos en nosotros mismos”, estamos aludiendo al concepto de identidad individual. Esa identidad no es posible
de construirla si no nos relacionamos con otras personas.

101
Tema 7. Sinónimos, antónimos y homónimos

Texto resumen

Sinónimos

Un sinónimo es una palabra que tiene un significado total o parcialmente idéntico a otra. Se lo
usa para no repetir palabras y, de esta manera, enriquecer el contenido de un texto. Por
ejemplo:

Estuve penando en escoger una carrera universitaria que no me demande demasiado tiempo
dados mis horarios de trabajo; sin embargo, un amigo me está convenciendo de que debo
seleccionar una opción acorde a mis aptitudes y pasiones aun cuando los esfuerzos resulten
intensos.

Las palabras subrayadas, escoger y seleccionar, significan lo mismo.

Antónimos

Un antónimo es una palabra que se opone a otra, es decir, que significa lo contario. Por ejemplo:

En esta vida, decía mi abuela feminista, “no está mal desear los placeres sexuales. La
concupiscencia no debe ser un privilegio masculino”. Así lo decía, hasta que un primo de ella,
sacerdote jesuita, le dio un sermón en público que terminó en una riña de palabras que no
olvidaré jamás. Este sacerdote quería convencerla de que las mujeres deben guardar continencia
hasta su matrimonio.

Concupiscencia y continencia son antónimos.

A continuación, proponemos un listado de palabras con sus respectivos significados, sinónimos


y antónimos. Las palabras seleccionadas suelen incluirse en las preguntas de razonamiento
verbal de los exámenes de acceso a la Universidad.

N° Palabra Significado Sinónimo Antónimo

Repulsión
1 Abominación Rechazo y condena enérgica de algo. Atracción
Animadversión

Amnistía
Acto de perdonar o absolver las ofensas o Castigo
2 Absolución Perdón
pecados de una persona Pena
Indulto

102
N° Palabra Significado Sinónimo Antónimo

Independiente
3 Absoluto Independiente, con pleno poder Dependiente
Único

Autoritario Condescendiente
Despótico Comprensivo
4 Absoluto De genio fuerte e inmoderado
Dominante Flexible
Arbitrario Tolerante

Abstracto
Concreto
Difícil de entender o que está oculto y Ambiguo
5 Abstruso Evidente
profundo Oculto
Obvio
Profundo

Humillación
Exaltación
6 Abyección Acción o efecto de degradar Degradación
Alabanza
Bajeza

Azar
Certeza
7 Acaso Suceso imprevisto o fortuito Casualidad
Certidumbre
Destino

Aceptar
Admitir Rechazar
8 Acceder Conceder a alguien lo que necesita
Conceder Rehusar
Consentir

Curar
Vulnerar
9 Acecinar Salar o ahumar la carne, conservar, curar Salar
Dañar
Conservar

Enfadar
Calmar
Fastidiar
Tranquilizar
10 Acosar Perseguir con persistencia y empeño Importunar
Apaciguar
Molestar
Serenar
Vulnerar

Artificial Auténtico
11 Afectado No espontáneo, poco natural
Forzado Natural

Azar
Suerte o azar de que depende el resultado
12 Albur Casualidad Predestinación
de un proyecto o un asunto
Contingencia

Desleal Sincero
Adjetivo que califica a una persona que es
13 Alevoso Pérfido Fiel
desleal
Traidor Noble

103
N° Palabra Significado Sinónimo Antónimo

Persona que vive en lugar solitario,


Solitario
14 Anacoreta entregada enteramente a la contemplación Sociable
Ermitaño
y a la penitencia

Concernir
Desentenderse
15 Atañer Acción de incumbir Pertenecer
Inhibirse
Incumbir

Observar
Desatender
Divisar
16 Atisbar Acción de mirar de forma recatada Descuidar
Avizorar
Desconocer
Atender

Predicción
Señal o anuncio que indica que algo va a Profecía
17 Augurio Equivocación
ocurrir Pronóstico
Presagio

Recto
Adjetivo que califica a alguien que está Torcido
18 Avieso Derecho
torcida o fuera de la norma Maquiavélico
Bueno

Sospecha
Sensación o sospecha de que algo va a Conjetura Confirmación
19 Barrunto
suceder Presunción Comprobación
Suposición

Fisgón Discreto
Persona que acostumbra a curiosear y a
20 Buscavidas Husmeador Prudente
entrometerse en la vida de los demás
Mirón Cauto

Misterioso Claro
Que es misterioso o que tiene un sentido
21 Cabalístico Secreto Abierto
enigmático.
Oculto Evidente

Índole, naturaleza o condición de una Índole


22 Calaña persona o de una cosa (generalmente Naturaleza -
negativa) Clase

Aprobar
Acción de aplaudir o aprobar a alguien o a Aclamar Condenar
23 Canonizar
una cosa Enaltecer Denigrar
Glorificar

Claudicar
Resistir
Transigir
24 Capitular Acción de rendirse Afrontar
Someterse
Desafiar
Rendirse

104
N° Palabra Significado Sinónimo Antónimo

Casta
Raza
25 Cepa Tronco u origen de una familia o linaje -
Raíz
Sangre

Tacañería
Modo de ser o de comportarse de las Mezquindad Generosidad
26 Cicatería
personas tacañas Avaricia Desprendimiento
Miseria

Contener
Refrenar Dejar
27 Coercer Impedir a una persona que haga una cosa
Impedir Libertar
Retener

Cabal
Adjetivo que describe algo que es integral Parcial
28 Completo Total
o cabal Incompleto
Integral

Sustituir
Mutar
29 Conmutar Cambiar una cosa por otra Fijar
Cambiar
Permutar

Adquirir compromisos, obligaciones o Adquirir Otorgar


30 Contraer
deudas Obtener Ceder

Ayudar
Coadyuvar o ayudar a la consecución de Auxiliar Eximir
31 Contribuir
una cosa Asistir Inhibirse
Favorecer

Injuria
Desagravio
Oprobio
Oprobio, injuria u ofensa dicha a una Exaltación
32 Contumelia Ofensa
persona en su cara. Halago
Agravio
Lisonja
Vilipendio

Normal Inusual
Que resulta o se establece en virtud de
33 Convencional Usual Infrecuente
hábitos cotidianos
Habitual Raro

Charla
Acción o resultado de hablar dos o más Plática
34 Conversación Silencio
personas Diálogo
Conversa

Disoluto
Moderado
Que está inclinado al desenfreno, al vicio, Libertino
35 Crapuloso Moral
al libertinaje o la borrachera Vicioso
Virtuoso
Depravado

105
N° Palabra Significado Sinónimo Antónimo

Juicio
Irreflexión
Discernimiento
36 Criterio Capacidad de juzgar o discernir Insensatez
Razonamiento
Irracionalidad
Cordura

Generoso Tacaño
Desprendido Avaro
37 Dadivoso Que es desinteresado o generoso
Obsequioso Miserable
Caritativo Mezquino

Traición
Abandono desleal de una causa que se Deslealtad Lealtad
38 Defección
defendía Infidelidad Fidelidad
Perfidia

Ingerir
Devolver
Masticar
39 Deglutir Acción de masticar o ingerir alimentos Echar
Comer
Vomitar
Engullir

Conversar
Dialogar
40 Departir Acción de conversar o hablar con alguien Silenciar
Parlamentar
Conferenciar

Desamparado
Cuidado
Apartado
41 Desolado Lugar que está vacío o sin vida Vigente
Desértico
Vivo
Olvidado

Esbozar
Hacer un esbozo de una idea o de un Trazar
42 Diseñar -
proyecto Delinear
Bosquejar

Crapuloso Moderado
Que está inclinado al desenfreno, al vicio,
43 Disoluto Vicioso Moral
al libertinaje o la borrachera
Libertino Virtuoso

Eximir
44 Dispensar Acción de justificar alguna acción Perdonar Culpar
Justificar

Loa
Reprobación
Alabanza
45 Ditirambo Alabanza o elogio exagerado Censura
Elogio
Crítica
Lisonja

Someter Revelar
46 Domeñar Someterse a algo o a alguien Avasallar Desobedecer
Rendir Resistir

106
N° Palabra Significado Sinónimo Antónimo

Éxtasis
Enajenación Desencanto
47 Embeleso Estado de disfrute o de placer intenso
Arrebato Desilusión
Maravilla

Sucio Limpiado
Engrasado Aseado
48 Emporcado Persona o cosa que se encuentra sucia
Percudido Lavado
Enlodado Restregado

Vislumbrar
Ignorar
49 Entrever Empezar a ver algo de manera confusa Divisar
Desconocer
Percibir

Escuálido Musculoso
50 Esmirriado Que está flaco, extenuado o consumido Extenuado Fornido
Consumido Vigoroso

Producir
51 Fabricar Producir una cosa por medios mecánicos Elaborar Consumir
Manufacturar

Enjuto
Débil Robusto
52 Feble Que está débil o enfermizo
Enfermizo Fornido
Flaco

Persona que tiene como oficio o profesión Barbero


53 Fígaro -
el cuidado de la barba y el cabello Peluquero

Aspecto que resulta del conjunto de Semblante


54 Fisonomía rasgos particulares del rostro de una Expresión -
persona Apariencia

Acomodaticio
Que no es riguroso y de adapta fácilmente Inflexible
55 Flexible Adaptable
a los cambios Rígido
Complaciente

Elegancia
Gracia que tiene una persona en su Donosura Desgarbo
56 Garbo
manera de actuar Distinción Insipidez
Gracia

Carga
Carga impuesta sobre un capital Exención
57 Gravamen Impuesto
monetario Privilegio
Tributo

107
N° Palabra Significado Sinónimo Antónimo

Acumular
Separar
Reunir y apretar en un sitio a un grupo de Aglomerar
58 Hacinar Aislar
personas Aglutinar
Esparcir
Apretujar

Erizado
Que está cubierto de pelo duro o de Espinoso Fino
59 Hirsuto
espinas Áspero Terso
Tieso

Huraño
Arisco Simpático
60 Hosco Que es antipático e intratable
Odioso Agradable
Antipático

Agudeza
Facultad de descubrir y manifestar lo Seriedad
61 Humor Ingenio
cómico y divertido Mesura
Gracia

Indemne Herido
62 Incólume Persona que no ha sido dañada o lesionada Ileso Dañado
Salvo Lesionado

Invasión Expulsión
Entrada violenta a un sitio. Penetración de
63 Incursión Irrupción Salida
un grupo a un territorio ajeno.
Penetración Huida

Pistas
Sucesos o cosas que permiten conocer o Vestigios Ausencias
64 Indicios
deducir la existencia de otras Signos Olvidos
Huellas

Pobreza Abundancia
Estrechez Riqueza
65 Indigencia Falta de medios para vivir
Miseria Exceso
Inopia Holgura

Apático
Desidioso Dinámico
66 Indolente Que no se conmueve o afecta
Insensible Entusiasta
Pelmazo

Ineptitud
Aptitud
67 Inepcia Inutilidad Incapacidad
Capacidad
Inutilidad

Familiaridad
Amistad y confianza profundas entre Animadversión
68 Intimidad Camaradería
personas Enemistad
Compañerismo

108
N° Palabra Significado Sinónimo Antónimo

Extraño
Persona que se ha introducido sin derecho Ajeno
69 Intruso Familiar
a un lugar ajeno Colado
Entrometido

Insólito
Usual
Desusado
70 Inusitado Que no es habitual o resulta extraño Habitual
Anticuado
Ordinario
Extraordinario

Contradecir Acoger
Oponerse o combatir por algo que se Refutar Aprobar
71 Impugnar
considera injusto Objetar Favorecer
Combatir Propugnar

Lamentación
Satisfacción
Lamentación o muestra exagerada de Sollozo
72 Jeremiada Contento
dolor Queja
Alegría
Lamento

Pleito
Acuerdo
Conocimiento de una causa o un pleito Caso
73 Juicio Convenio
ante un tribunal o un juez Litigio
Pacto
Querella

Hibernación Viveza
Sueño Despabilamiento
74 Letargo Sueño profundo y prolongado
Sopor Dinamismo
Modorra Agilidad

Diminuto Enorme
75 Liliputiense Persona que es muy pequeña Pigmeo Gigantesco
Minúsculo Crecido

Destruir Construir
76 Liquidar Poner término a algo o a alguien Exterminar Empezar
Aniquilar Comenzar

Resplandeciente
Apagado
Que emite rayos luminosos por Luminoso
77 Luminiscente Lúgubre
luminiscencia Fosforescente
Oscuro
Fluorescente

Dócil
Salvaje
Obediente
78 Manso Que es sosegado o tranquilo Arisco
Sumiso
Cerril
Doméstico

Apatía Vigor
79 Marasmo Estado de pérdida de vigor Desánimo Energía
Debilidad Fuerza

109
N° Palabra Significado Sinónimo Antónimo

Temeroso Valiente
Asustadizo Audaz
80 Medroso Que es propenso a tener miedo o recelo
Timorato Bizarro
Receloso Atrevido

Comodidad Sacrificio
81 Molicie Modo de vivir demasiado cómodo
Deleite Incomodidad

Insignificancia Importancia
Chiquillada Calidad
82 Nadería Cosa de poco valor e importancia
Pamplina Trascendencia
Chuchería Valor

Ignorante Ilustrado
83 Nesciente Que no sabe Iletrado Instruido
Incompetente Conocedor

Célebre Ignorado
Que es bastante conocido, célebre o
84 Nombrado Famoso Olvidado
famoso
Mentado Omitido

Antiguo
Actual
Arcaico
85 Obsoleto Que es o está anticuado o caído en desuso Moderno
Añejo
Contemporáneo
Vetusto

Obstruir
Abrir
Cerrar
86 Ocluir Cerrar un orificio o un conducto Destapar
Atascar
Desatorar
Taponar

Recóndito
Descubierto
Furtivo
87 Oculto Que no se ve o está escondido Expuesto
Clandestino
Desenmascarado
Subrepticio

Resguardarse
Exponerse
Protegerse
88 Parapetarse Resguardarse de algo o de alguien Arriesgarse
Cubrirse
Aventurarse
Ampararse

Bienes
Bienes que le pertenecen a un grupo por
89 Patrimonio Propiedad -
herencia histórica
Pertenencia

Apoyo
Apoyo para colocar encima una figura u Pedestal
90 Peana -
otra cosa Fundamento
Basa

110
N° Palabra Significado Sinónimo Antónimo

Comprender
Ignorar
Comprender una cosa en todos los Entender
91 Penetrar Desconocer
detalles Intuir
Inadvertir
Asimilar

Agudeza
Capacidad de entender con rapidez cosas Ingenio Desatino
92 Perspicacia
que pasan inadvertidas a otro Viveza Desacierto
Tino

Actitud
Postura
93 Pose Actitud afectada de una persona -
Gesto
Ademán

Beneficio
Ventaja
94 Prebenda Beneficio del que dispone una persona Pérdida
Provecho
Lucro

Predicción
Señal o anuncio que indica que algo va a Profecía
95 Presagio Equivocación
ocurrir Pronóstico
Augurio

Perverso
Bondadoso
Malo
96 Protervo Que es malo o perverso Caritativo
Avieso
Benévolo
Retorcido

Ángel caracterizado por la plenitud de Serafín


97 Querubín ciencia con que contempla la belleza Ángel Demonio
divina Angelito

Alegrar
Aburrir
98 Refocilar Divertir o alegrar con groserías Entretener
Entristecer
Divertir

Contentar Entristecer
99 Regocijar Causar alegría o placer a una persona Alegrar Aburrir
Satisfacer Llorar

Notoriedad
Desprestigio
Prestigio
100 Reputación Prestigio que tiene una persona Descrédito
Fama
Deshonor
Renombre

Reservado
Afectuoso
Parco
101 Reticente Que muestra desconfianza Afable
Circunspecto
Evasivo

111
N° Palabra Significado Sinónimo Antónimo

Restitución
Proceso de restitución de una cosa a su Reembolso
102 Reversión -
estado inicial Devolución
Restablecimiento

Levante
Llegada
103 Saliente Que sobresale de forma natural Naciente
Retorno
Orto

Encono Afecto
Rencor Perdón
104 Saña Intención rencorosa y cruel
Inquina Suavidad
Tirria Dulzura

Partidario
Que sigue la opinión o doctrina de otra Seguidor Adversario
105 Secuaz
persona Adepto Enemigo
Fanático

Bilateral Unidireccional
106 Sinalagmático Acuerdo bilateral o recíproco
Recíproco Asimétrico

Situado
Sacado
Fundado
107 Sito Que está situado en un lugar determinado Quitado
Asentado
Retirado
Localizado

Roñoso
Generoso
Tacaño
108 Sórdido Que se comporta con tacañería Desprendido
Avaro
Magnánimo
Mísero

Sumergir
Hundir o meter debajo de la tierra o del Hundir
109 Sumir Emerger
agua Ahondar
Abismar

Anuencia
Actitud de respeto y comprensión hacia Intolerancia
110 Tolerancia Respeto
las ideas y acciones ajenas Irrespeto
Consideración

Maquinar Aclarar
Preparar un plan o un asunto con mala Urdir Esclarecer
111 Tramar
intención Conspirar Desenmarañar
Intrigar Descubrir

Tumba
112 Túmulo Sepultura que sobresale del suelo -
Catafalco

112
N° Palabra Significado Sinónimo Antónimo

Predicción
Señal o anuncio que indica que algo va a Augurio
113 Vaticinio Equivocación
ocurrir Pronóstico
Presagio

Vendible
114 Venal Que se deja sobornar con dádivas Sobornable Honesto
Deshonesto

Rígido
Flexible
Inflexible
115 Yerto Que está rígido o inmóvil Quebrantable
Duro
Frágil
Rudo

Soez Educado
Que se comporta de manera tosca y Insolente Respetuoso
116 Zafio
grosera Patán Atento
Vulgar Comedido

Torpe Afable
Tosco Distinguido
117 Zamarro Hombre tosco, rústico y lerdo
Zopenco Galante
Zoquete Educado

Homónimos

Las palabras homónimas son aquellas cuya pronunciación es igual o similar, pero difieren en su
significado. Dentro de estas, se distinguen las palabras homófonas y las homógrafas o
polisémicas.

Las palabras homófonas suenan igual, pero se escriben de un modo diferente y tienen
significados distintos. Por ejemplo, abrazar y abrasar son palabras homófonas:

• Abrazar es un acto realmente hermoso. Estrecharse entre los brazos en señal de cariño lo
haces con quienes amas de verdad.

• Vamos a abrasar esa carne que está bastante jugosa y suave. Primero debemos untarla sal
en grano para que su sabor quede exquisito.

A continuación, proponemos un listado de palabras homófonas:

113
Palabras Homófonas

Palabra 1 Significado Palabra 2 Significado

1 Abollado Hundido por un golpe Aboyado Finca con bueyes

2 Abrazarse Estrecharse con los brazos Abrasarse Quemarse

3 Acerbo Áspero o duro Acervo Colección de cosas

4 Agito Conjugación del verbo agitar Ajito Ajo pequeño

5 Alagar Llenar de lagos Halagar Mostrar afecto

6 Aprender Adquirir conocimientos Aprehender Asir

7 Arrollo Conjugación del verbo arrollar Arroyo Río pequeño

8 As Campeón deportivo Has Del verbo haber

9 As Carta de baraja Haz Manojo, superficie

10 Asia Continente Hacia Preposición

11 Ato Conjugación del verbo atar Hato Conjunto de ganado

12 Baca Elemento de transporte Vaca Hembra del toro

13 Bacía Vasija Vacía Conjugación del verbo vaciar

14 Bacilo Bacteria Vacilo Conjugación del verbo vacilar

15 Basar Asentar sobre la base Vasar Armario o balda de cocina

16 Basto Tosco o grosero Vasto Ancho

17 Baya Fruto carnoso Vaya Conjugación del verbo ir

18 Bello Hermoso Vello Pelo corto y suave

19 Bienes Posesiones Vienes Conjugación del verbo venir

20 Bobina Carrete Bovina Perteneciente al toro o la vaca

21 Botar Arrojar Votar Sufragar

114
Palabras Homófonas

Palabra 1 Significado Palabra 2 Significado

22 Botas Calzado Votas Conjugación del verbo votar

23 Cabo Grado militar Cavo Conjugación del verbo cavar

24 Callado Conjugación del verbo callar Cayado Báculo

Concavidad del terreno por


25 Cause Conjugación del verbo causar Cauce
donde corre un río
Pequeña isla con una playa de
26 Cayo Callo Conjugación del verbo callar
baja profundidad

27 Cayó Conjugación del verbo caer Calló Conjugación del verbo callar

28 Ceda Conjugación del verbo ceder Seda Tejido

29 Cede Conjugación del verbo ceder Sede Lugar o local

30 Ciego Invidente Siego Conjugación del verbo segar

31 Cierra Conjugación del verbo cerrar Sierra Instrumento de corte

Trabajador del campo en épocas


32 Ciervo Animal salvaje Siervo
feudales

33 Consejo Cuerpo de consejeros Concejo Cuerpo de concejales

34 Desmallar Quitar malla Desmayar Desvanecer

35 Errar Sinónimo de equivocarse Herrar Poner herradura a una caballería

36 Graba Conjugación del verbo grabar Grava Carga o tributo

37 Grabe Conjugación del verbo Grave De gravedad

38 Hablando Gerundio del verbo hablar Ablando Presente del verbo ablandar

39 Hasta Preposición Asta Lanza o palo

40 Hojear Pasar las hojas Ojear Escudriñar

41 Hola Saludo Ola De mar

42 Libido Deseo carnal Lívido Intensamente pálido

115
Palabras Homófonas

Palabra 1 Significado Palabra 2 Significado

Civilización antigua de
43 Malla Red Maya
Centroamérica

44 Masa Volumen Maza Arma de guerra

45 Onda Ondulación Honda Profunda

46 Pollo Cría de ave Poyo Banco de piedra

47 Rebelar Sublevarse Revelar Descubrir, sacar fotografías

48 Reusar Volver a usar Rehusar Rechazar

49 Riza Conjugación del verbo rizar Risa Expresión de alegría

50 Sabia Persona que sabe Savia Jugo de las plantas

51 Tubo Pieza hueca cilíndrica Tuvo Conjugación del verbo tener

52 Valla Línea de estacas o tablas Vaya Conjugación del verbo ir

53 Vidente Que puede ver Bidente Dos dientes

54 Zeta Letra del abecedario Seta Hongo

55 Zueco Calzado de madera Sueco de Suecia

Las palabras homógrafas o polisémicas se escriben de forma idéntica, pero tienen varios
significados. Por ejemplo: tierno puede referirse a algo que es blando y fácil de partir, a algo
que tiene poco tiempo de existencia, a alguien o algo afectuoso y cariñoso, o a alguien que se
emociona fácilmente.

El contexto en que se emplea una palabra polisémica permite distinguir su significado. Este
significado se llama significado contextual. Debemos fijarnos en el contexto en que se usa la
palabra, es decir, en el tema que se aborda en todo el enunciado o frase. Identificado este tema,
podremos deducir el significado del término.

“A veces el propio contexto nos vale también para descubrir el significado de palabras que
desconocemos y no sabemos que quieren decir. ¿Es necesario comprender todas las palabras
cuando uno está leyendo algo? ¡Por supuesto que no! Pero podemos llegar a comprender el
significado de acuerdo al contexto. Aunque desconoces algunas palabras, de todas formas,

116
puedes captar el sentido general del párrafo. Esto sucede porque el resto del texto te da pistas
que te permiten adivinar las palabras restantes”. 36

Pongamos un ejemplo:

El juego estuvo divertido y esa niña encestó 3 veces el aro.

¿Cuál es el significado contextual del término subrayado?

a) Anillo de compromiso matrimonial


b) Canasta de baloncesto
c) Aparato de gimnasia olímpica para realizar diversos ejercicios
d) Armadura de madera que sostiene el tablero de una mesa

La respuesta correcta es b) canasta de baloncesto. ¿Por qué? Porque encestar es una acción
propia del baloncesto. No encestas en un anillo de compromiso, ni en un aro de gimnasia ni en
una armadura de madera.

A continuación, proponemos un listado de palabras homógrafas:

Palabras homógrafas

Palabra Significado 1 Significado 2 Significado 3

Alude al dueño de una


1 Amo Conjugación del verbo amar
mascota

2 Armado Que porta armas Que se encuentra articulado

3 Banco Entidad financiera Mueble para sentarse

Artefacto para bombear aire


4 Bomba Artefacto explosivo
o agua

Primera versión de algún


Elemento que se utiliza
5 Borrador escrito que se presta a
para borrar una pizarra
futuras correcciones
Persona que en un hotel se Pieza de madera, plástico o
encarga de llevar el metal que se utiliza en las
6 Botones
equipaje de las personas prendas de ropa para
que se hospedan allí. abrochar

Infusión hecha con semillas Planta que produce


7 Café Color marrón
de café semillas de café

36 El Lápiz Rojo (2013). Significado contextual. En: https://ellapizrojo.wordpress.com/2013/02/20/el-significado-contextual/

117
Palabras homógrafas

Palabra Significado 1 Significado 2 Significado 3

Conjugación del verbo


8 Calle Carretera de cemento
callar

Adjetivo que expresa que


9 Cara Rostro de una persona Lado de algo
algún objeto es costoso

Comunicación escrita que se


10 Carta Juego de mesa (baraja) Menú de restaurante
envía por correspondencia

Parte superior de un Recipiente de vidrio o cristal


11 Copa
sombrero o de un árbol para beber

Admiración afectuosa de Ritos o ceremonias con que Adjetivo que se usa para
12 Culto que son objeto algunas se manifiesta la adoración a describir a una persona
cosas una divinidad con mucha educación

Conjugación del verbo Sacerdote de una Iglesia


13 Cura
curar Católica

14 Falda Corte de carne vacuna Parte inferior de un monte Prenda femenina

Herramienta para limar las


15 Lima Fruta cítrica Capital de Perú
uñas

Adjetivo que indica que una Enumeración, en un papel,


16 Lista
persona es inteligente de una serie de elementos

Pieza de madera o metal,


Mono de cuerpo robusto que
de forma cilíndrica, en que
17 Mandril vive en las zonas boscosas de
se asegura lo que se ha de
África Occidental
tornear

Parte que permite agarrar un


18 Mango Fruta
utensilio de cocina

19 Mono Animal Vestimenta

20 Mora Fruta Esposa de un moro

Juguete que representa a


21 Muñeca Parte del brazo humano
niñas o mujeres

118
Palabras homógrafas

Palabra Significado 1 Significado 2 Significado 3

Instrumento para pesar Elemento para ejercitar los


22 Pesa
cosas músculos

23 Planta Parte inferior de los pies Vegetal Fábrica

Dejar pasar un líquido a


Manifestar o dejar traslucir
24 Rezumar través de los poros de un
una cualidad o sentimiento
cuerpo

Accidente geográfico de
25 Río agua que fluye Conjugación del verbo reír
constantemente

Forma cotidiana de llamar


26 Sal Imperativo del verbo salir
al cloruro de sodio

27 Vacilar Titubear o estar indeciso Tomar el pelo a una persona

Elemento hecho de cera


28 Vela Parte del barco
para iluminar un espacio

29 Vino Conjugación del verbo venir Bebida alcohólica

Parte extrema del dedo


30 Yema Sustancia nutritiva del huevo Brote de una planta
opuesta a la uña

Referencias bibliográficas

• El Lápiz Rojo (2013). Significado contextual. En: https://ellapizrojo.wordpress.com/2013/02/20/el-significado-


contextual/

• Mi clase de lenguaje 6. Sinónimos, antónimos, homónimos y parónimos. En:


https://miclasedelenguaje6.wordpress.com/2012/07/05/sinonimos-antonimos-homonimos-y-paronimos/

• Ministerio de Educación (2016). Lengua y Literatura. Noveno Año. Texto para el estudiante. Quito: UASB

• 50 ejemplos de palabras homógrafas. En:


https://www.ejemplos.co/20-ejemplos-de-palabras-homografas/#ixzz5NuLDxeer

119
Banco de preguntas y respuestas

1)

Con base en el texto, identifique el sinónimo de la palabra subrayada

… Para un empleado de cubículo existe un pálpito excitante, y es el presagio de que el viernes se acerca. Algo así
como un anuncio novedoso del paso de un cometa.

a) Deseo
b) Presentimiento
c) Recuerdo
d) Delirio

Respuesta correcta: b)

Un “presagio” es un indicio o señal que anuncia un suceso futuro. Un sinónimo de presagio NO es “deseo”, “recuerdo”
ni “delirio”. Un sinónimo de dicho término es “presentimiento”.

2)

Con base en el texto, identifique el sinónimo de la palabra subrayada

… Para un empleado de cubículo existe un pálpito excitante, y es el vaticinio de que el viernes se acerca. Algo así
como un anuncio novedoso del paso de un cometa.

a) Deseo
b) Delirio
c) Recuerdo
d) Augurio

Respuesta correcta: d)

“Vaticinio” es la acción de anunciar un hecho futuro a partir de ciertos indicios o por simple intuición. Un sinónimo
de vaticinio NO es “deseo”, “recuerdo” ni “delirio”. Un sinónimo de dicho término es “augurio”.

3)

Identifique el sinónimo de la palabra subrayada

En revisión detallada del pasado y examen precario del presente, me despertó un raro pensamiento

a) Usual
b) Inusitado
c) Normal
d) Habitual

Respuesta correcta: b)

En la oración citada, “raro” significa “extraño” o “no habitual”. Inusitado también tiene un significado similar; por
tanto, es un sinónimo correcto de “raro”.

120
4)

Identifique el sinónimo de la palabra subrayada:

El esquema táctico del equipo resultó obsoleto para un campeonato tan competitivo

a) Moderno
b) Vigente
c) Inaudito
d) Caduco

Respuesta correcta: d)

En la oración citada, “obsoleto” significa “caduco”, es decir, “algo que ha quedado claramente anticuado”.

5)

Identifique el sinónimo de la palabra subrayada:

Me gustó la obra teatral que se presentó en el Teatro del Barrio. No entiendo por qué algunos sienten abominación
al verla.

a) Asco
b) Desilusión
c) Despecho
d) Miedo

Respuesta correcta: a)

En la oración citada, “abominación” significa rechazo y condena, en este caso, hacia una obra teatral. Un sinónimo de
abominación es asco.

6)

Identifique el sinónimo de la palabra subrayada:

Los integrantes del grupo de música salieron del hotel y los periodistas se abalanzaron hacia ellos. Los músicos se
sintieron tan acosados que tuvieron que pedir ayuda a sus guardaespaldas y a los policías locales.

a) Comprometidos
b) Molestos
c) Asediados
d) Vigilados

Respuesta correcta: c)

En la oración citada, “acosados” es sinónimo de “asediados” (perseguidos con persistencia y empeño).

7)

Identifique el sinónimo de la palabra subrayada:

Ciertamente, en estos días de vacaciones mi hijo ha pasado encerrado en casa, sin compañía y sin ánimo de salir a
la calle. Quiero ver opciones de un curso vacacional de deportes, sino, seguro se convertirá en un chico anacoreta.

121
a) Aburrido
b) Ermitaño
c) Tímido
d) Encarcelado

Respuesta correcta: b)

Ermitaño es sinónimo de anacoreta: persona que vive en un lugar solitario.

8)

Identifique el sinónimo de la palabra subrayada:

¡Qué tal cicatería! Ni un dólar soltó Juan cuando le pedimos apoyo para los damnificados del terremoto.

a) Tacañería
b) Malicia
c) Grosería
d) Despreocupación

Respuesta correcta: a)

Tacañería es sinónimo de cicatería: modo de ser o de comportarse de las personas avaras.

9)

Identifique el sinónimo de la palabra subrayada:

Hubo un altercado en la calle y por eso he llegado unos minutos tarde. Les ruego me dispensen.

a) Acojan
b) Reciban
c) Eximan
d) Favorezcan

Respuesta correcta: c)

Eximir es sinónimo de dispensar: acción de disculpar a alguien, en este caso, por haber llegado unos minutos tarde.

10)

Identifique el sinónimo de la palabra subrayada:

¿Has escuchado el pasillo “Al besar un pétalo”? Pon atención a la letra: “Al besar tu boca que es cual rosa de un
bello jardín, siento el embeleso de ese beso, ensueño gentil, al besar tus labios como pétalos fragantes, siento que
revivo y solo vivo para ti”. ¡Me encanta!

a) Deseo
b) Ideal
c) Arrebato
d) Cosquilleo

Respuesta correcta: c)

Arrebato o éxtasis es sinónimo de embeleso: estado de disfrute o de placer intenso.

122
11)

Identifique el sinónimo de la palabra subrayada:

La abuelita de María José es una señora impertinente. ¿Puedes creer que, en frente de sus amigas, le dijo
esmirriada? ¡Y recalcó que debería preocuparse más por su cuerpo!

a) Obesa
b) Gorda
c) Escuálida
d) Pálida

Respuesta correcta: c)

Escuálida es sinónimo de esmirriada: persona que está flaca, extenuada o consumida.

12)

Identifique el sinónimo de la palabra subrayada:

Mi tío abuelo me contó que antes, su madre, lo llevaba cada domingo donde un fígaro.

a) Zapatero
b) Sastre
c) Barbero
d) Sacerdote

Respuesta correcta: c)

Barbero es sinónimo de fígaro: persona que tiene como oficio o profesión el cuidado de la barba y el cabello.

13)

Identifique el sinónimo de la palabra subrayada:

Muchos migrantes están viviendo hacinados en casas del sur y del norte de nuestra ciudad.

a) Indigentes
b) Pobres
c) Apretujados
d) Inconformes

Respuesta correcta: c)

Apretujado es sinónimo de hacinado: persona que vive apretada con otras en un sitio puntual.

14)

Identifique el sinónimo de la palabra subrayada:

Increíblemente, las estaturas de ciertas personas las hacen ver liliputienses.

a) Gigantes
b) Diminutas
c) Aletargadas
d) Risueñas

123
Respuesta correcta: b)

Diminuto es sinónimo de liliputiense: persona que es muy pequeña.

15)

Identifique el sinónimo de la palabra subrayada:

No deberías preocuparte porque tienes canas en tu cabello, no veo ningún problema en ello. Es una nadería que
tu cabello se vea blanco.

a) Insignificancia
b) Incomodidad
c) Oportunidad
d) Preciosura

Respuesta correcta: a)

Nadería significa “cosa de poco valor o importancia”; la palabra “insignificancia” resulta ser un sinónimo adecuado
para dicho término.

16)

Identifique el antónimo de la palabra subrayada:

No hay animal manso que atado no se irrite

a) Bondadoso
b) Mansurrón
c) Domado
d) Salvaje

Respuesta correcta: d)

Manso, en este caso, significa domado, domesticado o amansado, es decir, que no es feroz. Por tanto, feroz o
“salvaje” es un antónimo de “manso”.

17)

Identifique el antónimo de la palabra subrayada:

No hay animal manso que atado no se irrite

a) Bondadoso
b) Mansurrón
c) Domado
d) Indómito

Respuesta correcta: d)

Manso, en este caso, significa domado, domesticado o amansado, es decir, que no es feroz. “Indómito” es algo
difícil de domar y, por tanto, es antónimo de manso.

124
18)

Identifique el antónimo de la palabra subrayada:

El juicio presentado ante el tribunal fue resuelto

a) Proceso
b) Pleito
c) Litigio
d) Acuerdo

Respuesta correcta: d)

Juicio, en este caso, es sinónimo de proceso, pleito o litigio, es decir, significa “controversia jurídica entre dos o más
partes que se someten a un tribunal”. Por tanto, un antónimo de “juicio” es “acuerdo”.

19)

Identifique el antónimo de la palabra subrayada

El litigio presentado ante el tribunal fue resuelto

a) Juicio
b) Proceso
c) Pleito
e) Consenso

Respuesta correcta: e)

Litigio, en este caso, es sinónimo de juicio, proceso o pleito, es decir, significa “controversia jurídica entre dos o más
partes que se someten a un tribunal”. Por tanto, un antónimo de “litigio” es “consenso” entendido como “acuerdo o
conformidad en algo”.

20)

Identifique el antónimo de la palabra subrayada.

La misión del científico es contribuir al desarrollo de su pueblo.

a) Truncar
b) Secundar
c) Ayudar
d) Influir

Respuesta correcta: a)

Contribuir significa “Ayudar a otros para lograr algún fin”. Es decir, es sinónimo de ayudar o secundar. Un antónimo
sería “truncar” o interrumpir el logro de un fin.

21)

Identifique el antónimo de la palabra subrayada.

Hace un año fui operada de una oclusión intestinal, y al abrir el doctor encontró diversos tumores con adherencias.

a) Taponamiento
b) Obstrucción
c) Apertura
d) Adherencia

125
Respuesta correcta: c)

Oclusión significa “obstrucción” o “taponamiento”. Su antónimo, por tanto, es “apertura”.

22)

Identifique el antónimo de la palabra subrayada.

Pensé que podría contar con él, pero tuvo una actitud proterva.

a) Retorcida
b) Indiferente
c) Abierta
d) Benévola

Respuesta correcta: d)

Protervo significa “que es malo o perverso”. Su antónimo, por tanto, es “benévolo”.

23)

Identifique el antónimo de la palabra subrayada.

No se calló toda la noche y pasó refocilando a su grupo de amigos.

a) Entreteniendo
b) Aburriendo
c) Gritando
d) Molestando

Respuesta correcta: b)

Refocilar significa “divertir o alegrar con groserías”. Su antónimo, en este caso, sería “aburrir”.

24)

Identifique el antónimo de la palabra subrayada.

Cuando cayó tu anillo a la pileta, se sumió con rapidez.

a) Sumergió
b) Hundió
c) Emergió
d) Arregló

Respuesta correcta: c)

Sumir significa “hundir o sumergir algo”. Su antónimo, en este caso, sería “emerger”.

25)

Identifique el antónimo de la palabra subrayada.

Ahora que trabajas en esa institución te has convertido en una persona venal.

126
a) Sobornable
b) Trabajadora
c) Ociosa
d) Honesta

Respuesta correcta: d)

Venal significa “que se deja sobornar con dádivas”. Su antónimo, en este caso, sería “honesta”.

26)

Identifique el antónimo de la palabra subrayada.

No pudo llegar a la cima del volcán. Se quedó abajo, yerto de frío.

a) Muerto
b) Flexible
c) Inquebrantable
d) Vivo

Respuesta correcta: b)

Yerto significa “que está rígido o inmóvil”. Su antónimo, en este caso, sería “flexible”.

27)

Identifique el antónimo de la palabra subrayada.

La enfermedad lo ha sumergido en un marasmo físico.

a) Vigor
b) Alegría
c) Debilitamiento
d) Agotamiento

Respuesta correcta: a)

Marasmo significa “estado de pérdida de vigor”. Su antónimo, por tanto, sería “vigor”.

28)

Identifique el antónimo de la palabra subrayada.

En aquel parque hay un telescopio donde puedes ver cuerpos luminiscentes.

a) Resplandecientes
b) Grandes
c) Oscuros
d) Pequeños

Respuesta correcta: c)

Luminiscente significa “que emite rayos luminosos”. Su antónimo, por tanto, sería “oscuro”.

127
29)

Identifique el antónimo de la palabra subrayada.

Tiene el cabello hirsuto y poco domable.

a) Grasoso
b) Churón
c) Terso
d) Brillante

Respuesta correcta: c)

Hirsuto es sinónimo de erizado o áspero. Su antónimo, en este caso, sería “terso”.

30)

Identifique el antónimo de la palabra subrayada.

El auto está emporcado.

a) Percudido
b) Estacionado
c) Oloroso
d) Lavado

Respuesta correcta: d)

Emporcado es sinónimo de sucio o percudido. Su antónimo, en este caso, sería “lavado”.

31)

Identifique el antónimo de la palabra subrayada.

Se cansó de deglutir tantas muestras de comida.

a) Oler
b) Vomitar
c) Probar
d) Masticar

Respuesta correcta: b)

Deglutir es la acción de masticar e ingerir alimentos; el antónimo de este término, por tanto, sería “vomitar”.

32)

Complete la oración con las palabras homófonas adecuadas:

En lo más profundo de la noche __________ una estrella y él __________ mientras pedía el mismo deseo de
siempre.

a) Callo – cayó
b) Cayo – callo
c) Calló – callo
d) Cayó – calló

Respuesta correcta: d)

128
Las palabras homófonas son términos que suenan igual, pero se escriben de un modo diferente y tienen significados
distintos. Cayo y callo, y cayó y calló suenan de forma similar pero su definición es distinta:

Cayo es una pequeña isla con una playa de baja profundidad, formada en la superficie de un arrecife de coral.

Callo tiene dos acepciones: a) es el nombre coloquial de la “hiperqueratosis” o zona de piel en la que se produce
acumulación de queratina; y b) es la conjugación en presente y en primera persona del singular del verbo “callar”; se
usa, por ejemplo, cuando se dice: “Yo callo cuando tú hablas”.

Cayó es el pasado del verbo “caer”, entendido como “moverse de arriba hacia abajo”.

Calló, en cambio, es el pasado del verbo “callar” (sinónimo de silenciar). Una estrella puede caerse (moverse de
arriba hacia abajo); no puede, de ningún modo, “callarse” como lo hace un ser humano (una estrella no habla).
Cuando cae una estrella, muchas personas se callan (guardan silencio) y piden un deseo. Por eso, es correcto decir:

En lo más profundo de la noche cayó una estrella y él calló mientras pedía el mismo deseo de siempre.

33)

Complete la oración con las palabras homófonas adecuadas:

Como no tenía agua, fui al _________ del río y recolecté un poco de líquido. Al levantarme con el pozuelo, sin
embargo, me caí en la orilla y me _____________ el mismo río.

a) Arrollo – arrolló
b) Arroyo – arroyó
c) Arrollo – arroyó
d) Arroyo – arrolló

Respuesta correcta: d)

Las palabras homófonas son términos que suenan igual, pero se escriben de un modo diferente y tienen significados
distintos. Arroyo y arrolló no significan lo mismo: Arroyo es un río pequeño. Arrolló es la conjugación en pasado del
verbo arrollar cuyo sinónimo es arrojar. Por eso, es correcto decir:

Como no tenía agua, fui al arroyo del río y recolecté un poco de líquido. Al levantarme con el pozuelo, sin embargo,
me caí en la orilla y me arrolló el mismo río.

34)

Complete la oración con las palabras homófonas adecuadas:

El muchacho le pidió a su cliente que le _______ un poco de su tiempo para mostrarle una _____ traída de China.

a) Ceda – seda
b) Seda – ceda
c) Seda – seda
d) Ceda – ceda

Respuesta correcta: a)

Las palabras homófonas son términos que suenan igual, pero se escriben de un modo diferente y tienen significados
distintos. Ceda y seda no significan lo mismo: Ceda proviene del verbo ceder. Seda es una tela suave típica de China.
Por eso, es correcto decir: El muchacho le pidió a su cliente que le ceda un poco de su tiempo para mostrarle una
seda traída de China.

129
35)

Complete la oración con las palabras homófonas adecuadas:

El _______ que trabajaba en las tierras del Norte aseguró que en ese sector hay un ______ que anda suelto.

a) Ciervo – siervo
b) Siervo – ciervo
c) Siervo – siervo
d) Ciervo – ciervo

Respuesta correcta: b)

Las palabras homófonas son términos que suenan igual, pero se escriben de un modo diferente y tienen significados
distintos. Ciervo y siervo no significan lo mismo: Ciervo es un animal salvaje. Siervo es un campesino. Por eso, es
correcto decir: El siervo que trabajaba en las tierras del Norte aseguró que en ese sector hay un ciervo que anda
suelto.

36)

Complete la oración con las palabras homófonas adecuadas:

Se dirigió al terreno y ________ la cerca. No sé si fue el sol o la falta de alimento pero, cuando llegó a casa, se
________.

a) Desmayó - desmalló
b) Desmayo - desmallo
c) Desmalló - desmayó
d) Desmayó - desmallo

Respuesta correcta: c)

Las palabras homófonas son términos que suenan igual, pero se escriben de un modo diferente y tienen significados
distintos. Desmalló y desmayó no significan lo mismo: Desmalló es la conjugación en pasado del verbo desmallar, que
significa “quitar una malla”. Desmayó es la conjugación en pasado del verbo desmayar, que significa “desvanecerse”.
Por eso, es correcto decir: Se dirigió al terreno y desmalló la cerca. No sé si fue el sol o la falta de alimento pero,
cuando llegó a casa, se desmayó.

37)

Complete la oración con las palabras homófonas adecuadas:

Mientras ella ____ su cabello yo me muero de ____ leyendo un cómic.

a) Risa - risa
b) Riza - riza
c) Riza - risa
d) Risa - riza

Respuesta correcta: c)

Las palabras homófonas son términos que suenan igual, pero se escriben de un modo diferente y tienen significados
distintos. Riza y Risa no significan lo mismo: Riza es la conjugación en presente del verbo rizar, que significa “ondular”.
Risa es una expresión de alegría. Por eso, es correcto decir: Mientras ella riza su cabello yo me muero de risa leyendo
un cómic.

130
38)

Complete la oración con las palabras homófonas adecuadas:

Jorge _____que ir a comprar un ______ de escape para su auto.

a) Tubo - tubo
b) Tuvo - tuvo
c) Tubo – tuvo
d) Tuvo - Tubo

Respuesta correcta: d)

Las palabras homófonas son términos que suenan igual, pero se escriben de un modo diferente y tienen significados
distintos. Tuvo y Tubo no significan lo mismo: Tuvo es la conjugación en pasado del verbo tener. Tubo es una pieza
hueca cilíndrica. Por eso, es correcto decir: Jorge tuvo que ir a comprar un tubo de escape para su auto.

39)

Complete las siguientes oraciones con la palabra homófona que corresponda:

Espero que él __________ a mi fiesta.


Él intento saltar la __________ pero estaba muy alta.
Mis compañeros me pidieron que _________ al concurso.
La _________ estaba muy madura.

a) baya – vaya – vaya – valla


b) baya – valla – baya – vaya
c) vaya – valla – vaya – baya
d) vaya – baya – valla – vaya

Respuesta correcta: c)

Las palabras homófonas son términos que suenan igual, pero se escriben de un modo diferente y tienen significados
distintos. Baya, vaya y valla suenan de forma similar pero su definición es distinta:

Baya es un término que se emplea para nombrar a un fruto carnoso que dispone de pulpa, en la cual se encuentran
las semillas.

Vaya es un concepto que puede emplearse de distintas maneras: a) como conjugación del verbo ir (“Cuando vaya a
tu casa, te enseñaré a usar la computadora), b) como interjección o enunciado exclamativo (“¡Vaya cena que has
preparado!”).

Valla también tiene diferentes acepciones: a) cerco que se arma con diferentes estacas para proteger o cerrar un
espacio, b) obstáculos que deben saltar los atletas en determinadas competencias, y c) carteles que se instalan en
espacios públicos para promocionar distintos productos.

Si conocemos el significado de cada palabra, podemos resolver este ejercicio del siguiente modo:

Espero que él vaya a mi fiesta


Él intento saltar la valla, pero estaba muy alta.
Mis compañeros me pidieron que vaya al concurso.
La baya estaba muy madura.

131
40)

Complete la oración con las palabras homónimas correctas:

Cuando fui a que reparen el reloj de papá, hicieron la pieza en un _______ y después pudimos ver a un _______ en
el zoológico.


a) mandril - mandril
b) gato - gato
c) imán - imán
d) coco - coco

Respuesta correcta: a)

Este caso corresponde a un par de palabras homónimas de un tipo específico: palabras homógrafas o polisémicas, es
decir, aquellas que se escriben igual pero que no tienen un mismo significado pues éste depende del contexto en que
están escritas. “Mandril” tiene dos significados que se aplican a la oración propuesta: “pieza de madera o metal, de
forma cilíndrica, en que se asegura lo que se ha de tornear” y “mono de cuerpo robusto que vive en las zonas boscosas
de África Occidental”.

41)

Complete la oración con las palabras homónimas correctas:

En diciembre aprovechamos para preparar deliciosos potajes. El otro día, mientras comía un _______, mi papá
preparaba unos deliciosos pristiños con miel. Casi se quema al agarrar el _______ del sartén donde los freía.

a) coco - coco
b) mango - mango
c) tomate – tomate
d) taco - taco

Respuesta correcta: b)

Este caso corresponde a un par de palabras homónimas de un tipo específico: palabras homógrafas o polisémicas, es
decir, aquellas que se escriben igual pero que no tienen un mismo significado pues éste depende del contexto en que
están escritas. “Mango” tiene dos significados que se aplican a la oración propuesta: “fruto oval, aromático y dulce”
y “parte o pieza por donde suelen cogerse ciertos utensilios de cocina”.

42)

Identifique el significado contextual del término subrayado:

Un lento caminar, cabizbajo iba con la lluvia, solamente podría decirse que sus ojos rezumaban de tristeza.

a) Dejar pasar un líquido a través de los poros de un cuerpo


b) Manifestar o dejar traslucir una cualidad o sentimiento
c) Dicho de algunas cosas como trabajos, desgracias: venir, caer sobre alguien con abundancia
d) Ocasionar un daño a las cosas, desmejorándolas o desluciéndolas

Respuesta correcta: b)

El verbo “rezumar” tiene varios significados; es decir, es una palabra homógrafa o polisémica cuyo significado se
comprende dependiendo del contexto en que está escrita. Uno de ellos es “dejar pasar un líquido a través de los
poros de un cuerpo”; por ejemplo: “esta pared rezuma humedad”. Otro significado es “manifestar o dejar traslucir
una cualidad o sentimiento”; por ejemplo: “aquella niña rezumaba de alegría”.

132
En nuestro caso, cuando decimos “sus ojos rezumaban de tristeza” estamos afirmando que “sus ojos dejaban traslucir
ese sentimiento llamado tristeza”. La opción correcta, entonces, es “manifestar o dejar traslucir una cualidad o
sentimiento”.

Nota: si la frase dijese “sus ojos rezumaban llanto”, el significado de “rezumar” sería “dejar pasar un líquido a través
de los poros de un cuerpo” pues estaríamos afirmando que “sus ojos dejaban pasar ese líquido llamado llanto”.

43)

Identifique el significado adecuado de la palabra subrayada:

El proyecto finalmente no pudo ejecutarse porque Antonio vaciló a la hora de tomar la decisión y no firmó el
acuerdo.

a) Engañar, tomar el pelo, burlarse o reírse de alguien


b) Dicho de una persona: titubear, estar indecisa
c) Dicho de una cosa: moverse indeterminadamente
d) Hablar o escribir sin concierto ni propósito determinado

Respuesta correcta: b)

El verbo “vacilar” tiene varios significados; es decir, es una palabra homógrafa o polisémica cuyo significado se
comprende dependiendo del contexto en que está escrita. En nuestro caso, ¿de qué se habla en toda la frase? De un
proyecto que no resultó porque Antonio, finalmente, no firmó un acuerdo necesario para que ello sea posible:
Antonio “vaciló” a la hora de tomar esa decisión de firmar. ¿Qué significa, entonces, “vaciló”? Significa “titubeó” o
“estuvo indeciso”.

44)

Identifique el significado contextual de la palabra subrayada

Muchos fanáticos se reunieron para recibir a la banda de culto que arribó al país el pasado martes, los admiradores
calificaron el acontecimiento como un premio a su devoción.

a) Admiración afectuosa de que son objeto algunas cosas


b) Dicho de las tierras o de las plantas cultivadas
c) Dotado de las calidades que provienen de la cultura o instrucción
d) Honor que se tributa religiosamente a lo que se considera sagrado

Respuesta correcta: a)

La palabra “culto” tiene varios significados; es decir, es una palabra homógrafa o polisémica cuyo significado se
comprende dependiendo del contexto en que está escrita. En nuestro caso, ¿de qué se habla en toda la frase? De un
grupo de fans y admiradores de una banda musical “de culto”. Entonces, ¿qué significa “culto”? Evidentemente, la
admiración afectuosa de que son objeto algunas cosas.

45)

Identifique el significado contextual de la palabra subrayada

Las calles se inundaron de seguidores que gritaban vivas y se regocijaban por el retorno de su compatriota, los
periódicos lo calificaron como un culto a su labor internacional.

a) Admiración afectuosa de que son objeto algunas cosas


b) Dicho de las tierras o de las plantas cultivadas
c) Dotado de las calidades que provienen de la cultura o instrucción
d) Honor que se tributa religiosamente a lo que se considera sagrado

133
Respuesta correcta: a)

La palabra “culto” tiene varios significados; es decir, es una palabra homógrafa o polisémica cuyo significado se
comprende dependiendo del contexto en que está escrita. En nuestro caso, ¿de qué se habla en toda la frase? Del
culto a la labor internacional de un compatriota cuyo retorno fue motivo de regocijo por parte de sus seguidores.
Entonces, ¿qué significa “culto”? Evidentemente, la admiración afectuosa de la que es objeto dicha labor
internacional.

46)

Identifique el significado contextual de la palabra subrayada

Su participación en las olimpiadas de gimnasia la dejaron exhausta. Ayer llegó casi a medianoche y se tumbó en su
lecho.

a) Capa de alguna materia extendida horizontalmente sobre una cosa


b) Suelo de los carros o carretas
c) Cauce del río o de un canal
d) Mueble formado por un armazón y un somier que sostienen un colchón

Respuesta correcta: d)

La palabra “lecho” tiene varios significados; es decir, es una palabra homógrafa o polisémica cuyo significado se
comprende dependiendo del contexto en que está escrita. En nuestro caso, “lecho” hace referencia a una cama: un
mueble formado por un armazón y somier que sostienen un colchón.

134
Tema 8. Analogías entre palabras

Texto resumen

Una analogía es la semejanza o afinidad de relaciones entre dos pares de palabras. Esta
semejanza emerge de un proceso de comparación y se consolida considerando los rasgos más
importantes y notorios de dichas relaciones.

Para entender este concepto, veamos un par de ejemplos de analogía verbal:

• Sal es a alimento como broma es a plática

“Para percibir la semejanza entre ambos pares, primero debemos saber cuál es el vínculo
existente entre sal y alimento, para luego compararlo con el vínculo que existe entre broma
y plática. La sal es la sustancia que se utiliza como ingrediente para dotar de sazón al
alimento, para hacerlo agradable al paladar y para que pueda ser ingerido por los comensales
con mayor deleite. De modo similar, la broma es el elemento que se utiliza en la plática para
darle amenidad y hacer que la conversación sea más agradable e interesante para los
interlocutores.

Además, la cantidad de sal en el alimento debe suministrarse con mucho tino: la falta de sal
hace insípida la comida y su exceso lo torna salado y de mal gusto. Del mismo modo, la broma
en la plática debe emplearse con prudencia: la falta de bromas hace tediosa o aburrida la
plática, y su exceso hace que se torne de mal gusto, grosera y hasta ofensiva.

En consecuencia, esta analogía se puede enunciar de la siguiente manera: así como la sal es
el ingrediente que da sabor al alimento, también la broma es el ingrediente que da gusto a
la plática”.37

• Silla es a comedor como butaca es a cinema

La silla es un mueble cuyo lugar adecuado es el comedor, así también la butaca es otro
mueble cuyo lugar adecuado es el cinema.

• Vértebra es a columna como montaña es a cordillera

Relacionando los términos de cada par, entendemos que, así como las vértebras son huesos
con apariencia similar que están unidos y ordenados formando parte de una columna
vertebral, las montañas son grandes masas de tierra y piedra que están unidas y ordenadas
formando parte de una cordillera.

37 Tomado de: http://razonamiento-verbal1.blogspot.com/2014/03/que-es-una-analogia-verbal-y-ejemplos.html

135
Referencias bibliográficas

• Qué es un analogía verbal y ejemplos. En: http://razonamiento-verbal1.blogspot.com/2014/03/que-es-una-


analogia-verbal-y-ejemplos.html

Banco de preguntas y respuestas

1)

Timar es a estafar como:

a) Ocaso es a aurora
b) Malo es a pésimo
c) Desinfectar es a purificar
d) Avalancha es a destrucción

Respuesta correcta: c)

Una analogía es una semejanza o afinidad de relaciones entre dos pares de palabras. La relación que identifiquemos
entre Timar y estafar debemos compararla con dos términos que se relacionen de una manera similar. Timar y estafar
son sinónimos. Ambas significan “quitar o robar una cosa con engaño”. De las opciones enlistadas, ¿cuál incluye dos
palabras que se relacionen entre sí porque son sinónimos?

Ocaso y aurora no son sinónimos. Ocaso es la “puesta del sol o de otro astro por el horizonte” y aurora es “luz que
aparece en el oriente antes de la salida del sol”.

Malo y pésimo no son palabras que signifiquen lo mismo. Malo significa “algo perjudicial”. Pésimo es el superlativo
de malo. Significa algo que es muy malo, que no puede ser peor.

Avalancha y destrucción tampoco son lo mismo. La avalancha (deslizamiento repentino de una masa de nieve, hielo
o rocas) puede provocar destrucción (pérdida irreparable), pero no son sinónimos.

Desinfectar y purificar, en cambio, sí son sinónimos: Ambas, en un contexto similar, significan “limpiar una cosa para
eliminar gérmenes nocivos”. Por tanto, timar es a estafar como desinfectar es a purificar.

2)

Inhumar es a exhumar como:

a) Denunciar es a encubrir
b) Ocultar es a la mostrar
c) Ascender es a descender
d) Hablar es a callar

Respuesta correcta: b)

Una analogía es una semejanza o afinidad de relaciones entre dos pares de palabras. La relación que identifiquemos
entre inhumar y exhumar debemos compararla con dos términos que se relacionen de una manera similar. Al inhumar
se cubre el cadáver con tierra y al exhumar se le hace visible, se le desentierra; análogamente, al ocultar se esconde
el objeto y al mostrar se facilita su observación.

136
3)

Mohíno es a exultado como:

a) Novato es a insipiente
b) Apático es a entusiasta
c) Ofuscado es a lúcido
d) Sereno es a agravado

Respuesta correcta: b)

Una analogía es una semejanza o afinidad de relaciones entre dos pares de palabras. La relación que identifiquemos
entre mohíno y exultado debemos compararla con dos términos que se relacionen de una manera similar. Mohíno y
exultado son palabras contrarias, se refieren a dos estados de ánimo de las personas: mohíno implica disminución
anímica y exultado se refiere al incremento de ánimo. De igual forma, apático y entusiasta son dos palabras contrarias
referidas al estado de ánimo de una persona y con significados similares a los antedichos: apático implica disminución
anímica y entusiasta se relaciona a un buen estado de ánimo.

4)

Imán es a hierro como:

a) Carnada es a presa
b) Trampa es a animal
c) Novedad es a curiosidad
d) Miel es a abeja

Respuesta correcta: a)

Una analogía es una semejanza o afinidad de relaciones entre dos pares de palabras. La relación que identifiquemos
entre imán y hierro debemos compararla con dos términos que se relacionen de una manera similar. El imán atrae al
hierro, haciendo que éste se desplace hacia donde se ubica dicho objeto. Del mismo modo, la carnada atrae a la
presa, haciendo que ésta se desplace hacia donde se ubica la carnada.

5)

Interés es a capital como:

a) Salario es a bonificación
b) Moneda es a déficit
c) Renta es a propiedad
d) Participación es a utilidad

Respuesta correcta: c)

Una analogía es una semejanza o afinidad de relaciones entre dos pares de palabras. La relación que identifiquemos
entre interés y capital debemos compararla con dos términos que se relacionen de una manera similar. El capital
produce interés. Del mismo modo, la propiedad produce renta.

6)

Cauce es a río como:

a) Carretera es a coche
b) Bahía es a mar
c) Pasaje es a viaje
d) Vuelo es a aire

137
Respuesta correcta: a)

Una analogía es una semejanza o afinidad de relaciones entre dos pares de palabras. La relación que identifiquemos
entre cauce y río debemos compararla con dos términos que se relacionen de una manera similar. El cauce es la
concavidad del terreno por donde corre un río. Del mismo modo, la carretera es espacio en donde corren los
automóviles conducidos por una persona.

7)

Inflar es a estallar como:

a) Excavar es a disminuir
b) Exponer es a evaporar
c) Exhalar es a disipar
d) Extender es a romper

Respuesta correcta: d)

Una analogía es una semejanza o afinidad de relaciones entre dos pares de palabras. La relación que identifiquemos
entre inflar y estallar debemos compararla con dos términos que se relacionen de una manera similar. Al inflar
demasiado un globo éste estalla (desaparece). Del mismo modo, al extender demasiado un hilo, éste se rompe
(desaparece).

8)

Actos es a drama como:

a) Sílabas es a verso
b) Capítulos es a novela
c) Escenas es a teatro
d) Palabras es a párrafo

Respuesta correcta: b)

Una analogía es una semejanza o afinidad de relaciones entre dos pares de palabras. La relación que identifiquemos
entre actos y drama debemos compararla con dos términos que se relacionen de una manera similar. Así como el
drama (género literario) se divide en actos, la novela (género literario) se divide en capítulos.

9)

Proliferación es a número como:

a) Calor es a temperatura
b) Elevación es a vuelo
c) Expansión es a volumen
d) Inclinación es a ascenso

Respuesta correcta: a)

Una analogía es una semejanza o afinidad de relaciones entre dos pares de palabras. La relación que identifiquemos
entre proliferación y número debemos compararla con dos términos que se relacionen de una manera similar. Así
como el número es una forma de medir la proliferación o incremento de algo, la temperatura es una forma de medir
el calor de algo.

138
10)

Irritante es a latosa como:

a) Valiente es a persistente
b) Intrigante es a odiosa
c) Rencorosa es a manipuladora
d) Descortés es a grosera

Respuesta correcta: d)

Una analogía es una semejanza o afinidad de relaciones entre dos pares de palabras. La relación que identifiquemos
entre irritante y latosa debemos compararla con dos términos que se relacionen de una manera similar. Irritante es
sinónimo de latosa (persona irritante); del mismo modo, descortés es sinónimo de grosera (persona que se comporta
sin cortesía).

11)

Complete la analogía:

____________ es a leche como oveja es a____________.

a) Bebida – esponjosa
b) Vaca – lana
c) Nutritiva – animal
d) Líquida - balido

Respuesta correcta: b)

La analogía, en este caso, plantea dos palabras que hacen referencia a un animal y al elemento básico que se extrae
de ese animal: de la vaca se extrae leche y de la oveja se extrae lana. Por tanto, “vaca es a la leche como oveja es a
lana”.

12)

Complete la analogía:

___________ es a tiburón como pulmón es a ____________.

a) Branquia – conejo
b) Branquia – aire
c) Espina – conejo
d) Espina – aire

Respuesta correcta: a)

La analogía, en este caso, plantea dos palabras que hacen referencia a un órgano de respiración y al animal que posee
dicho órgano: la branquia es un órgano vital del tiburón, así como el pulmón lo es del conejo.

13)

Complete la analogía:

___________ es a impedir como tratar es a ____________.

139
a) Oponer – curar
b) Fracasar - curar
c) Oponer - dañar
d) Fracasar - dañar

Respuesta correcta: a)

La analogía, en este caso, plantea dos palabras que se relacionan entre sí porque una de ellas es la causa de la otra.
Así, cuando alguien se opone a algo (causa), impide que ese algo salga adelante (consecuencia). De igual manera,
cuando alguien trata una enfermedad (causa), logra que su curación se haga realidad (consecuencia). Entonces:
oponer es a impedir como tratar es a curar.

14)

Complete la analogía:

___________ es a partir como robar es a ____________.

a) Irse - donar
b) Fugarse – donar
c) Fugarse – tomar
d) Llegar - tomar

Respuesta correcta: c)

La analogía, en este caso, plantea dos palabras que se relacionan entre sí porque una de ellas corresponde a la acción
que se requiere previamente para que la otra se concrete. Así, cuando alguien se fuga (acción concreta) requiere,
primero, partir del sitio desde donde lo hace. Así mismo, cuando alguien roba (acción concreta) requiere,
previamente, tomar el objeto del robo. Entonces: Fugarse es a partir como robar es a tomar.

15)

Complete la analogía:

___________ es a insensibilidad como rechazo es a ____________.

a) Dureza - acogida
b) Suavidad - desilusión
c) Anestesia - acogida
d) Anestesia - desilusión

Respuesta correcta: d)

La analogía, en este caso, plantea dos palabras que se relacionan entre sí porque una de ellas es causa de la otra. Así,
cuando a alguien se le coloca anestesia (causa) su cuerpo se torna insensible (consecuencia). Del mismo modo,
cuando a alguien le rechazan (causa) ese alguien se desilusiona (consecuencia). Entonces: Anestesia es a insensibilidad
como rechazo es a desilusión.

140
16)

Complete la analogía:

___________ es a experimento como sonata es a ____________.

a) Probeta - concierto
b) Ensayo – baile
c) Probeta - baile
d) Ensayo - concierto

Respuesta correcta: a)

La opción más coherente es la siguiente: probeta es un tipo de experimento, del mismo modo que una sonata es un
tipo de concierto musical.

17)

Complete la analogía:

___________ es a paz como balanza es a ____________.

a) Guerra - equilibrio
b) Paloma - justicia
c) Paloma - equilibrio
d) Guerra - justicia

Respuesta correcta: b)

La analogía, en este caso, plantea dos palabras que se relacionan entre sí porque una de ellas simboliza a la otra. Así
como la paloma es símbolo de paz, la balanza es símbolo de justicia. Entonces: paloma es a paz como balanza es la
justicia.

141
Tema 9. Prefijos y sufijos

Texto resumen

Los prefijos y sufijos son fonemas que ayudan a formar nuevas palabras. El prefijo antecede a
una palabra para modificar su sentido gramatical; cuando se escribe en forma aislada, no tiene
un sentido completo. Hay distintos prefijos:

Tipo Prefijo Significado Ejemplo

A-
negación apolítico, analfabeto
AN-

ANTI- contrario, oposición antinazi


PREFIJOS NEGATIVOS

CONTRA- opuesto, contrario contraespionaje

DES-
DIS- Inversión de significado desunión, disconforme, degenerar
DE-

EX- privación, más allá exculpar, exministro, extender

EXTRA- fuera de extraordinario

IN-
IM- negación, privación, dentro inmanente, imbatible, ilegal
I-

A- aproximación aterrizar, amenizar

ANTE- delante antesala, anteponer

ENTRE- situación intermedia,


entreplanta, interurbano
INTER- reciprocidad

IN- dentro inseminar, insuflar


PREFIJOS LOCATIVOS

INFRA- debajo de infrahumano

INTRA- dentro de intramuscular

PRO- en vez de, en lugar de pronombre

SUB-
bajo, inferioridad subacuático, socavar
SO-
TRANS- al otro lado, en la parte
transportar, trastienda
TRAS- opuesta

VICE- inmediatamente inferior a vicepresidente

142
Tipo Prefijo Significado Ejemplo

ANTE- anterioridad en el tiempo anteayer


PREFIJOS TEMPORALES

PRE- antelación preclásico

POS-
posterioridad posponer, postparto
POST-

PER- a través de pernoctar

ARCHI- muy, preeminencia archidiócesis


PREFIJOS DE INTENSIFICACIÓN

EXTRA- fuera de lugar extradivertido

HIPER- por encima de hiperacidez

HIPO- por debajo hipotensión

SUPER- frado sumo, exceso superproducción

ULTRA- en alto grado ultraligero

BI-
dos, doble bicolor, bisabuelo
BIS
PREFIJOS QUE INDICAN CANTIDAD

MINI pequeño minifalda

MONO único, uno solo monocromático

MULTI- abundancia, variedad multicolor

PLURI- multiplicidad plurinacional

SEMI- medio, casi semicírculo

UNI- uno solo unilateral

AERO- aire aeropuerto


PREFIJOS DE RAÍCES

ARISTO- el mejor aristocracia


GRECOLATINAS

AUTO- por uno mismo autógrafo

BI- dos bimensual

BIO- vida biografía

143
Tipo Prefijo Significado Ejemplo

BIBLIO- libro o materia impreso biblioteca

COSMO- universo cosmovisión

DECA- diez decámetro

ETNO- raza etnografía

FOTO- luz fotografía

GEO- tierra geografía

HIPER- exceso Hipertensión

HIPO- debajo de Hipotermia

MINI- pequeño Minifundio

PSICO- alma psicotécnico

SEUDO- falso seudoprofeta

TELE- lejos televisión

El sufijo es un término que, a diferencia del prefijo, va escrito al final (y no al comienzo) de una
palabra para modificar y completar su sentido. Hay distintos sufijos, entre otros:

Tipo Sufijo Significado Ejemplo

aumentativo, golpe,
-AZO perrazo, porrazo, artistazo
valoración
SUFIJOS AUMENTATIVOS

-ÓN
repetición, despectivo preguntón, fregona
-ONA

-OTE, -OTA aumentativo, peyorativo camarote, gafotas

-UDO aumentativo, peyorativo forzudo, melenudo

144
Tipo Sufijo Significado Ejemplo

-ITO
disminución, valor afectivo casita, pajarita
-ITA
SUFIJOS DIMINUTIVOS

afectividad, aunque también


-ILLO perrillo, articulillo
desafecto

-ÍN moderado valor peyorativo tontín

-UELO despectivo pero también


ladronzuelo, callejuela
-UELA jocoso

-ADO acción, grupo de, cargo lavado, campesinado, principado

-AJE medida, oficio o labor Kilometraje, caudillaje


SUFIJOS QUE FORMAN SUSTANTIVOS

-ANCIA
abstractos de cualidad tolerancia, bonanza
-ANZA

-CIÓN acción de recaudación, señalización

lugar de venta, abstracción


-ERÍA lechería, palabrería
con matiz despectivo

-ERO profesión lechero, panadero

-EZ
abstracto de cualidad robustez, belleza
-EZA

-MENTO
acción de juramento, conocimiento
-MIENTO

-ISTA agente, seguidor de guionista, marxista

-AL relativo a musical

natural de (gentilicio), que


-ANO zamorano, cercano
está
SUFIJOS QUE FORMAN ADJETIVOS

-BLE
-ABLE posibilidad de, cualidad convertible, amable
-IBLE

-OIDE parecido a humanoide

-IENTO cualidad hambriento

-IL cualidad varonil

145
Tipo Sufijo Significado Ejemplo

-ALGIA dolor neuralgia

-ANDRIA varón poliandria

-CRACIA
fuerza, denominación democracia, autocracia
-CRATA

-CRONÍA tiempo isocronía

-FILIA amistad bibliofilia

-FOBIA que siente repulsión, horror homofobia. claustrofobia


SUFIJOS DE RAÍCES GRECOLATINAS

-GÉNESIS origen orogénesis

-GRAMA escrito telegrama, crucigrama

-HÍDRICO ácido que contiene oxígeno clorhídrico

-LOGO especialista en psicólogo

-MANÍA locura cleptomanía

-MORFO que tiene forma amorfo

-PATÍA sentimiento, dolencia telepatía

-TECNIA técnica pirotecnia

-TERAPIA curación bailoterapia

-VORO que come herbívoro

Referencias bibliográficas

• Lista de prefijos y sufijos con sus significados. En: http://lenguajelenguayhabla.blogspot.com/2011/12/lista-de-


prefijos-y-sufijos-con-sus.html

• Prefijos y sufijos. Concepto y ejemplos. En: https://www.aboutespanol.com/prefijos-y-sufijos-concepto-y-


ejemplos-3970174

146
Banco de preguntas y respuestas

1)

Identifique la palabra que contiene un prefijo:

El director de la biblioteca recibió una calurosa felicitación por su puntualidad

a) Felicitación
b) Director
c) Biblioteca
d) Puntualidad

Respuesta correcta: c)

El prefijo antecede a una palabra para modificar su sentido gramatical. El sufijo es un término que, a diferencia del
prefijo, va escrito al final (y no al comienzo) de una palabra para modificar y completar su sentido.

De las palabras enlistadas, “biblioteca” contiene un prefijo (biblio) y “felicitación” un sufijo (ción).

2)

Identifique las oraciones que utilizan prefijos:

1. Continúa la campaña de prevención de embarazo en adolescentes


2. Varios planteles educativos recibieron charlas sobre el tema
3. La información se complementó con cuadros estadísticos
4. Se esperó el apoyo internacional para superar esta situación

a) 1, 2
b) 1, 4
c) 2, 3
d) 3, 4

Respuesta correcta: b)

El prefijo antecede a una palabra para modificar su sentido gramatical. El sufijo es un término que, a diferencia del
prefijo, va escrito al final (y no al comienzo) de una palabra para modificar y completar su sentido.

De las oraciones enlistadas, la 1 y la 4 contienen prefijos:

• Continúa la campaña de prevención de embarazo en adolescentes (pre).


• Se esperó el apoyo internacional para superar esta situación (inter).

3)

Identifique las oraciones que emplean sufijos:

1. En muchos lugares de Tungurahua se encuentran talleres de pirotecnia.


2. El Vicepresidente del curso no asistirá a la charla sobre prevención de drogas.
3. Ese hombre dice que sabe mucho de religión pero yo lo veo como un seudoprofeta.
4. Es una lástima que aún existan personas que dicen tener homofobia.

a) 1, 2
b) 1, 4
c) 2, 3
d) 3, 4

147
Respuesta correcta: b) 1, 4.

El prefijo antecede a una palabra para modificar su sentido gramatical. El sufijo es un término que, a diferencia del
prefijo, va escrito al final (y no al comienzo) de una palabra para modificar y completar su sentido.

De las oraciones enlistadas, la 1 y la 4 contienen sufijos:

• En muchos lugares de Tungurahua se encuentran talleres de pirotecnia.


• Es una lástima que aún existan personas que dicen tener homofobia.

4)

Lea la frase e identifique la palabra que está formada por el sufijo que significa capacidad o posibilidad

El talentoso adolescente consiguió una batería recargable para su proyecto de Mecánica

e) Mecánica
f) Adolescente
g) Recargable
h) Talentoso

Respuesta correcta: c)

En efecto, la palabra recargable contiene el sufijo -able que significa posibilidad o capacidad: del verbo recargar se
forma el adjetivo recargable (capacidad de recargar).

5)

Lea la frase e identifique la palabra que está formada por el sufijo que significa horror o repulsión

Ciertamente mi psicólogo me sugirió que ponga de parte y que use los ascensores para ir superando mi
claustrofobia. Pero desde hoy hago un juramento: no vuelvo a subir a esos aparatos porque, además, siento que
son los causantes de mis neuralgias.

a) Psicólogo
b) Claustrofobia
c) Juramento
d) Neuralgias

Respuesta correcta: b)

En efecto, la palabra claustrofobia contiene el sufijo -fobia que significa horror, miedo o repulsión de algo; e este
caso, claustrofobia significa horror de quedarse enclaustrado o encerrado.

6)

Seleccione los prefijos y sufijos con las palabras que los ejemplifican:

1. Prefijo a. niñez – técnicamente – simbolismo - caminante

b. fideo – álgebra – barrio - guitarra

2. Sufijo c. infrahumano – subterráneo – intercambio - hidratante

148
a) 1a, 2b
b) 1b, 2c
c) 1c, 2a
d) 1c, 2b

Respuesta correcta: c) 1c, 2a.

El prefijo antecede a una palabra para modificar su sentido gramatical. El sufijo es un término que, a diferencia del
prefijo, va escrito al final (y no al comienzo) de una palabra para modificar y completar su sentido.

De los listados de ejemplos citados, el numeral “c” contiene palabras con prefijos: infrahumano – subterráneo –
intercambio – hidratante. La opción “a”, en cambio, contiene sufijos: niñez – técnicamente –simbolismo - caminante.

7)

Seleccione los prefijos y sufijos con las palabras que los ejemplifican:

1. Prefijo a. psicólogo – cleptomanía - herbívoro

b. agua – azúcar - aceite

2. Sufijo c. pronombre – inseminar - transportar

a) 1a, 2b
b) 1c, 2a
c) 1b, 2a
d) 1c, 2b

Respuesta correcta: b) 1c, 2a.

El prefijo antecede a una palabra para modificar su sentido gramatical. El sufijo es un término que, a diferencia del
prefijo, va escrito al final (y no al comienzo) de una palabra para modificar y completar su sentido.

De los listados de ejemplos citados, el numeral “c” contiene palabras con prefijos: pronombre – inseminar -
transportar. La opción “a”, en cambio, contiene sufijos: psicólogo, cleptomanía, herbívoro.

149
Tema 10. Elementos de la comunicación

Texto resumen

La comunicación es el proceso de intercambio de información entre un emisor y un receptor.


En este proceso intervienen varios elementos:

• Emisor: individuo, grupo social o máquina que transmite la información

• Receptor: individuo, grupo social o máquina que recibe la información

• Código: conjunto de signos que el emisor utiliza para transmitir el mensaje

• Canal: medio o elemento físico (natural o técnico) a través del cual el receptor transmite la
información

• Mensaje: información que se transmite.

• Contexto: circunstancias físicas, psicológicas, ambientales y socioculturales que rodean el


hecho o acto comunicativo y que permiten comprender el mensaje en su justa medida.

¿Cómo identificar los elementos de la comunicación en un texto? Veamos un ejemplo:

En una tira cómica de Quino, Mafalda y Miguel se encuentran estudiando, al parecer, en una
biblioteca. Miguel, un poco angustiado, le dice a Mafalda: “Nuestro derecho a la educación es
tan indiscutible que no hay la más mínima esperanza de que algún alma caritativa nos lo quite”.
Mafalda no dice nada, se queda con una mirada atónita pues todos sabemos que para ella la
educación es uno de los derechos más importantes.

En este caso, los elementos de la comunicación serían los siguientes:

• Emisor: Miguel

• Receptor: Mafalda

• Código: Lenguaje español

• Canal: Voz de Miguel

• Mensaje: “Nuestro derecho a la educación es tan indiscutible que no hay la más mínima
esperanza de que algún alma caritativa nos lo quite”.

• Contexto: La biblioteca en la cual Mafalda y Miguel se encuentran estudiando. Miguel, un


poco angustiado.

150
151
Referencias bibliográficas

• Ministerio de Educación de España (2007). Elementos de la comunicación. En:


http://recursos.cnice.mec.es/lengua/profesores/eso1/t1/teoria_1.htm

• Morales Acosta, Alex. Aprendiendo a identificar los elementos de la comunicación. En:


http://eduteka.icesi.edu.co/proyectos.php/1/7465

Banco de preguntas y respuestas

1)

Lea el siguiente texto e identifique al receptor:

Andrea se dirigió a su habitación y realizó una llamada. Te cuento que hoy he tenido un día horrible, dijo. Acabo
de despertar hace un rato y, como no logro volverme a dormir, he pensado que quizá conversando contigo se me
pase el tiempo; puede que incluso me dé sueño o que llegue Flora, mi hermana mayor, o Bruno, mi papá; no lo sé.
Como te decía, hoy ha sido un día terrible. Para empezar, el despertador se ha roto por culpa de Laura, mi prima,
que se ha enfadado al oír la alarma. ¿Te das cuenta Francisco? Así empezó mi día.

a) Laura
b) Francisco
c) Bruno
d) Andrea

Respuesta correcta: b)

En la comunicación interviene un emisor, un receptor, un código, un canal, un mensaje y un contexto. En el texto


citado, el emisor es Andrea, quien, a través de una llamada telefónica (canal) y haciendo uso de su lengua española
(código) transmitió a Francisco (receptor) varios acontecimientos que describieron su horrible inicio del día (mensaje).
El receptor, entonces, es Francisco.

2)

Identifique al receptor del texto:

El debate entre corazón y mente se ha extendido y ha generado una gran controversia. Catalina opina que lo más
importante es el corazón, por eso siempre se basa en las emociones a la hora de tomar decisiones. Miguel considera
que para él es más importante la mente y pensar bien las cosas. Liliana cree que lo mejor es que haya un equilibrio
entre mente y emociones. Felipe, ¿qué piensas?

a) Catalina
b) Felipe
c) Liliana
d) Miguel

Respuesta correcta: b)

En la comunicación interviene un emisor, un receptor, un código, un canal, un mensaje y un contexto. En el texto


citado, un “narrador” desconocido transmite a Felipe las opiniones de Catalina, Miguel y Liliana respecto del debate
entre corazón y mente. Este narrador funge de emisor. El receptor es Felipe, a quien, finalmente, dicho narrador le
pregunta qué piensa del tema.

152
3)

Tomando en cuenta el siguiente correo electrónico, relaciona el elemento de la comunicación con su ejemplo:

Para: Yoli, Eduardo, Lily, Anita, Adrianita, Elena Pazos


De: jvera@gmail.com
Asunto: CONVOCATORIA TALLERES
Fecha: viernes, 15 de marzo de 2013, 10:27 am

Estimados Señores (as) Rectores (as) Coordinadores (as): Este comunicado tiene la finalidad de convocar a los
talleres de la próxima semana a los docentes de las asignaturas según CRONOGRAMA DE CAPACITACION que les
envié en correos anteriores.

Los talleres se desarrollarán en Quito, Distrito Metropolitano, a las 08h00 del día lunes 18 de marzo de 2013, en las
instalaciones de la institución.

Saludos cordiales,
Lucía Vera López
TÉCNICO DOCENTE
Quito, Ecuador

1. Emisor a. Correo

2. Receptor b. Técnico docente

3. Canal c. Rectores

a) 1a, 2b, 3c
b) 1b, 2a, 3c
c) 1b, 2c, 3a
d) 1c, 2b, 3a

Respuesta correcta: c) 1b, 2c, 3a

En la comunicación interviene un emisor, un receptor, un código, un canal, un mensaje y un contexto. En el correo


electrónico citado, el emisor es el técnico docente (la persona que transmite el mensaje), los receptores son los
rectores (ellos reciben la información enviada) y el canal es el correo, por tratarse del medio físico a través del cual
se envía el mensaje.

4)

Identifique el contexto en el que Bronislaw Manilowski empezó a escribir Los Argonautas del Pacífico Occidental.

La génesis de este libro es singular por más de una razón. En 1914, a poco de estallar la Gran Guerra, Malinowski
se hallaba en Inglaterra, donde se había graduado hacía poco en Antropología por la London School of Economics.
Al ser súbdito austríaco –Cracovia, su ciudad natal, pertenecía entonces al Imperio austrohúngaro–, podía ser
llamado a filas en cualquier momento. A fin de evitar esa posibilidad, se embarcó hacia Nueva Guinea, en la
Melanesia. A este viaje le impulsaron también otros motivos, que él mismo explicó posteriormente: el tedio de la
vida académica, el deseo de aventura, el cansancio de las formas de vida que impone la civilización mecanizada.

En Nueva Guinea, Malinowski se instaló primero entre los indígenas de Mailu, donde realizó su primer trabajo de
campo. En mayo de 1915, una vez cumplidos los objetivos que se había propuesto, decidió cambiar de escenario.
Arribó entonces a las islas Trobriand, y en una aldea del distrito de Kiriwina, comenzó a recoger las primeras
informaciones de lo que al cabo de unos años, en 1922, sería Los Argonautas del Pacífico Occidental.

153
a) Islas Trobriand

b) Isla Mailu
c) Inglaterra
d) Imperio austrohúngaro

Respuesta correcta: a)

En la comunicación interviene un emisor, un receptor, un código, un canal, un mensaje y un contexto. El contexto


corresponde a las circunstancias físicas, psicológicas, ambientales y socioculturales que rodean el hecho o acto
comunicativo. El acto comunicativo, en este caso, corresponde a la escritura de las primeras notas de la obra Los
Argonautas del Pacífico Occidental a cargo del antropólogo Bronislaw Malinowski, quien vendría a ser el emisor. ¿Cuál
fue el contexto en que este autor escribió su obra? Claramente se dice que fueron las islas Trobriand, específicamente
en una aldea del distrito de Kiriwina.

5)

Identifique el contexto en el que Oscar Peterson decide tocar el piano:

Me despertó un sonido musical muy agradable, era la melodía del pianista que vive al lado de mi departamento. No
sé por qué decidió tocar su piano a las seis de la mañana. Era un día soleado y alegre. Me levanté inmediatamente y,
luego de ducharme y desayunar, decidí salir a caminar. Al abrir la puerta me topé con un hombre alto. Le pregunté:
“¿tu eres el pianista del edificio?” Sonriente me dijo: “Sí, siento mucho si te desperté”. Finalmente, los dos fuimos a
caminar. Hablamos mucho. Su nombre es Oscar Peterson, uno de los mejores pianistas de jazz.

a) El deseo de tocar el piano


b) La necesidad de salir a caminar temprano
c) El deseo de despertar a la vecindad
d) El día soleado y alegre

Respuesta correcta: d)

En la comunicación interviene un emisor, un receptor, un código, un canal, un mensaje y un contexto. El contexto son
las circunstancias físicas, psicológicas, ambientales y socioculturales que rodean el hecho o acto comunicativo. El acto
comunicativo, en este caso, corresponde a la ejecución de una melodía musical por parte de Oscar Peterson a las seis
de la mañana. ¿Cuál fue el contexto en que músico decidió tocar su piano? De las opciones citadas, la que guarda
coherencia con el párrafo citado es “el día soleado y alegre”.

154
Tema 11. Algunas reglas ortográficas

Texto resumen

Uso de la S

Se escriben con S:

• Los vocablos compuestos y derivados de otros que también se escriben con S. Ejemplos:
pesar / pesado, sensible / insensibilidad

• Las terminaciones -esa, -isa que signifiquen dignidades u oficios de mujeres. Ejemplos:
princesa, poetisa

• Los adjetivos que terminen en -aso, -eso, -oso, -uso. Ejemplos: escaso, travieso, perezoso,
difuso

• Las terminaciones -ísimo, -ísima. Ejemplos: altísimo, grandísima.

• Las palabras que terminan en -sión siempre y cuando se deriven de otras terminadas en -
sor o -sivo. Ejemplos: compresor / compresión, expresivo / expresión

• Las terminaciones de algunos adjetivos gentilicios singulares. Ejemplos: inglés, portugués,


francés, danés, irlandés.

• Las sílabas iniciales des- o dis-. Ejemplos: desinterés, discriminación.

• Las palabras que terminan en -esto o -esta. Ejemplos: detesto, orquesta

Uso de la C

El uso de la C es correcto en los siguientes casos:

• Cuando se usan las terminaciones -cito, -cita, -cillo, -cilla, -cecillo, -cecilla. Ejemplos:
pedacito, nochecita, calzoncillo, manecilla, pececillo o lucecilla.

• Cuando las palabras que en singular terminan en z y se transforman en palabras en plural.


Ejemplos: luz-luces, cruz-cruces, lápiz-lápices o capaz-capaces.

• Cuando las palabras terminan en -ción, siempre y cuando se deriven de otras terminadas
en -tor y no en -sor. Ejemplos: destructor-destrucción o director-dirección.

• En los infinitivos de los verbos que terminen en alguna de las voces -cer, -ceder, -cir, -
cendir, -cibir o -cidir. Ejemplos: cocer, conceder, decir o recibir. Excepción: asir y coser

• En todos los compuestos y derivados de las palabras que se escriban con c. Ejemplos:
cebolla-encebollado o ceja-cejijunto.

155
Uso de la V

Se escriben con V:

• Los adjetivos que terminan en -ave, -avo, -eva, -evo, -eve, -iva o -ivo. Ejemplos: grave, bravo,
nueva, activa. Excepción: árabe, las formas grabo, grabe del verbo grabar

• Los verbos andar, tener y estar en los tiempos pretérito indefinido del modo indicativo o
imperfecto del modo subjuntivo: anduve, anduviera, tuvieron, tuviéramos, estuvieron,
estuviésemos.

• El verbo ir en el tiempo presente del modo indicativo: voy, vas, va, vamos, vais, van.

• El verbo ir en modo Imperativo: ve, vaya, vayan

• El verbo ir en presente del modo subjuntivo: vaya, vayas, vaya, vayamos, vayáis, vayan

• Generalmente las palabras que comienzan con el sonido vi seguido de vocal. Ejemplos:
viento, violencia, viuda, viaje. Excepciones: cuando bi significa dos o doble (bienal) y cuando
bio significa vida (bioquímica).

• Las palabras que comienzan con las consonantes n y ll. Ejemplos: llave, nieve, llevar, nave.
Excepciones: nube, nebulosa, nabo.

• Las palabras que empiezan con las voces villa- o vice-. Ejemplos: villano, vicepresidente.
Excepción: billar.

Uso de la B

Se escriben con B:

• Palabras que combinen la letra m con la b. Ejemplos: tambor, mambo, temblor.

• Los verbos terminados en -bir. Ejemplos: escribir, describir, inscribir. Excepciones: hervir,
vivir y servir.

• Todos los verbos conjugados en pretérito imperfecto del modo indicativo, cuyos infinitivos
terminen en -ar así como el verbo ir conjugado en este tiempo. Ejemplos: terminaba,
caminábamos, saltabais, amabas iba, ibas, iba, íbamos, ibais, iban.

• Las palabras en las cuales la b anteceda a las letras l o r. Ejemplos: blanco, amable, brisa,
costumbre.

• Los compuestos y derivados de palabras que llevan esta misma letra. Ejemplos: cabeza y
cabezal, tubo y entubado, borracho y borrachera.

• Los vocablos que empiezan con bibli- o con las sílabas bu-, bur- o bus-. Ejemplos: bíblico,
Buda, burguesía, busto.

156
• Las palabras que empiecen con las sílabas ca-, ce-, co- y cu-. Ejemplos: cabello, cebada,
cobarde, cubo. Excepciones: caverna, cavar, cavilar, cavidad, caviar.

• Las palabras que empiecen con las sílabas al-, ar- y ur-. Ejemplos: alba, árbitro, urbe.

• Las palabras que empiecen con las sílabas sa-, si-, so- o su-. Ejemplos: sabor, Siberia,
soberbia, subasta. Excepciones: savia, soviético

• Las palabras que comienzan con bo y que van seguidas de las consonantes d, ch, f, n, r o t.
Ejemplos: boda, bofetada, bonachón, borde, botón, bochinche. Excepciones: voraz, votar,
vorágine.

• Cuando los sonidos bi, bis o biz tienen el significado de dos o doble. Ejemplos: bisílaba,
bisabuelo, bizco

• Las palabras que empiecen con las sílabas ab, ob o sub. Ejemplos: abdomen, obsequio,
subrayar.

• Las palabras con las terminaciones -bundo, -bunda, -bilidad o -ble. Ejemplos: meditabundo,
abunda, agradable, disponibilidad. Excepciones: movilidad, civilidad.

Uso de la J

Se escriben con J:

• Las palabras que tienen las siguientes combinaciones: aje, eje, ije, oje, uje. Ejemplos: ajeno,
ejecutivo, dije, ojeras, cuje. Excepciones: Las conjugaciones de los verbos cuyos infinitivos
terminan en ger o gir como: elegir, coger, proteger, estratagema, exagerar, tragedia, digerir,
sugerir y las palabras: cónyuge y vegetal. También son excepciones cuando después de dichas
combinaciones siguen las consonantes l, n o s como: flagelo, agente, digestivo.

• Las terminaciones -jero, -jera y -jería: conserjería, cerrajero, cerrajería, etcétera.

• Los tiempos de los verbos cuyo infinitivo lleva esa letra: cruje de crujir o trabaja de trabajar

• Las palabras derivadas de otras que llevan j: cajita de caja o herejía de hereje

• Las formas verbales con sonido je o ji, si los infinitivos correspondientes no llevan ni g ni j:
distrajimos de distraer, dedujimos de deducir, dijimos de decir, etcétera.

• Las formas verbales cuyos infinitivos terminan en -ger o -gir en primera persona del singular
del presente del modo indicativo. Ejemplo: proteger / protejo, dirigir / dirijo. También en las
formas usted y ustedes del modo imperativo. Ejemplo: proteja, protejan / dirija, dirijan. Y
también en yodas las personas del tiempo presente del modo subjuntivo. Ejemplo: proteja,
protejas, proteja, protejamos, protejáis, protejan, dirija, dirijas, dirija, dirijamos, dirijáis,
dirijan.

157
Uso de la G

Se escriben con G:

• Las palabras que inician con la sílaba geo-. Ejemplos: geométrico, geología.

• Las palabras que contienen el sonido gen en cualquier posición (inicial, intermedia o final).
Ejemplos: gente, agente, imagen. Excepciones: avejentar, berenjena, ajeno.

• Los sonidos ge y gi de los verbos cuyos infinitivos terminan en –ger o –gir. Ejemplos:
recogemos de recoger, dirigimos de dirigir. Excepciones: crujir, tejer.

• Los compuestos y derivados de las palabras que se escriben con g, también se escriben con
esta letra. Ejemplos: general, generalizado, generalmente.

• Las palabras que tienen las siguientes terminaciones: -génico, -genario, -geneo, -genio, -
genito, -gesimal, -gésimo, -gético, -giénico, -gía, -ginal, - gineo, -ginoso -gismo, -gio, -gión, -
gionario, -gioso, -gírico, -ógica -ígena, -ígeno, -igero. Ejemplos: ginecología, morfología,
angélico, regia, plagio, región, trigésimo, honagenario, original, ligero, neologismo.
Excepción: espejismo.

Referencias bibliográficas

• Todo sobre Gramática. En: http://www.deperu.com/abc/gramatica/752/

Banco de preguntas y respuestas

1)

Seleccione las oraciones que utilizan correctamente la b y la v

1. El profesor afirmó, furibundo, que absolutamente todos los conejos son herbívoros
2. No creía ser un hervíboro, por lo que se pudo furivundo cuando vió que solo había ensalada
3. Pusimos a herbir el té y cortamos el pan para los bocadillos. Era la fiesta para el anciano más longebo del
pueblo
4. No pensó ser tan longevo, reflexionó mucho al respecto mientras ponía a hervir el agua para prepararse un
café

a) 1, 2
b) 1, 4
c) 2, 3
d) 3, 4

Respuesta correcta:

La respuesta correcta es b) 1, 4.

158
Las oraciones 2 y 3 contienen palabras con errores en el uso de la “b” y la “v”: “hervíboro” en lugar de “herbívoro”;
“furivundo” en lugar de “furibundo”, “herbir” en lugar de “hervir”, y “longebo” en lugar de “longevo”.

Las oraciones 1 y 4 están escritas correctamente.

2)

Identifique la oración en la que se emplea de forma adecuada la letra J

a) Luis llegó ayer, tras varios años de vivir en el extranjero


b) El periodo de jestación varía mucho de una especia a otra
c) Sentía como su un ente malijno habitara en la casa
d) Tuvo que imprejnar el algodón con agua oxigenada

Respuesta correcta: a)

En 3 oraciones se incluye la J en lugar de la G: “jestación” en lugar de “gestación”, “malijno” en lugar de “maligno” e


“imprejnar” en lugar de “impregnar”.

La siguiente opción está escrita de forma correcta: “Luis llegó ayer, tras varios años de vivir en el extranjero”.

3)

Identifique la oración en la que se emplea de forma adecuada la letra J

a) Los tablones de madera crujieron toda la noche


b) El infante hospitalizado contajió a su madre con gripe
c) El docente corrijió los exámenes de séptimo año de básica
d) Los rescatistas protejieron a los damnificados del huracán

Respuesta correcta: a)

En 3 oraciones se incluye la J en lugar de la G: “contajió” en lugar de “contagió”, “corrijió” en lugar de “corrigió” y


“protejieron” en lugar de “protegieron”.

La siguiente opción está escrita de forma correcta: “Los tablones de madera crujieron toda la noche”.

159
Tema 12. Léxico en Ecuador: quichuismos, arcaísmos, extranjerismos y neologismos

Texto resumen

El léxico es el conjunto de palabras que conforman una determinada modalidad o variante


lingüística. En Latinoamérica se habla español. El español procede del latín y, a lo largo del
tiempo, ha sufrido las más variadas influencias, de los pueblos germánicos, de los árabes y de
las lenguas originarias americanas.

De manera particular, el léxico de Ecuador se caracteriza por emplear préstamos, quichuismos,


arcaísmos, extranjerismos y neologismos

Préstamos y quichuismos

Los préstamos son términos de otras lenguas que se han incorporado al español y tienen
entrada en el diccionario. El español de América tiene muchos préstamos de las lenguas de los
indígenas que poblaron sus territorios antes llegada de los españoles. Aquí unos ejemplos:

• cacique (de lengua caribe)


• chicle (del náhuatl)

• canoa (del taíno)
• cigarro (del maya)

• huracán (del taíno)
• iguana (del arahuaco antillano)
• chocolate (del náhuatl)
• papa (del quichua)

• aguacate (del náhuatl)
• puma (del quichua)

Hay préstamos del inglés o del francés que, con los años, también han sido aceptados por la
Real Academia de la Lengua Española. Por ejemplo: rock, pizza, blues, bluyín, champú, sándwich,
béisbol, clip o chip.

Sin embargo, el español del Ecuador tiene algunas particularidades, pues existen ciertos
quichuismos que solo son empleados dentro del país. Los quichuismos son préstamos de la
lengua quichua que están integrados en el español del Ecuador, y especialmente en el español
de la región Interandina38. Veamos algunos ejemplos de quichuismos utilizados en nuestro país:

• achachay (¡qué frío!)


• atatay (¡qué feo!)
• arraray (¡que caliente!)
• canguil (maíz para rosetas)
• carishina (machona)
• chacra (pedazo de tierra)
• chapa (policía)

38 Ministerio de Educación (2016). Lengua y Literatura. Primer Año del BGU. Texto para el estudiante. Quito: UASB, página 167.

160
• chaquiñán (atajo)
• chugchucara (plato típico)

• chulla (impar)
• guagua (niño o niña)
• guango (trenza)
• huasipungo (pequeño terreno)

• llucho (desnudo)

• mote (maíz)
• pampa (llanura)
• pupo (ombligo)
• yahuarlocro (sopa típica)

Existen quichuismos como préstamos puros y quichuismos adaptados al español. Por ejemplo,
palabras como chacra, guagua, pampa, yahuarlocro, achachay, arraray, atatay, changa o
carishina son calcadas de la lengua quichua, mientras que amarcar (del quichua marcana),
poroto (de purutu) y cancha (de kancha) se han adaptado para que empaten con las formas del
español. No todas están registradas en el diccionario de la lengua española.

Ñaña es la palabra que nosotros hemos adquirido del quichua. Ésta, si bien es un calco, también
la hemos adaptado para referirnos a un hermano, cambiando el género de la palabra a ñaño. En
kichwa, para referirse a un hermano se usa wawki si se lo hace de varón a varón y turi si se lo
hace de mujer a varón.

Arcaísmos

Los arcaísmos son términos del español antiguo que se utilizan como si fuesen del lenguaje
actual aun cuando, para las generaciones jóvenes, suenen anticuados. Por ejemplo, en Cuenca
(provincia del Azuay) se emplea la frase: Vino el señor donoso. Donoso tiene la acepción de
guapo.39 Otros ejemplos de arcaísmos son: adrede (apropósito o con intención), candela (vela
de cera), cuasi (casi), orear (secar), sopapo (bofetada) y tunda (paliza).

Extranjerismos

Los extranjerismos son palabras tomadas de otra lengua (principalmente del inglés y del
francés) cuyo uso es innecesario e incorrecto, pues el castellano dispone de un término propio
para nombrar la realidad a la que se refieren. A diferencia de los préstamos y quichuismos, los
extranjerismos no están registrados en el diccionario de la lengua española. Cuando los
utilizamos, su escritura no se altera y suelen escribirse en cursiva. Por ejemplo: hall en lugar de
vestíbulo, man en lugar de hombre, boutique en lugar de tienda de ropa, look en lugar de estilo,
brother en lugar de hermano o amigo, bye en lugar de adiós, block en lugar de libreta, all right
en lugar de “de acuerdo” y camping en lugar de acampada.

Introducción de neologismos

Se llaman neologismos a las palabras nuevas que surgen por el uso cotidiano o son formadas
intencionalmente para poder designar las cosas; su uso es muy extenso, abarcando

39 Ministerio de Educación (2016). Lengua y Literatura. Décimo Año. Texto para el estudiante. Quito: UASB.

161
principalmente a la ciencia y las tecnologías, lo que nos da como resultado muchas palabras
como son:

• Laptop
• Notebook
• Tablet
• Smartphone

La palabra neologismo proviene del griego “neo” (nuevo) y “logo” (palabra), indicando que se
trata de una nueva palabra que es introducida al léxico de un idioma. Algunos neologismos
suelen ser aceptados por la Real Academia de la Lengua como “chatear” o “zum”.40

Referencias bibliográficas

• Diccionario de la Lengua Española. En: http://dle.rae.es/

• Ejemplo de Neologismos. En: http://www.ejemplode.com/12-clases_de_espanol/3694-


ejemplo_de_neologismos.html

• Ministerio de Educación (2016). Lengua y Literatura. Décimo Año. Texto para el estudiante. Quito: UASB.

• Ministerio de Educación (2016). Lengua y Literatura. Primer Año del BGU. Texto para el estudiante. Quito:
UASB.

Banco de preguntas y respuestas

1)

Los préstamos son términos de otras lenguas que se han incorporado al español y tienen entrada en el diccionario.
El español de América tiene muchos préstamos de las lenguas nativas y de lenguas extranjeras como el inglés y el
francés.

Seleccione las oraciones que utilicen préstamos de las lenguas nativas:

1. El día de ayer mi hermano fumó un cigarro bastante fuerte


2. Definitivamente el blues no un ritmo musical que me encante
3. El chocolate que preparé ayer estuvo muy espeso
4. Yo no me lavo mi cabello con champú sino con un jabón suave que no contiene sal

a) 1, 2
b) 1, 3
c) 2, 3
d) 2, 4

Respuesta correcta: b)

En dos de las oraciones citadas se emplean palabras que se originan de lenguas nativas: cigarro, del idioma maya, y
chocolate, del idioma náhuatl.

40 Ejemplo de Neologismos. En: http://www.ejemplode.com/12-clases_de_espanol/3694-ejemplo_de_neologismos.html

162
2)

De la lista de palabras, seleccione los quichuismos.

1. Escuela
2. Cancha
3. Chacra
4. Libros
5. Guagua
6. Pampa
7. Borrador

a) 1, 2, 3, 6
b) 1, 3, 4, 7
c) 2, 3, 5, 6
d) 2, 4, 5, 7

Respuesta correcta: c) 2, 3, 5, 6.

Quichuismos son palabras de origen quichua que, a lo largo del tiempo, lograron ser asimiladas dentro del lenguaje
español. Del listado de palabras citadas, 4 son de origen quichua:

• Cancha viene del término quichua kancha que, traducido al español, significa “recinto”. El vocablo se utiliza para
referirse al espacio reservado para la práctica de distintos deportes o para la organización de espectáculos.
• Chacra significa pedazo de tierra
• Guagua significa niño o niña
• Pampa designa cualquier llanura o planicie

3)

Seleccione las oraciones que utilicen quichuismos:

1. Las iglesias coloniales tienen pinturas bastante elaboradas.


2. Qué más da. Ya no hay chance de hacer lo que quería.
3. Mi guagua salió contenta de la fiesta de su mejor amiga.
4. ¡Achachay! El frío de Quito está incontrolable.

a) 1, 2
b) 1, 4
c) 2, 3
d) 3, 4

Respuesta correcta: d) 3, 4.

En dos de las oraciones citadas se emplean palabras quichuismos: guagua (niño o niña) y achachay (¡qué frío!).

4)

Seleccione las oraciones que utilicen arcaísmos:

1. La policromía es una técnica muy antigua


2. Lo sabía, ¡lo hiciste adrede!
3. Qué perfumado está aquel señor donoso
4. No entiendo por qué le dices a tu alumno que es vago

a) 1, 2
b) 1, 4
c) 2, 3
d) 3, 4

163
Respuesta correcta: c) 2, 3.

En dos de las oraciones citadas se emplean arcaísmos: adrede (a propósito) y donoso (guapo).

5)

Relacione los quichuismos y extranjerismos con sus ejemplos.

a. Chamiza, guagua, charco, shigra, changa, chambra

1. Quichuismo b. Poster, body, man, ok, boom, cocktail

2. Extranjerismo c. Cancha, chuspa, guambra, carishina, mishqui, mucha

d. Full, lunch, coffee break, shopping

a) 1ab, 2dc
b) 1ac, 2bd
c) 1bc, 2ad
d) 1bd, 2ac

Respuesta correcta: b) 1ac, 2bd

Quichuismos son palabras de origen quichua que, a lo largo del tiempo, lograron ser asimiladas dentro del lenguaje
español. Extranjerismos, en cambio, son palabras de origen extranjero -en nuestro caso, de Estados Unidos- que
también han sido asimiladas como parte del lenguaje coloquial.

• Chamiza, guagua, charco, shigra, chambra o changa, cancha, chuspa, carishina mucha y mishqui son quichuismos.
• Extranjerismos, en cambio, son: poster, body, man, ok, boom, cocktail, full, lunch, coffee break.

6)

Seleccione las oraciones que utilicen neologismos:

1. El ratón de la computadora no está funcionando correctamente


2. Este momento no puedo atenderte. Mejor chateamos en la tarde.
3. Pasé por el Mall y me compré un bluyín
4. En ese evento no ofrecieron un coffee break

a) 1, 2
b) 1, 4
c) 2, 3
d) 3, 4

Respuesta correcta: a) 1, 2.

Se llaman neologismos a las palabras nuevas que surgen principalmente por el uso cotidiano de aparatos
tecnológicos. En dos de las oraciones citadas se emplean neologismos: “ratón”, para referirse a un instrumento de la
computadora (mouse en inglés), y “chateamos”, para referirse a la acción de mantener una conversación mediante
chats.

164
Tema 13. Uso adecuado de lo (s), la (s) y le (s)

Texto resumen

Para usar adecuadamente los pronombres átonos de tercera persona lo (s), la (s), le (s) deben
tenerse en cuenta las siguientes normas:

• Cuando el pronombre desempeña la función de complemento directo, deben usarse las


formas lo y los para el masculino (singular y plural, respectivamente) y la y las para el
femenino (singular y plural, respectivamente). Por ejemplo:

✓ ¿Has visto a Jorge? Sí, lo vi ayer.


✓ ¿Has visto a Jorge y a sus hijos? Sí, los vi en el parque.
✓ ¿Has encontrado a tu hija? Sí, la encontré debajo de su cama.
✓ La comida tenía un mal sabor. Por eso, no la ingerí.
✓ ¿Has recogido a las niñas? Si, las recogí antes de ir al taller.

• Cuando el pronombre desempeña la función de complemento indirecto, deben usarse las


formas le y les (singular y plural, respectivamente), con independencia del género de la
palabra a la que se refiera:

✓ Le pedí disculpas a mi madre


✓ Le dije a su hermana que viniera
✓ Les di un regalo a los niños

Cuadro resumen del uso correcto de lo (s), la (s) y le (s)

Complemento Singular Plural

lo
masculino (también le, cuando el referente es un los
hombre)

Directo femenino la las

neutro lo -

le les
Indirecto
(o se ante otro pronombre átono) (o se ante otro pronombre átono)

Fuente: Real Academia de la Lengua Española.


En: http://www.rae.es/consultas/uso-de-los-pronombres-los-las-les-leismo-laismo-loismo

165
El leísmo

El leísmo es la sustitución de los pronombres personales lo y la por le en la posición de


complemento directo. Se condena el leísmo referido a cosa, pero se permite cuando se
refiere a una persona masculina singular. El leísmo plural siempre ha sido censurado por la
Academia. Por ejemplo, es correcto decir: ¿Has visto a Jorge? Sí, le vi ayer. Pero es incorrecto
decir: ¿Has visto a Jorge y a sus hijos? Sí, les vi en el parque. Y también es incorrecto decir:
La comida tenía un mal sabor. Por eso, no le ingerí.

Referencias bibliográficas

• Real Academia de la Lengua Española. En: http://www.rae.es/consultas/uso-de-los-pronombres-los-las-les-


leismo-laismo-loismo

Banco de preguntas y respuestas

1)

Elige los enunciados que presentan leísmo.

1. Yo le di una serenata para demostrar el cariño que siento por ella


2. Para que las niñas no se aburrieran, les llevé a pasear al parque
3. Los muchachos salieron al campo, yo les pedí que no se alejaran
4. Hoy visité a los miembros del grupo de rescate, les vi preocupados
5. Caminé con la carta en la mano, luego le puse en el buzón de Julia

a) 1, 2, 4
b) 1, 3, 4
c) 2, 3, 5
d) 2, 4, 5

Respuesta correcta: d) 2, 4, 5.

De las opciones citadas, tres emplean le o les en la posición de complemento directo; es decir, utilizan leísmo de
forma incorrecta:

• Para que las niñas no se aburrieran, les llevé a pasear al parque (debe decir “las llevé”)
• Hoy visité a los miembros del grupo de rescate, les vi preocupados (debe decir “los vi”)
• Caminé con la carta en la mano, luego le puse en el buzón de Julia (debe decir “la puse”)

Se acepta el leísmo en la posición de complemento directo únicamente cuando se refiere a una persona masculina
singular. Por ejemplo: Hoy visité a Juan, le vi preocupado.

2)

Seleccione las frases que utilizan adecuadamente los pronombres átonos de tercera persona lo (s), la (s) o le (s):

166
1. A María le vieron por la calle
2. Dile por favor a Juan que lo quiero mucho
3. A Jorge y a María les vieron en el parque
4. No sé qué decirte. A ella la encontré ya sin vida.

a) 1, 3
b) 1, 2
c) 2, 3
d) 2, 4

Respuesta correcta: d) 2, 4.

De las opciones citadas, dos emplean correctamente los pronombres átonos:

• Dile por favor a Juan que lo quiero mucho. El pronombre desempeña la función de complemento directo,
usándose correctamente la forma “lo” para el masculino en singular.
• No sé qué decirte. A ella la encontré ya sin vida. El pronombre desempeña la función de complemento directo,
usándose correctamente la forma “la” para el femenino en singular.

3)

Seleccione las frases que utilizan adecuadamente los pronombres átonos de tercera persona lo (s), la (s) o le (s):

1. Les leí un cuento a los niños


2. La dije a su cuñada que se quedara
3. Le dije a su hermana que viniera
4. La comida tenía un mal sabor. Por eso, no le ingerí.

a) 1, 3
b) 1, 2
c) 2, 3
d) 2, 4

Respuesta correcta: a) 1, 3.

De las opciones citadas, dos emplean correctamente los pronombres átonos:

• Les leí un cuento a los niños


• Le dije a su hermana que viniera

En ambos casos, el pronombre desempeña la función de complemento indirecto y por eso se emplean las formas les
y le, respectivamente, con independencia del género de la palabra a la que se refieren.

167
Tema 14. Figuras literarias

Texto resumen

«Las figuras literarias, también conocidas como figuras retóricas, son formas no convencionales
de emplear las palabras para dotarlas de expresividad, vivacidad o belleza, con el objeto de
sorprender, emocionar, sugerir o persuadir».41 Las figuras literarias son típicas de los textos
literarios, pero no son exclusivas de éstos pues pueden emplearse inclusive en nuestro lenguaje
coloquial.

A continuación, detallamos algunas de las figuras literarias más utilizadas:

Metáfora

La metáfora es una figura literaria que se usa para identificar una analogía o semejanza entre
dos ideas o imágenes. Una idea o imagen es literal, otra se usa en sentido figurado. Por ejemplo:

Tus cabellos de oro


(para referirse a los cabellos rubios).

Símil o comparación

El símil o comparación consiste en establecer una relación de semejanza entre dos elementos a
través de un nexo relacional explícito (como, cual, igual que). Por ejemplo:

... murmullo que en el alma


se eleva y va creciendo
como volcán que sordo
anuncia que va a arder
(Gustavo Adolfo Bécquer).

41 Qué son las figuras literarias. En: https://www.significados.com/figuras-literarias/

168
Hipérbole

La hipérbole es una figura literaria que consiste en exagerar las cualidades, características o
costumbres de personas, lugares, animales u objetos. Por ejemplo:

Dame, señor, todas las lágrimas del mar para


llorarte por toda una eternidad (Garcilaso de la
Vega).

Repetición

La repetición es un recurso literario que consiste en el empleo repetitivo, en un mismo texto, de


fonemas, sílabas, morfemas, palabras o frases. 42

Triste, triste, triste, es mi vida errante,


duro, duro, duro, el camino agreste,
vana, vana, vana, e inútil la esperanza
de encontrar al final de mí camino,
el dulce, dulce, dulce, mirar de ella,
que nada, nada, nada, pero nada me
ama (Anónimo).

Aliteración

La aliteración consiste en la repetición de un mismo sonido o sonidos similares, sobre todo


consonánticos, en una misma frase u oración, con la finalidad de producir cierto efecto sonoro
en la lectura. Suele ser usada en trabalenguas y versos. 43 Por ejemplo:

42 Tomado de: http://www.ejemplode.com/11-escritos/252-ejemplo_de_repeticion.html


43 Ejemplos tomados de: http://www.ejemplode.com/11-escritos/254-ejemplo_de_aliteracion.html

169
La tempestad, el ruido con que
rueda la ronca tempestad
(José Zorrilla).

Hipérbaton

El hipérbaton es una figura literaria en la cual el orden convencional de las palabras en la oración
es alterado por razones expresivas o, en el caso de la poesía, para ajustarlo a la métrica, el ritmo
o la rima de la frase. Por ejemplo:

A Dios gracias
(en lugar de gracias a Dios).

Paradoja

La paradoja implica el uso de expresiones, ideas, conceptos o frases en las cuales hay una
supuesta contradicción que, en realidad, tiene la intención de enfatizar o darle un nuevo sentido
a aquello de lo que habla. Por ejemplo:

Si anhelas la paz,
prepárate para la guerra.

Onomatopeya

La onomatopeya es la representación escrita de un sonido.

El chasquido del látigo me asustó.


Corrí para ver que ocurría.

170
Prosopopeya

La prosopopeya o personificación es una figura literaria que consiste en atribuir cualidades o


acciones propias de seres humanos a animales, objetos o ideas abstractas.

Los invisibles átomos del aire


en derredor palpitan y se inflaman
(Rimas, Gustavo Adolfo Bécquer).

Oxímoron

El oxímoron es una figura retórica de pensamiento que consiste en la utilización de dos


conceptos de significados contrarios dentro de la misma estructura sintáctica. Debido a la
incoherencia de la expresión en sentido literal, ya que son sentidos opuestos, se fuerza al lector
a pensar en el sentido metafórico que incluye la frase.

Apresúrate despacio, amor mío...


(Ismael Serrano).

Referencias bibliográficas

• Qué son las figuras literarias. En: https://www.significados.com/figuras-literarias/

• Mil ejemplos de lenguaje. En: https://www.milejemplos.com/lenguaje/ejemplos-de-oximoron.html

• Ministerio de Educación (2016). Lengua y Literatura. Noveno Grado. Texto para el estudiante. Quito: UASB.

171
Banco de preguntas y respuestas

1)

Con base en el sintagma subrayado, identifique el elemento metaforizado.

… El mar y sus caracolas


te hacen soñar,
cerrando los ojos
no necesitas más.
El mar y sus arcos de diamante
que se rompen sin cesar.
Viento, arena y mar
Juegan sin parar

a) Caracolas
b) Olas
c) Viento
d) Arena

Respuesta correcta: b)

La metáfora es un recurso literario que se usa para identificar a dos términos entre los cuales existe alguna clase de
semejanza.

El verso citado recurre a la metáfora “arcos de diamante” para referirse a las “olas” pues éstas tienen forma de arcos
y, al estrellarse, se tornan blancas, como los diamantes de máxima pureza. En suma: las olas son como arcos de
diamante.

2)

Con base en el sintagma subrayado, identifique el elemento metaforizado.

La alegría del amanecer


rompe la calma del mar.
En un vaivén eterno
de arcos de diamante
se rompe fugaz su silencio

a) Amanecer
b) Olas
c) Vaivén
d) Arena

Respuesta correcta: b)

La metáfora es un recurso literario que se usa para identificar a dos términos entre los cuales existe alguna clase de
semejanza. El verso citado recurre a la metáfora “arcos de diamante” para referirse a las “olas” pues éstas tienen
forma de arcos y, al estrellarse, se tornan blancas, como los diamantes de máxima pureza. En suma: las olas son como
arcos de diamante.

3)

¿Cuál es el color de los ojos de la mujer que se describe en el fragmento?

172
Mujer con ojos de pradera
Mujer con ojos de bosque
Ojos firmes con armadura dorada
Y agujas imantadas

a) Café
b) Dorado
c) Verde
d) Gris

Respuesta correcta: c)

Las metáforas “ojos de pradera” y “ojos de bosque” nos permite comparar el color de los ojos con el color de la
pradera o del bosque. ¿Qué color tiene una pradera o un bosque (conjunto de árboles)? Por lo general, VERDE. El
verso “ojos firmes con armadura dorada” nos remite a esos ojos verdes que llevan, posiblemente, unos lentes
(armadura) de color dorado.

4)

Con base en el fragmento, identifique el color metaforizado

Ojos de bosque
que entre esmeralda y vida
su mirada dirige al azul del cielo.
Ojos de aceituna
que con armadura violeta
se esconden bajo
el negro profundo
de pestañas alegres.

a) Negro
b) Violeta
c) Verde
d) Azul

Respuesta correcta: c)

La metáfora es un recurso literario que se usa para identificar a dos términos entre los cuales existe alguna clase de
semejanza. Las metáforas del fragmento son “ojos de bosque” y “ojos de aceituna”. En este caso, debemos inferir de
qué color son los ojos si son comparables con un bosque y con una aceituna. Por lo general, los bosques (conjunto de
árboles) y las aceitunas son de color verde. Ergo, los ojos son de color verde.

5)

Con base en la frase subrayada, identifique el elemento metaforizado.

Abejas cantoras,
tus alitas de cristal
cuando sales del panal
zumban alegre a la aurora.

Eres tú, madrugadora,


la que tras el polen vas,
buscando aquí, y por allá…
acariciando las flores;
combinando mil sabores
la sabrosa miel nos das

173
a) Transparencia
b) Densidad
c) Tonalidad
d) Coloración

Respuesta correcta: a)

La expresión “alitas de cristal” podría referirse a que las alas de abeja son “transparentes como un cristal”. No son
densas ni coloridas ni tienen una tonalidad especial; sí suenan cuando zumban, pero ese sonido no es comparable
con un cristal. El término “cristal” tiene que ver, definitivamente, con la transparencia.

6)

Con base en el verso subrayado, ¿a qué aspecto de las alas se refiere la metáfora?

Abejas cantoras, no a la miel,


sino a las flores […]
con sus alas
recién hechas de agua […]
volad a las flores

a) Transparencia
b) Densidad
c) Tonalidad
d) Coloración

Respuesta correcta: a)

La expresión “con sus alas recién hechas de agua” podría referirse a que las alas de abeja son “transparentes como el
agua”. No son densas ni coloridas ni tienen una tonalidad especial. El término “hechas de agua” tiene que ver,
definitivamente, con la transparencia.

7)

Lea el párrafo y relacione los versos del poema con las figuras literarias correspondientes

Danse D´anitra

Va ligera, va pálida, va fina,


cual si una alada esencia poseyere.
Dios mío, esta adorable danzarina,
se va a morir, va a morir… se muere.

Tan aérea, tan leve, tan divina,


se ignora si danzar o volar quiere;
y se torna su cuerpo un ala fina,
cual si el soplo de Dios la sostuviere.

Sollozan perla a perla cristalina,


las flautas en ambiguo miserere…
Las arpas lloran y la guzla trina…
¡Sostened a la leve danzarina,
porque se va a morir… porque se muere!

174
Figuras literarias:

1. Hipérbole
2. Repetición

Versos:

a. Las arpas lloran y guzla trina


b. porque se va a morir… porque se muere
c. Las flautas en ambiguo miserere
d. tan aérea, tan leve, tan divina
e. cual si el soplo de Dios la sostuviere
f. va ligera, va pálida, va fina

a) 1abf, 2cde
b) 1ace, 2bdf
c) 1bce, 2adf
d) 1bdf, 2ace

Respuesta correcta: b) 1ace, 2bdf

La hipérbole es un recurso literario que consiste en exagerar las cualidades, características o costumbres de personas,
lugares, animales u objetos. En el poema Danse D´anitra se recurre a varias hipérboles: las arpas lloran y guzla trina
(exageración del sonido de las arpas y de la guzla), las flautas en ambiguo miserere (exageración de la ambigüedad
de las flautas) y cual si el soplo de Dios la sostuviere (exageración del movimiento del cuerpo de la danzarina).

La repetición, en cambio, es un recurso literario que consiste en el empleo repetitivo, en un mismo texto, de fonemas,
sílabas, morfemas, palabras o frases. Repeticiones son, por ejemplo: porque se va a morir… porque se muere; tan
aérea, tan leve, tan divina; y va ligera, va pálida, va fina.

8)

Identifique la figura literaria que se emplea en el cuarteto.

¡Ya viene el cortejo!



¡Ya viene el cortejo!
Ya se oyen los claros clarines,

la espada se anuncia con vivo reflejo;

ya viene, oro y hierro, el cortejo de los paladines


Rubén Darío, Marcha triunfal.

a) Símil
b) Aliteración
c) Prosografía
d) Hipérbaton

Respuesta correcta: b)

La aliteración consiste en la repetición de un mismo sonido o sonidos similares, sobre todo consonánticos, en una
misma frase u oración con la finalidad de producir cierto efecto sonoro en la lectura: claros clarines, y cortejo de los
paladines, por ejemplo.

El cuarteto de Rubén Darío también emplea el recurso literario conocido como repetición: empleo repetitivo, en un
mismo texto, de fonemas, sílabas, morfemas, palabras o frases: ¡Ya viene el cortejo! ¡Ya viene el cortejo!, por ejemplo.
Sin embargo, esta figura literaria no consta dentro de las opciones de respuesta.

175
9)

Seleccione la frase que emplea la figura literaria llamada Hipérbole:

a) Dame, Señor, todas las lágrimas del mar para llorarle por toda una eternidad
b) ¡Oh, soledad, que a fuerza de andar sola se siente de sí misma compañera!
c) Estaba todo en silencio, solo se oía el tictac de un reloj
d) Ven, ven, ven y quédate conmigo

Respuesta correcta: a)

La hipérbole es una figura literaria que consiste en exagerar las cualidades, características o costumbres de personas,
lugares, animales u objetos. De las opciones citadas, una de ellas expresa exageración: Dame, Señor, todas las
lágrimas del mar para llorarle por toda una eternidad. ¿Se puede llorar toda una eternidad? Por supuesto que no.
Hay una exageración de la acción de una persona.

10)

Identifique qué figura literaria se emplea en la siguiente frase:

La noche estaba oscura y silenciosa. Solo escuchábamos el tic tac del reloj que colgaba en la pared de la sala.

a) Símil
b) Onomatopeya
c) Prosografía
d) Hipérbaton

Respuesta correcta: b)

La onomatopeya es la representación escrita de un sonido. Así, en esta frase se dice “escuchábamos el tic tac del reloj
que colgaba en la pared de la sala”.

11)

Identifique la frase en la cual se emplea que se emplea una paradoja como figura literaria:

a) Vivo sin vivir en mí, y tan alta vida espero que muero porque no muero (Santa Teresa)
b) El coche se quejaba adolorido por los años (Leopoldo Alas)
c) Porque te miro y muero (Mario Benedetti)
d) Verde nativo, verde de yerba que sueña, verde sencillo (Miguel de Unamuno)

Respuesta correcta: a)

La paradoja implica el uso de expresiones, ideas, conceptos o frases en las cuales hay una supuesta contradicción
que, en realidad, tiene la intención de enfatizar o darle un nuevo sentido a aquello de lo que habla. Por ejemplo:
“Vivo sin vivir en mí, y tan alta vida espero que muero porque no muero” (Santa Teresa).

176

También podría gustarte